SMB Week 1

अब Quizwiz के साथ अपने होमवर्क और परीक्षाओं को एस करें!

A 57-year-old man comes to the emergency department due to a 1-day history of pain and swelling in his right knee. The pain is severe, and he has had difficulty bearing weight on the right leg. He previously felt well. Past medical history is notable for hypertension and seasonal allergies. The patient does not use tobacco and drinks 1 or 2 alcoholic beverages a week. On examination, the right knee is red and swollen. He has pain with passive range of motion at the knee. The knee ligaments show no significant laxity. Synovial fluid obtained from the involved joint shows a white blood cell count of 25,000/mm3, with a neutrophilic predominance, and numerous crystals (shown below). ID=1451 These crystals are most likely composed of which of the following substances? Calcium hydroxapatite Calcium oxalate Calcium pyrophosphate Cholesterol Monosodium urate Uric acid

Pseudogout, or calcium pyrophosphate deposition disease (CPPD), results from accumulation of calcium pyrophosphate dihydrate crystals in the synovial fluid. Patients present with an acute mono- or oligoarticular arthritis characterized by pain, joint swelling, erythema, and warmth. Pseudogout and gout can be difficult to distinguish clinically. The knee joint is involved in >50% of pseudogout cases, whereas the first metatarsophalangeal joint is more frequently involved in gout. In pseudogout, synovial fluid analysis reveals an elevated white blood cell count with neutrophilic predominance. Identification of rhomboid-shaped calcium pyrophosphate dihydrate crystals is diagnostic. These crystals are positively birefringent under compensated polarized light, meaning that the color pattern is the opposite of that seen in gout. Pseudogout crystals are blue when aligned parallel and yellow when aligned perpendicular to the slow ray of the compensator. (Choice A) Calcific tendonitis results from the deposition of calcium hydroxyapatite crystals in periarticular soft tissues (especially tendons). The rotator cuff tendons are most commonly affected. (Choice B) Calcium oxalate is one of the most common constituents of renal calculi. (Choice D) Soft-tissue deposits of cholesterol and other lipids are called xanthomas. These appear as yellow papules on the knees, elbows, and tendon insertion sites in patients with hyperlipidemia. (Choices E and F) In gout, monosodium urate (the salt of uric acid) is deposited in joints and soft tissues. Urate crystals are needle-shaped and negatively birefringent. Educational objective: Synovial fluid analysis showing rhomboid-shaped calcium pyrophosphate dihydrate crystals is diagnostic of pseudogout. These crystals are positively birefringent under compensated polarized light. The knee joint is involved in >50% of cases.

A 49-year-old man comes to the office due to "aching bones." He has a 2-month history of insidious-onset pain that is most pronounced in the back, pelvis, and lower extremities. The pain is dull and increases after weight-bearing activities. The patient has no prior medical conditions and takes no medications. He emigrated from Central Africa 5 years ago and works overnight shifts as a cab driver. Vital signs are within normal limits. Physical examination shows normal muscle strength in the upper and lower extremities bilaterally. A thorough laboratory evaluation establishes the diagnosis. After discussing the likely cause of his condition, the patient starts spending more time outdoors in the sun. Which of the following enzymatic steps will most likely be affected by this change in activity? ID=990 7-dehydrocholesterol> cholecalciferol cholecalciferol> 25-hydroxyvitamin D3 25-hydroxyvitamin D3> 1,25-dihydroxyvitamin D3 25-hydroxyvitamin D3> 24,25-dihydroxyvitamin D3 1,25-dihydroxyvitamin D3> 1,24,25-dihydroxyvitamin D3

Review table ID On exposure to sunlight, 7-dehydrocholesterol (provitamin D3) in the skin absorbs ultraviolet (UV) B rays. This opens the B ring of 7-dehydrocholesterol, forming previtamin D3, which then undergoes thermal isomerization to form vitamin D3 (cholecalciferol). Vitamin D3 is then hydroxylated in the liver to 25-hydroxyvitamin D and subsequently to 1,25-hydroxyvitamin D (the active form) in the kidneys. Vitamin D deficiency can lead to osteomalacia with bone pain or tenderness, muscle weakness or cramps, gait abnormalities, and increased fracture risk. Factors associated with limited UV exposure and increased risk of vitamin D deficiency include the following: Reduced time outdoors: elderly individuals, patients living in residential care or who are frequently hospitalized, people who avoid going outdoors due to high risk of skin cancer Low UV sun exposure: individuals living at extreme northern or southern latitudes Predominantly nocturnal lifestyle: shift work or occupations requiring overnight work Blockade of sunlight exposure: people who use high-grade sunblock or wear full-coverage clothing Reduced UV penetration: individuals with heavily pigmented skin (Choices B and C) The conversion of vitamin D2 and D3 to 25-hydroxycholecalciferol by 25-hydroxylase in the liver is primarily regulated by feedback inhibition from 25-hydroxycholecalciferol. The conversion to 1,25-dihydroxycholecalciferol by 1-alpha-hydroxylase in the kidney is primarily regulated by parathyroid hormone (PTH) and plasma calcium levels. Neither of these steps takes place in the skin or is affected by sunlight exposure. (Choices D and E) 25-hydroxyvitamin D-24-hydroxylase converts 25- and 1,25-dihydroxyvitamin D into inactive 24-hydroxylated metabolites. It is upregulated by 1,25-dihydroxyvitamin D (to prevent excess vitamin D activity) and suppressed by parathyroid hormone (to facilitate replenishment of circulating calcium levels) and thus functions as a counter-regulatory homeostatic enzyme. However, it is not affected by sunlight. Educational objective: Sunlight exposure catalyzes conversion of 7-dehydrocholesterol to cholecalciferol (vitamin D3) in the skin. Subsequent 25-hydroxylation in the liver and 1-hydroxylation in the kidneys produce 1,25-dihydroxyvitamin D, the active form. Inadequate exposure to sunlight can lead to vitamin D deficiency.

A 55-year-old man comes to the emergency department with severe right foot pain that started suddenly in the middle of the night. He has never had such symptoms before. The patient has a history of diet-controlled type 2 diabetes mellitus. BMI is 32 kg/m2. Physical examination shows swelling and tenderness of the first metatarsophalangeal joint. Leukocyte count is 13,500/mm3 and serum creatinine is 0.8 mg/dL. Joint aspiration is performed, and synovial fluid microscopic findings are shown in the image below: ID=860 Blue crystals perpendicular, Yellow crystals parallel Which of the following is the best initial treatment for this patient? Cyclooxygenase inhibitor Lipoxygenase inhibitor TNF-alpha inhibitor Uriosuric agent Xanthine oxidase inhibitor

Synovial fluid analysis in this patient with acute monoarthritis shows needle-shaped, negatively birefringent crystals (ie, monosodium urate crystals) under polarized light, which are diagnostic for gout. Phagocytosis of urate crystals by neutrophils causes the release of various cytokines and inflammatory mediators that leads to additional neutrophil recruitment, resulting in a positive feedback loop that amplifies the inflammatory response. Nonsteroidal anti-inflammatory drugs (NSAIDs) are the first-line treatment for most patients with acute gouty arthritis. They inhibit cyclooxygenase enzymes and therefore decrease prostaglandin synthesis and exert a broad anti-inflammatory effect that includes inhibition of neutrophils. NSAIDs should be avoided in patients with renal and hepatic dysfunction or those at high risk for peptic ulcers. Alternate agents for acute gout depend on patient tolerance and comorbidities and include colchicine and oral or intraarticular glucocorticoids. (Choice B) Lipoxygenase inhibitors (eg, zileuton) decrease production of proinflammatory leukotrienes and are indicated for treatment of asthma. (Choice C) TNF-alpha inhibitors (eg, infliximab, etanercept) are used as disease-modifying agents in rheumatoid arthritis. (Choices D and E) Uric acid-lowering therapy with uricosuric agents (eg, probenecid) or xanthine oxidase inhibitors (eg, allopurinol, febuxostat) is used to prevent acute attacks in patients with recurrent and progressive gouty arthritis and for those with macroscopic tophi. However, these drugs should not be initiated during an acute gout attack because they can exacerbate acute arthritis. Educational objective: Nonsteroidal anti-inflammatory drugs are the first-line treatment for acute gouty arthritis. They inhibit cyclooxygenase and therefore decrease prostaglandin synthesis and exert a broad anti-inflammatory effect that includes inhibition of neutrophils.

A group of investigators is studying the process of bone remodeling in response to steroid hormones. Twenty adult female rats are randomly divided into 2 groups, with one group undergoing bilateral oophorectomy and the other undergoing sham laparotomy to serve as a control group. Eight weeks after the surgery, bone samples are obtained from all animals. Immunohistochemical evaluation shows overexpression of receptor activator of nuclear factor kappa B (RANK) on the surface of certain bone cells in the oophorectomized animals. Which of the following is the most likely effect of the observed finding? Decreased bone mineralization Decreased osteocyte apoptosis Decreased osteoid formation Increased bone resorption Increased osteoprotegerin levels

The 2 most important factors in osteoclast differentiation are macrophage colony-stimulating factor (M-CSF) and receptor activator of nuclear factor kappa B ligand (RANK-L), which stimulate the development of mature, multinucleated osteoclasts. The interaction of RANK-L with RANK (receptor) is blocked by osteoprotegerin (OPG), which acts as a decoy receptor. By binding RANK-L, OPG reduces the differentiation and survival of osteoclasts, resulting in decreased bone resorption and increased bone density. Bone turnover is therefore regulated by the ratio of OPG to RANK-L; bone turnover increases when OPG is low and RANK-L is high. Estrogen maintains bone mass in premenopausal women by inducing the production of OPG by osteoblasts and stromal cells. It also decreases the expression of RANK on osteoclast precursors. By contrast, the loss of estrogen effect (eg, menopause, oophorectomy) increases the expression of RANK-L and decreases production of OPG (Choice E). The decreased OPG to RANK-L ratio leads to increased osteoclast activity and bone resorption. Denosumab is a monoclonal antibody used in the treatment of postmenopausal osteoporosis. It works in a manner similar to OPG by binding RANK-L and blocking its interaction with RANK. (Choices A and C) RANK is not present on osteoblasts, so the decreased OPG to RANK-L ratio seen following oophorectomy does not affect bone mineralization or osteoid formation. (Choice B) Osteocytes are derived from osteoblasts. Estrogen reduces osteoblast and osteocyte apoptosis through activation of extracellular signal-regulated kinases rather than effects on RANK/RANK-L. Educational objective: The receptor activator of nuclear factor kappa B (RANK)/RANK ligand (RANK-L) interaction is essential for the formation and differentiation of osteoclasts. Osteoprotegerin blocks binding of RANK-L to RANK and reduces formation of mature osteoclasts. Low estrogen states cause osteoporosis by decreasing osteoprotegerin production, increasing RANK-L production, and increasing RANK expression in osteoclast precursors

A 34-year-old man comes to the hospital due to several hours of difficulty swallowing, dry mouth, and blurred vision. The patient has a history of major depression. Examination shows mydriasis and poorly reactive pupils. Electrodiagnostic studies reveal normal nerve conduction velocity but decreased compound muscle action potential (CMAP). Rapid, repetitive nerve stimulation leads to facilitation of CMAP. Further review of this patient's medical history would most likely reveal which of the following? Consumption of home-canned food Diarrheal disease after eating undercooked poultry Episodic diplopia and ptosis Occupational exposure to insecticides Recent consumption of puffer fish Tricyclic antidepressant overdose

The combination of nicotinic (eg, diplopia, dysphagia) and muscarinic blockade (eg, dry mouth) is strongly suggestive of food poisoning with Clostridium botulinum toxin, a highly potent preformed neurotoxin. The toxin inhibits acetylcholine release from presynaptic nerve terminals at the neuromuscular junction (NMJ), thereby preventing muscular contraction. This effect can be seen on electromyography as a decrease in the compound muscle action potential (CMAP, or electrical response of the muscle) following stimulation of a motor nerve. High-rate, repetitive nerve stimulation improves the deficit as the rapid depolarization rate increases calcium concentration in the presynaptic nerve terminal, helping mobilize additional acetylcholine vesicles. The anaerobic environment within canned foods contaminated by C botulinum spores allows for germination and growth of the organism. The toxin (produced by vegetative bacteria) is not actively secreted, remaining intracellular until autolysis causes its release into food. The toxin is readily destroyed by heat; however, if food containing the toxin is not cooked properly, the classic diplopia, dysphagia, and dysphonia ("3 Ds") can develop within 12-36 hours of consumption. (Choice B) Guillain-Barré syndrome can develop following Campylobacter jejuni infection and is characterized by absent reflexes and progressive symmetric weakness, typically starting from the lower extremities (unlike botulism, which commonly causes descending paralysis starting from the cranial nerves). (Choice C) Myasthenia gravis (autoimmune attack against postsynaptic nicotinic receptors) is characterized by episodic diplopia and ptosis. However, muscarinic symptoms (eg, dry mouth, mydriasis) are not characteristic, and CMAP does not improve with rapid, repetitive nerve stimulation. In contrast, Lambert-Eaton syndrome (autoantibodies against presynaptic voltage-gated calcium channels) can present with muscarinic symptoms and improved CMAP with rapid nerve stimulation, but the initial weakness typically involves the proximal extremities (particularly legs). (Choice D) Poisoning due to organophosphates (acetylcholinesterase inhibitors) used in insecticides reflects depolarizing blockade (muscle weakness, fasciculations, paralysis), central nervous system effects (lethargy, seizures), and muscarinic overstimulation (miosis, bradycardia, increased lacrimation and salivation). (Choice E) Tetrodotoxin poisoning (due to contaminated puffer fish ingestion) leads to inhibition of sodium influx into nerve cells. Manifestations include weakness, paresthesias (face, extremities), loss of reflexes, and sometimes severe hypotension. Patients commonly remain conscious, although paralyzed. (Choice F) Anticholinergic toxicity is one of the manifestations of tricyclic antidepressant overdose. However, this produces muscarinic (not nicotinic) blockade with no effect on the NMJ (ie, no alterations in CMAP). Educational objective: Botulinum neurotoxin produced by Clostridium botulinum prevents the release of acetylcholine from presynaptic nerve terminals. The toxin impairs peripheral muscarinic and nicotinic neurotransmission, causing both autonomic symptoms (eg, fixed pupillary dilation, dry mouth) and skeletal muscle weakness (eg, diplopia, dysphagia, respiratory depression). Its effects at the neuromuscular junction can be seen on electromyography as a decrease in muscle response following stimulation of a motor nerve.

A 64-year-old man is brought to the emergency department due to severe chest pain, diaphoresis, and shortness of breath. Symptoms began suddenly on awakening and have worsened over the past 2 hours. Blood pressure is 150/90 mm Hg and pulse is 102/min. Physical examination reveals an S4. ECG shows ST-segment depression and T-wave inversion in the lateral leads. Coronary angiography reveals a ruptured atherosclerotic plaque, with a thrombus in the left circumflex artery that causes near-total occlusion. Normal endothelial cells surrounding the lesion have released large amounts of a chemical substance that help decrease thrombus propagation by inhibiting platelet aggregation. Which of the following substances was most likely secreted by these endothelial cells? Hageman factor Kallikrein Prostacyclin Protein C Serotonin Thromboxane A2

The formation of a platelet plug (primary hemostasis) is essential for preventing bleeding after damage to vascular endothelium; it occurs in 3 steps: Platelet adhesion takes place via von Willebrand factor acting as a connector that binds platelets to underlying collagen Platelets become activated and secrete multiple substances, including adenosine diphosphate, ionized calcium, and fibrinogen, from their alpha and delta (dense) granules. Thromboxane A2 (Choice F) is released and acts as a vasoconstrictor and potent stimulator of platelet aggregation. Adenosine diphosphate also stimulates platelet aggregation. Balance is required as excessive platelet plug formation can lead to a pathologic thrombus that restricts blood flow (eg, myocardial infarction). To oppose the functions of thromboxane A2, the endothelium secretes prostacyclin (prostaglandin I2), which is derived from arachidonic acid and synthesized from prostaglandin H2 by prostacyclin synthase. Once secreted, prostacyclin acts locally to inhibit platelet aggregation and adhesion to the vascular endothelium and to cause vasodilation. Nitric oxide aids in these functions as well. Atherosclerosis can impair the ability of endothelial cells to synthesize prostacyclin and nitric oxide, creating localized predisposition to excessive platelet thrombus formation. A synthetic prostacyclin, epoprostenol, is used in the treatment of pulmonary hypertension, peripheral vascular disease, and Raynaud syndrome. (Choice A) Hageman factor (factor XII) is synthesized by the liver and is activated by exposed collagen following damage to endothelial cells. It participates in secondary hemostasis by activating the intrinsic clotting pathway; it also activates fibrinolysis. (Choice B) Kallikrein converts kininogen into bradykinin. It also may play a role in triggering the fibrinolytic pathway, but it does not inhibit platelet aggregation. (Choice D) Protein C is a vitamin K-dependent factor synthesized by the liver. It regulates the coagulation cascade by inactivating factors Va and VIIIa. (Choice E) Serotonin has a wide range of functions, including mediation of mood (in the central nervous system) and gastrointestinal motility. It is also released from the delta granules of activated platelets and facilitates localized vasoconstriction. Educational objective: Prostacyclin (prostaglandin I2) is synthesized from prostaglandin H2 by prostacyclin synthase in vascular endothelial cells. Once secreted, it inhibits platelet aggregation and causes vasodilation to oppose the functions of thromboxane A2 and help maintain vascular homeostasis.

A 35-year-old man with a known history of peptic ulcer disease comes to the physician with sudden onset of pain, swelling, and redness at the base of his great toe. The patient was awakened in the middle of the night by the pain and was unable to go back to sleep. He has no history of trauma to the joint. Physical examination shows swelling, erythema, and exquisite tenderness involving the right first metatarsophalangeal joint. Fine-needle aspiration of the joint shows needle-shaped, negatively birefringent crystals. After making the diagnosis, the physician prescribes an appropriate medication. Shortly after starting the medication, the patient develops nausea, vomiting, and diarrhea. Which of the following is the most likely mechanism of action of the drug prescribed to this patient? Decreased renal tubular uric acid reabsorption Inhibition of cyclooxygenase activity Inhibition of microtubule formation Inhibition of phospholipase A2 activity Inhibition of xanthine oxidase activity

Acute onset of pain and swelling of the first metatarsophalangeal joint and joint aspiration showing needle-shaped, negatively birefringent crystals are diagnostic for acute gouty arthritis. Nonsteroidal anti-inflammatory drugs (NSAIDs) are the mainstay of treatment for most patients. However, colchicine is often used in patients with mild-to-moderate renal failure, peptic ulcer disease, or other contraindications to NSAIDs. Colchicine is an effective anti-inflammatory agent in acute gouty arthritis and acts by binding to the intracellular protein tubulin, preventing tubulin polymerization into microtubules. This leads to impaired leukocyte migration and phagocytosis, reducing the inflammation seen in gouty arthritis. Because colchicine also disrupts microtubule formation in gastrointestinal mucosal cells, many patients develop diarrhea and, less commonly, nausea, vomiting, and abdominal pain. (Choices A and E) Probenecid decreases proximal tubular uric acid reabsorption, and allopurinol inhibits xanthine oxidase (converts xanthine to uric acid). These medications are prescribed to lower serum uric acid levels in chronic gout. These agents are contraindicated in acute gouty arthritis as they can mobilize tissue stores of uric acid and precipitate or worsen acute attacks. (Choice B) NSAIDs induce their anti-inflammatory effects by inhibiting cyclooxygenase (COX)-1 and COX-2 isoenzymes, thus blocking prostaglandin synthesis. Because they can cause significant gastrointestinal irritation (eg, nausea, dyspepsia, bleeding), their use is contraindicated in patients with peptic ulcer disease. (Choice D) Glucocorticosteroids exert their anti-inflammatory effect by inhibiting phospholipase A2 activity. These medications can be used in acute gout and are not associated with significant diarrhea. Educational objective: Colchicine is a second-line agent for treating acute gouty arthritis. It inhibits tubulin polymerization and microtubule formation in leukocytes, reducing neutrophil chemotaxis and emigration to sites inflamed by tissue deposition of monosodium urate crystals. Gastrointestinal mucosal function is also impaired by microtubule disruption, leading to diarrhea and, less commonly, nausea, vomiting, and abdominal pain.

A 68-year-old woman with end-stage renal disease receiving intermittent hemodialysis comes to the office due to back pain. She says that she may have "pulled a muscle" while getting out of her car a week ago; since then she has had progressively worsening back pain. She has no leg numbness or weakness but reports malaise and fatigue. The patient also has hypertension and type 2 diabetes mellitus and was treated several weeks ago for staphylococcal bacteremia associated with the dialysis catheter. Her temperature is 38 C (100.4 F). On examination, she has tenderness over the upper lumbar vertebrae without overlying skin changes. The straight leg raise test is negative. Which of the following is the best next step in management of this patient? Analgesics and close follow-up CT myelogram Lumbar puncture MRI of the spine Serum protein electrophoresis

Although this patient believes her back pain may be due to a muscle sprain, her presentation (localized bony back pain, low-grade fever, recent staphylococcal bacteremia) suggests vertebral osteomyelitis. Bacteria can access the spine by hematogenous spread from a distant infection (eg, skin or soft tissue, intravenous catheter), direct invasion from trauma or local spinal procedures (eg, lumbar puncture, spinal surgery), or direct spread from adjacent soft tissue infection. Hematogenous spread is most common as the adult vertebral bone has a very rich and vascular marrow. Increasing age can cause the nutrient arteries to develop a "corkscrew" anatomy, which allows bacteria to more easily penetrate the marrow cavity and cause local infection. Bacteremia due to intravascular devices (eg, hemodialysis catheters) increases risk of health care-related vertebral osteomyelitis. Offending organisms include Staphylococcus (most common is S aureus or coagulase-negative staphylococci) and various gram-negative organisms (eg, Pseudomonas). Vertebral osteomyelitis should be suspected in patients with new or worsening back pain, fever, and recent endocarditis or bacteremia (especially S aureus). It should also be suspected if there are new neurologic findings and fever, with or without back pain. Initial evaluation includes blood cultures and MRI of the spine, which is the most sensitive imaging method for diagnosing vertebral osteomyelitis. (Choice A) Analgesics and close follow-up are recommended for musculoskeletal causes of low back pain, but this patient's clinical findings (fever, localized bony tenderness, recent bacteremia) are concerning for more serious etiologies and require further evaluation. (Choice B) CT myelogram, which can diagnose spinal stenosis or other vertebral pathology (including osteomyelitis), is performed by injecting contrast into the lumbar spine prior to CT imaging. However, it is more invasive than MRI and usually reserved for patients who cannot undergo MRI (eg, those with pacemaker, metallic implants). (Choice C) Lumbar puncture is typically used for diagnosing meningitis and has a low yield for diagnosing vertebral osteomyelitis. CT-guided bone biopsy is usually preferred for isolating the organism after imaging confirms the diagnosis of vertebral osteomyelitis. (Choice E) Serum protein electrophoresis is typically done to evaluate for monoclonal gammopathies (eg, multiple myeloma). Although multiple myeloma can present with back pain, this patient's fever and recent bacteremia make vertebral osteomyelitis more likely. Educational objective: Vertebral osteomyelitis should be suspected in patients with new or worsening back pain, fever, and recent endocarditis or bacteremia (especially Staphylococcus aureus). It should also be suspected if there are new neurologic findings and fever with or without back pain. MRI of the spine is preferred for diagnosis.

A 28-year-old man comes to the office with a 2-month history of dull low back pain and morning stiffness. The pain had an insidious onset, and the patient does not recall any back trauma. He attempted treatment with acetaminophen and spinal manipulation, which did not provide significant relief. The patient had been a back sleeper, but now must lie on his side to fall asleep easily. Past medical history is unremarkable. Vital signs are normal. Physical examination shows limited anterior flexion of the spine. There is no swelling or warmth of any peripheral joints. X-rays reveal narrowing of the sacroiliac joints. Which of the following is most strongly associated with this patient's condition? A specific human leukocyte antigen class I allele A specific human leukocyte antigen class II allele Auto-reactive immunoglobulin M antibodies Deficient complement component Selective immunoglobulin deficiency

Ankylosing spondylitis is a chronic inflammatory disorder of the sacroiliac joints and axial skeleton. It is most common in young and middle-aged men and presents with morning stiffness and low back pain. Ankylosing spondylitis is characterized by destruction of articular cartilage with resulting stiffness and fusion of axial joints. The sacroiliac joints are often tender to palpation, and the spine may have decreased range of motion. X-ray of the sacroiliac joints may reveal erosions, sclerosis, and narrowing; ultimately fusion of the joint spaces. Spine x-rays reveal sclerosis, ligamentous calcification, and vertebral fusion ("bamboo spine"). Ankylosing spondylitis, reactive arthritis, arthritis associated with inflammatory bowel disease, and psoriatic arthritis are seronegative spondyloarthropathies, so-called due to the absence of serum rheumatoid factor. Patients with these diseases have a higher incidence of the human leukocyte antigen (HLA) B27 allele, which encodes a specific HLA class I molecule, compared to the general population (although most patients with HLA B27 will not develop spondyloarthropathies). (Choice B) Class I HLA proteins (eg, HLA B27) are expressed by all nucleated cells and present endogenous antigens to CD8+ cytotoxic T cells. By contrast, HLA class II proteins (eg, DR, DP, DQ alleles) are expressed by antigen-presenting cells (eg, macrophages, dendritic cells) and present predominantly foreign antigens to CD4+ helper T cells. Conditions associated with specific HLA class II alleles include rheumatoid arthritis, type I diabetes mellitus, and celiac disease. (Choice C) Rheumatoid factor (IgM antibodies against self IgG) is present in the majority of patients with rheumatoid arthritis and may be seen in a variety of other autoimmune diseases as well as in some healthy individuals. (Choice D) Complement component deficiencies are associated with recurrent infections and systemic lupus erythematosus. (Choice E) IgA deficiency causes recurrent mucosal and respiratory infections as well as anaphylactic reactions after transfusion of blood products. Educational objective: The seronegative spondyloarthropathies include ankylosing spondylitis, reactive arthritis, psoriatic arthritis, and arthritis associated with inflammatory bowel disease. Individuals expressing HLA B27 (a specific HLA class I molecule) are at increased risk for the seronegative spondyloarthropathies.

Researchers are studying treatment of osteoporotic fractures. Subjects with pelvic fractures due to osteoporosis are divided into 2 groups. The control group receives physical therapy along with calcium and vitamin D supplementation. The experimental group receives a parathyroid hormone analogue in addition to the standard therapy. It is found that the subjects who receive hormonal treatment have a shorter time to fracture healing and improved functional outcome. The benefits of the investigational medication are attributed to increased osteoblastic activity. An elevated level of which of the following markers is most likely to reflect the medication effect in these subjects? Serum alkaline phosphatase Serum aspartate aminotransferase Serum calcitonin Serum tartrate-resistant acid phosphatase Urinary hydroxyproline Urinary type I collagen telopeptide

Bone is continually broken down and reformed by the process known as bone remodeling, which consists of the coordinated activity of osteoblasts (responsible for bone formation) and osteoclasts (responsible for bone resorption). The subjects in this study are receiving a recombinant parathyroid hormone analog (eg, teriparatide), which promotes bone formation by stimulating maturation of pre-osteoblasts into osteoblasts. Osteoblasts synthesize bone matrix and express alkaline phosphatase (AlkP), which promotes normal bone mineralization by increasing local concentrations of inorganic phosphorus. Serum AlkP levels correlate with osteoblastic activity. However, total serum levels are nonspecific, as AlkP is also produced by the hepatobiliary tree, intestine, and placenta and may be elevated due to other causes (eg, pregnancy, biliary obstruction). If the source of AlkP is uncertain, AlkP isoenzymes (ie, liver, placental, intestinal, bone) can be differentiated with additional laboratory techniques (eg, electrophoresis, immunoassay). Other markers of osteoblast activity include N-terminal propeptide of type 1 procollagen (PINP), which is released during post-translation cleavage of type 1 procollagen before its assembly into mature type 1 collagen fibrils. (Choice B) Aspartate aminotransferase (AST) is predominantly found in liver, muscle, and kidneys. AST is commonly used as a marker for hepatic injury and is also elevated in rhabdomyolysis and myocardial infarction. (Choice C) Calcitonin is secreted by the parafollicular C cells of the thyroid and inhibits bone resorption by osteoclasts. As a laboratory marker, calcitonin is primarily used in the diagnosis of medullary thyroid cancer; it does not mark osteoblastic activity. (Choice D) Acid phosphatase is a family of lysosomal enzymes present in a wide variety of cells, including osteoclasts (the acid phosphatase in osteoclasts is resistant to degradation by tartrate). However, osteoblasts do not express acid phosphatase. (Choices E and F) Osteoclastic activity (ie, bone resorption) leads to breakdown of fibrillar collagen with release of biomarkers such as hydroxyproline and collagen telopeptides. Increased urine levels are associated with states of increased bone resorption (rather than formation). Educational objective: Bone-specific alkaline phosphatase levels correlate with osteoblast activity. Other markers of osteoblast activity include N-terminal propeptide of type 1 procollagen, which is released during post-translation cleavage of type 1 procollagen.

A 45-year-old woman comes to the office due to a long history of joint pain, swelling, and stiffness. She recently immigrated to the United States and had little access to medical care in her original country. On physical examination, her proximal interphalangeal and metacarpophalangeal joints are tender and swollen bilaterally, and her fingers appear significantly deformed. Subcutaneous nodules are palpated near the elbow. Laboratory studies are obtained to confirm the diagnosis. This patient's serum is most likely to contain IgM antibodies against which of the following substances? Centromeres Double-stranded DNA Fc portion of human IgG Mitochondrial extract Nuclear basic proteins Phospholipids Sheep eryhthrocytes

Bilateral pain, stiffness, and deformity of the proximal interphalangeal and metacarpophalangeal joints are typical of advanced rheumatoid arthritis (RA). Morning stiffness, tenderness, and edema of the affected joints predominate early in this disease. Later, osteopenia, erosions, and joint space narrowing lead to decreased range of motion and deformities (eg, swan neck, ulnar deviation). Patients can also develop rheumatoid nodules, firm, nontender, subcutaneous nodules that occur at pressure points such as the elbows or forearms. RA is due to failure of immune tolerance, with development of an autoreactive immune response directed against joint components (eg, type II collagen, citrullinated vimentin). CD4+ T-helper cells become activated by these self-antigens and release cytokines that promote chronic inflammatory synovitis. CD4+ T cells also induce B cells to synthesize rheumatoid factor and anti-citrullinated protein antibodies (ACPAs). Rheumatoid factor is an antibody (typically IgM) specific for the Fc component of IgG. Rheumatoid factor binds circulating IgG and ACPAs bind modified self-proteins, forming immune complexes that deposit on the synovium and cartilage. These complexes activate complement in those locations, contributing to chronic inflammation and joint destruction. (Choice A) Anticentromere antibodies are found in the majority of patients with CREST syndrome. (Choice B) Antibodies to double-stranded DNA (anti-dsDNA) are specific for systemic lupus erythematosus. (Choice D) Antimitochondrial antibodies are found in patients with primary biliary cholangitis. (Choice E) The presence of antinuclear antibodies is a nonspecific finding in many connective tissue disorders. These can occur in IgM form, but would be found less frequently than rheumatoid factor in patients with RA. (Choice F) Antiphospholipid antibodies can be found in patients with systemic lupus erythematosus and antiphospholipid antibody syndrome. Antiphospholipid antibodies cause a hypercoagulable state with paradoxical PTT prolongation. (Choice G) The Monospot test assesses the serum's ability to agglutinate sheep erythrocytes. It is positive in the presence of heterophile antibodies and is used to diagnose infectious mononucleosis due to Epstein-Barr virus. Educational objective: Rheumatoid arthritis results from an immune response directed against autoantigens in the joints. Infiltrating CD4+ T cells secrete cytokines that promote inflammatory synovitis. They also stimulate B cells to produce rheumatoid factor (IgM antibody specific for Fc component of IgG) and anti-citrullinated protein antibodies that contribute to chronic inflammation and joint destruction.

A prospective observational study is performed to determine factors affecting bone mass and fracture risk. Detailed demographic and clinical information is obtained from a large number of volunteers, and bone mass is monitored over time. The data obtained from 2 specific groups of women are shown below. ID=628 Bone Mass A>Bone Mass B Which of the following factors is most likely present in group A? Early menopause Increased physical activity Lower body mass index Positive smoking history Use of glucocorticoids

Bone mass in adulthood is determined by peak bone mass and the subsequent rate of bone loss. Most of the variation in peak bone mass is genetically determined, but environmental factors such as nutritional status (eg, calcium and vitamin D intake) and physical activity also have a significant impact. Establishment of a high peak bone mass during early adulthood is important because bone mass subsequently declines with age. By convention, osteoporosis is defined as a bone density that is 2.5 or more standard deviations below the mean (T score ≤ -2.5) for peak young adult bone density. The subjects in group A show a higher peak bone mass, slightly lower rate of post-peak bone loss, and higher late-life bone mass than the subjects in group B. This is consistent with the effects of regular exercise, especially weight-bearing exercise. In contrast, factors that lead to a lower peak bone mass include genetic and ethnic factors (eg, Asian or Caucasian ethnicity), chronic inflammatory disease, exposure to glucocorticoid medications, and sedentary lifestyle. In addition, prolonged caloric deficits (often seen with excessive exercise) can lead to functional hypothalamic amenorrhea, suppression of estrogen production, and rapid bone loss. (Choice A) Estrogens have an anabolic effect on bone by increasing osteoblastic activity and decreasing osteoclastic activity. Loss of estrogen effect (eg, menopause, premature ovarian failure) causes accelerated bone loss. (Choice C) Bone density is proportional to BMI. BMI <22 kg/m2 (or weight <57.6 kg [127 lb]) is associated with increased risk for osteoporosis. (Choice D) Smoking causes accelerated bone loss and is a major risk factor for osteoporosis and osteoporotic fractures. (Choice E) Chronic glucocorticoid use is a major risk factor for osteoporosis. Glucocorticoids decrease the gastrointestinal absorption of calcium, inhibit collagen synthesis by osteoblasts, decrease GnRH (leading to hypogonadism), and increase urinary calcium loss. Educational objective: Regular exercise leads to increased peak bone mass, a reduced rate of bone loss, and a decreased risk of osteoporosis.

A 65-year-old man is admitted to the intensive care unit with second- and third-degree burns over 25% of his total body surface area. Intravenous fluids and analgesics are administered, and his wounds are cleaned and debrided. After 7 days, the patient develops a fever and his leukocyte count rises to 16,000 cells/µL. Examination of the burns on his left leg shows surrounding warmth, erythema, and induration. Gram stain of his wound tissue is shown in the image below. ID=8858 Culture on MacConkey agar grows oxidase-positive colonies that do not ferment lactose. Which of the following is the best initial treatment for this patient? Cefepime Ceftriaxone Doxycycline Moxifloxacin Trimethoprim-sulfamethoxazole

Burn patients are at increased risk of bacterial infection due to loss of the barrier function of the skin, post-burn immune dysfunction, and lack of blood flow to necrotic areas. Common pathogens include Gram-positive (eg, Staphylococcus aureus, Enterococcus species) and Gram-negative (eg, Pseudomonas aeruginosa, Escherichia coli, Klebsiella pneumoniae, and Acinetobacter species) bacteria. This patient's burn is infected with Gram-negative rods that are oxidase-positive and non-lactose- fermenting, making P aeruginosa the most likely pathogen. Cephalosporins such as cefepime and ceftazidime have good anti-pseudomonal coverage. (Choice B) Ceftriaxone is a third-generation cephalosporin used primarily for the treatment of community-acquired pneumonia and bacterial meningitis. It has activity against some Gram-positive (eg, Streptococcus pneumoniae, S pyogenes) and Gram-negative (eg, Haemophilus influenzae, K pneumoniae, E coli) pathogens. Unlike ceftazidime, it has poor activity against P aeruginosa. (Choice C) Doxycycline is effective against Lyme disease and many tickborne rickettsial diseases but does not have good anti-pseudomonal coverage. (Choice D) Moxifloxacin is a fluoroquinolone used mainly for the treatment of bacterial infections that cause exacerbation of chronic obstructive pulmonary disease, sinusitis, and community-acquired pneumonia. Unlike ciprofloxacin and levofloxacin, moxifloxacin has poor anti-pseudomonal activity. (Choice E) Trimethoprim-sulfamethoxazole is used to treat susceptible organisms that cause urinary tract, gastrointestinal, and skin infections. It is the drug of choice for organisms such as Pneumocystis jiroveci, Toxoplasma gondii, Nocardia, and Stenotrophomonas maltophilia. It does not have activity against P aeruginosa. Educational objective: Pseudomonas aeruginosa is a major pathogen in burn patients. Only a few specific penicillins (eg, ticarcillin, piperacillin) and cephalosporins (eg, ceftazidime, cefepime) have activity against it. Certain aminoglycosides, fluoroquinolones (eg, ciprofloxacin, levofloxacin), and carbapenems (eg, imipenem, meropenem) are also effective.

A 48-year-old man comes to the office due to several hours of severe right knee pain. The patient has a history of peptic ulcer disease and gastroesophageal reflux disease. His right knee is swollen, erythematous, and tender. Arthrocentesis is performed and synovial fluid analysis shows needle-shaped, negatively birefringent crystals with many neutrophils. The medication given to this patient selectively binds to an interleukin-1 inducible enzyme that is highly expressed by inflammatory cells and undetectable in the surrounding normal tissue. Which of the following is most likely the drug used in this patient's treatment? Aspirin Celecoxib Colchicine Infliximab Prednisone

Cyclooxygenase (COX), also known as prostaglandin endoperoxide synthase, catalyzes the conversion of arachidonic acid into prostanoids. It exists in 2 isoforms designated COX-1 and COX-2. COX-1 is constitutively expressed in various tissues and is involved in a number of "housekeeping functions" (eg, platelet aggregation, gastric mucosal protection, vascular homeostasis). COX-2 is an inducible enzyme that is undetectable in most tissues under normal conditions. During inflammation, infiltrating cells secrete cytokines (eg, interleukin-1, TNF-α) that cause COX-2 upregulation in the inflamed tissue. COX-2 then drives the synthesis of pro-inflammatory arachidonic acid metabolites. This patient's acute gout attack should be treated with an agent that provides prompt pain relief without exacerbating his gastrointestinal problems. Nonsteroidal anti-inflammatory drugs (NSAIDs) such as ibuprofen, indomethacin, and naproxen inhibit both COX-1 and COX-2 and can cause significant gastrointestinal injury. Selective COX-2 inhibitors (eg, celecoxib) decrease inflammation but have no effect on COX-1, minimizing gastroduodenal toxicity. (Choice A) Like other NSAIDs, aspirin nonselectively inhibits both COX-1 and COX-2. Aspirin irreversibly modifies these enzymes; therefore, restoration of enzymatic activity requires synthesis of new enzymes. (Choice C) Colchicine binds to tubulin, which inhibits microtubule formation. This results in impaired neutrophil mitosis and decreased neutrophil chemotaxis. (Choice D) Infliximab is a monoclonal antibody that irreversibly binds to and inhibits TNF-α, a cytokine involved in the inflammatory response. TNF-α is an intercellular signaling protein, not an enzyme. (Choice E) Glucocorticoids (eg, prednisone) bind to cytoplasmic receptors that then translocate to the nucleus where the expression of anti-inflammatory peptides is upregulated. Glucocorticoids decrease COX-2 transcription but do not bind to COX-2 directly. Educational objective: Cyclooxygenase-2 (COX-2) is an inducible enzyme upregulated during inflammation by interleukin-1 and TNF-α. Selective COX-2 inhibitors (eg, celecoxib) decrease inflammation by inhibiting COX-2 production of pro-inflammatory arachidonic acid metabolites. Because they do not affect COX-1, they have minimal gastroduodenal toxicity.

A 58-year-old woman comes to the office with left knee pain. The pain began a year ago and has progressively become more severe. The pain is worse later in the day and is now limiting her normal day-to-day activities. She has no history of injury to the joint and has no significant pain in any other joints. The patient has attempted treatment with acetaminophen, but that has not provided adequate relief. She does not use tobacco, alcohol, or illicit drugs and works as a school bus driver. Her BMI is 34 kg/m2. Left knee joint examination shows a mild effusion, crepitus on movement, and joint line tenderness. The patient is started on celecoxib therapy. The presence of which of the following conditions in this patient's medical history would make this medication the preferred treatment? Hyperlipidemia Hypertension Peptic ulcer disease Renal impairment Stable angina

Cyclooxygenase (COX; prostaglandin synthase) initiates the synthesis of prostanoids (eg, prostaglandins, thromboxane) from arachidonic acid. COX exists in 2 primary forms: COX 1, which plays a physiologic role in a number of normally functioning tissues (eg, platelets, gastrointestinal tract) COX 2, which is preferentially expressed at sites of inflammation. Nonsteroidal anti-inflammatory drugs (NSAIDs) work by inhibiting COX to block prostaglandin synthesis. Most NSAIDs inhibit both the COX 1 and COX 2 isoenzymes. However, inhibition of COX 1 may lead to severe adverse effects, including gastric ulceration (reduced mucosal protection) and increased bleeding (decreased platelet aggregation). Selective COX 2 inhibitors (eg, celecoxib) have potent anti-inflammatory effects with less risk of bleeding and gastrointestinal ulceration. They are recommended for use in patients at risk for gastrointestinal complications, such as those with prior peptic ulcer disease. The risk for ulcers can also be reduced by combining a nonselective NSAID with a second drug (eg, proton pump inhibitors, misoprostol) to reduce gastric acid secretion. (Choice A) Selective COX 2 inhibitors do not have a substantial effect on serum lipids. (Choices B and D) Both COX 1 and COX 2 are expressed in renal tissues. Therapy with either nonselective NSAIDs or selective COX 2 inhibitors can cause fluid retention, aggravation of hypertension, and impaired renal function. (Choice E) COX 2 is expressed in vascular endothelial cells and vascular smooth muscle cells, where it plays a role in the local production of prostacyclin, which has anticoagulant and vasodilatory actions. COX 2 inhibitors have been associated with an increased risk of cardiovascular events. Educational objective: Selective cyclooxygenase 2 (COX 2) inhibitors relieve pain with lower risk of bleeding and gastric ulceration than nonselective nonsteroidal anti-inflammatory drugs.

A 10-year-old boy is brought to the emergency department after experiencing high fevers and chills for the last few days. He also complains of dull pain just above his left knee. He has no history of recent trauma other than minor scrapes to his knees and elbows while playing outside. Physical examination shows point tenderness 3 cm above the kneecap. There is no joint effusion. Radiographs show soft-tissue swelling and a periosteal reaction over the lower end of the femur. Which of the following organisms is most likely responsible for this patient's symptoms? Enterococcus faecalis Moraxella catarrhalis Staphylococcus aureus Staphylococcus epidermidis Streptococcus agalactiae Streptococcus pneumoniae Streptococcus pyogenes

Osteomyelitis is an infection of bone and bone marrow that occurs by 1 of 3 mechanisms: Hematogenous seeding due to an episode of bacteremia Spread from a contiguous focus of infection, as occurs in an infected diabetic foot wound Direct inoculation of bone, such as with a compound fracture Hematogenous osteomyelitis occurs predominantly in children (particularly boys) and most frequently affects the long bones. The tibia, fibula, and femur are most often involved. Adults who develop the condition are more likely to have vertebral involvement and frequently have a predisposition to bacteremia due to risk factors such as IV drug abuse or indwelling vascular catheters. The presenting symptoms of hematogenous osteomyelitis are vague, and a high index of suspicion is required to make the diagnosis. Initial symptoms such as malaise and fevers are non-specific. As the infection progresses, infants and younger children may refuse to move the affected extremity. Older children often complain of pain over a long bone. Bone pain develops as the abscess expands within the bone, leading to bone necrosis, periosteal disruption, and swelling of the surrounding soft tissue. Staphylococcus aureus is implicated in most cases of acute hematogenous osteomyelitis in otherwise healthy children. (Choice A) Enterococcus faecalis causes a variety of infections, including endocarditis, meningitis, and urinary tract infections. Enterococcus can cause vertebral osteomyelitis after a recent urinary tract infection via bacteremic spread. (Choice B) Moraxella catarrhalis is a part of the normal flora of the upper respiratory tract. It causes otitis media and sinusitis in healthy individuals and is frequently responsible for causing exacerbation of chronic obstructive pulmonary disease. (Choice D) Staphylococcus epidermidis is ubiquitous in nature and is commonly isolated in cultures as a contaminant. However, S epidermidis can also be pathogenic, colonizing intravenous catheters and other foreign bodies such as prosthetic heart valves and orthopedic hardware, leading to bacteremia and sepsis. (Choice E) Streptococcus agalactiae (group B streptococcus) frequently colonizes the gastrointestinal and urogenital tracts. Infants born vaginally to colonized mothers can develop serious neonatal infections, including sepsis, pneumonia, and meningitis. For this reason, pregnant women testing positive for group B streptococci are treated with antibiotic prophylaxis during labor and delivery. (Choice F) Streptococcus pneumoniae is the most common etiologic agent of community-acquired pneumonia. It also causes otitis media in children, sinusitis, meningitis, and sepsis. (Choice G) After Staphylococcus aureus, Streptococcus pyogenes (group A streptococcus) is the second most common cause of hematogenous osteomyelitis in children. Group A streptococci are also responsible for streptococcal pharyngitis and skin infections such as impetigo and necrotizing fasciitis. Educational objective: Hematogenous osteomyelitis is predominantly a disease of children that most frequently affects the long bones. Staphylococcus aureus is implicated in most cases secondary to a bacteremic event. Streptococcus pyogenes (group A streptococcus) is the second most common cause of hematogenous osteomyelitis.

The public health department of a developing country performs an epidemiologic study to assess the nationwide incidence of upper respiratory infections among children. The data show a high rate of childhood bacterial pharyngitis. Current practice guidelines indicate that a rapid test should be performed in children to identify the presence of bacterial antigens. If this test is negative, a culture, the gold standard for definitive diagnosis, is then performed. To offset cost and avoid losing patients to follow-up, the department is considering recommending the empiric use of penicillin for the treatment of suspected bacterial pharyngitis. Which of the following would be expected to decrease after long-term implementation of this guideline? Deaths associated with diarrheal illness Diagnosis of serum sickness-like reactions Incidence of nephritic syndrome Need for cardiac surgery Use of broad-spectrum antibiotics

Early penicillin treatment of group A streptococcal pharyngitis is important for the prevention of acute rheumatic fever (ARF), a major cause of cardiovascular death in many developing nations. ARF primarily affects the heart and central nervous system because host antibodies produced in response to streptococcal antigens cross-react with host antigens in these organs. Chronic cardiac inflammation can progress to rheumatic heart disease, specifically valvular disease. The mitral valve is the most commonly affected and gradually thickens, fibroses, and calcifies, eventually requiring valvotomy or more invasive surgical intervention. Empiric therapy for a condition must be considered in the context of the host characteristics, pre-test probability of the disease, benefits/risks of waiting for a definitive diagnosis, and cost of therapy versus its potential complications. If all cases of acute streptococcal pharyngitis were treated empirically, the incidence of rheumatic heart disease and associated cardiac procedures would likely decrease. (Choice A) Antibiotic use can promote diarrhea by altering the gut microbial milieu. This alteration can lead to overgrowth of Clostridium difficile, a gram-positive bacterium that can cause profuse, watery diarrhea and life-threatening fulminant colitis. Therefore, increasing antibiotic use would likely result in increased diarrheal illness and potential complications. (Choice B) Serum sickness-like reactions are associated with certain infections (eg, hepatitis B) and antibiotics (eg, penicillin). The development of circulating drug-specific immune complexes may cause fever, rash, and arthritis. If penicillin was used more frequently, serum sickness-like reactions would potentially increase, not decrease. (Choice C) Post-streptococcal glomerulonephritis (PSGN) is the most common cause of nephritic syndrome in children worldwide. It is caused by deposition of immune complexes in glomeruli following pharyngitis or skin infections with group A Streptococcus. Although early antibiotics are effective in preventing ARF, they have not been shown to reduce the risk of PSGN. (Choice E) Empiric use of potentially unnecessary antibiotics may cause an increase in antibiotic resistance, thereby increasing the need for broad-spectrum antibiotics. For example, methicillin-resistant Staphylococcus aureus (MRSA) evolved from methicillin-sensitive S aureus via alteration of the protein binding site for beta-lactam antibiotics. As a result, broad-spectrum antibiotics (eg, vancomycin, daptomycin) are required for MRSA infections. Educational objective: Acute rheumatic fever is a complication of untreated group A streptococcal pharyngitis. Rheumatic heart disease is the most common cause of acquired valvular heart disease and cardiovascular death in developing countries. The incidence of acute rheumatic fever and rheumatic heart disease has been reduced in industrialized nations with prompt treatment of streptococcal pharyngitis with penicillin.

A 67-year-old man comes to the emergency department with low-grade fevers and fatigue. He has no history of heart disease. His temperature is 38.4 C (101.1 F), blood pressure is 117/76 mm Hg, pulse is 82/min, and respirations are 14/min. On examination, there is a diastolic murmur at the left sternal border. He has nontender erythematous macules on his hands and feet. Blood cultures show gram-positive cocci that are catalase-negative and able to grow in hypertonic (6.5%) saline and bile. This patient's medical history is most likely to include which of the following procedures within the past month? Cystoscopy Dental extraction Nasal polypectomy Sinus drainage Skin biopsy

Enterococcal endocarditis (representing up to 30% of nosocomial endocarditis cases) usually occurs in elderly men who have recently undergone manipulation of areas colonized by this organism. The genitourinary tract (eg, cystoscopy) is the most common portal of entry; gastrointestinal (eg, colonoscopy) or obstetric procedures are also risk factors. Enterococci can also cause urinary tract infections (most common), intraabdominal and pelvic infections, and wound infections. Enterococci are typically gamma-hemolytic (no hemolysis on agar), catalase-negative, pyrrolidonyl arylamidase-positive, Lancefield Group D gram-positive cocci that can grow in hypertonic (6.5%) saline and bile. The most common species of enterococci (formerly classified under the genus Streptococcus) are Enterococcus faecalis (80%-90%) and Enterococcus faecium (10%-15%). Enterococci are resistant to many antibiotics and can be very difficult to treat. A cell wall-active agent (such as ampicillin) is often combined with an aminoglycoside for synergistic effect, but increasingly bacteria resistant to both aminoglycosides and β-lactams (including penicillinase-resistant types) are emerging. Strains with vancomycin or linezolid resistance have also appeared. (Choice B) Dental extraction is associated with endocarditis caused by viridans group streptococci (VGS). In most cases, VGS cause subacute bacterial endocarditis in already abnormal heart valves (eg, congenital valvular abnormalities, valves damaged by rheumatic fever). VGS do not grow on hypertonic saline. In addition, almost all VGS species do not grow on bile (VGS are also considered bile-insoluble as they exhibit no cell lysis in the presence of bile, in contrast to the bile-soluble S pneumoniae). (Choice C) Removal of a nasal polyp can cause transient bacteremia. Organisms that colonize the nose include Corynebacterium species, Haemophilus species, staphylococci, streptococci, and Neisseria species. Of these, staphylococci and streptococci may cause endocarditis, but the organism described displays characteristics of Enterococcus. (Choice D) Sinus infections are usually caused by S pneumoniae, Haemophilus influenzae, or Moraxella catarrhalis. S pneumoniae would be a very rare cause of endocarditis and is usually identified by being bile-soluble and optochin-sensitive. (Choice E) Skin biopsy is a benign procedure that would be unlikely to cause endocarditis. Skin is colonized with many bacteria, including Staphylococcus and Streptococcus species, Corynebacterium species, and Bacillus species. Educational objective: Enterococcus is a component of the normal colonic and urogenital flora and is capable of growing in hypertonic saline and bile. It is gamma-hemolytic, catalase-negative, and pyrrolidonyl arylamidase-positive. Genitourinary instrumentation or catheterization has been associated with enterococcal endocarditis.

As part of a long-term cohort study, members of a large extended family undergo periodic analysis of multiple serum markers. Many male participants are found to have abnormal laboratory results despite no obvious signs of disease. Further analysis shows that these men have an X-linked mutation affecting the phosphoribosyl pyrophosphate (PRPP) synthetase gene, resulting in greatly increased substrate conversion. Item 1 of 2 Which of the following organs is most likely to develop pathology secondary to this mutation? Aorta Heart Joints Liver Pancreas

Gout is a disease caused by tissue deposition of monosodium urate crystals. Elevated uric acid levels are a known risk factor for gout and increased purine metabolism is one possible cause of hyperuricemia. Phosphoribosyl pyrophosphate (PRPP) synthetase is the enzyme responsible for the production of the activated ribose necessary for de novo synthesis of purine and pyrimidine nucleotides. The mutation described in the question stem will cause increased production of purines due to feed-forward activation of the purine synthesis pathway. As a result, more purine molecules will undergo degradation, resulting in hyperuricemia and an increased risk of gout. (Choice A) The aorta can develop aneurysms in patients with Marfan syndrome, which results from defects in fibrillin-1. (Choices B and D) The heart and liver can be affected by glycogen storage diseases resulting from a variety of enzyme deficiencies such as glucose-6-phosphatase deficiency (von Gierke disease) and acid maltase deficiency (Pompe disease). (Choice E) The pancreas is affected in patients with cystic fibrosis, which results from a mutation in the cystic fibrosis transmembrane conductance regulator gene. Common sequelae include pancreatitis, pancreatic insufficiency, and destruction of islet cells. Educational objective: Gout occurs with increased frequency in patients with activating mutations involving phosphoribosyl pyrophosphate synthetase due to increased production and degradation of purines.

A 6-hour-old boy is evaluated in the newborn nursery due to temperature instability, grunting, and lethargy. The patient was born at full term to a 21-year-old woman who came to the emergency department in active labor with ruptured membranes; she had not received adequate prenatal care. On physical examination, the boy is hypothermic, tachycardic, and tachypneic, and he has mottled skin. Blood samples are obtained, and empiric antibiotics are initiated for the treatment of sepsis. Cultures on blood agar grow bacterial colonies with a narrow zone of clear hemolysis. When these bacteria are grown in the presence of Staphylococcus aureus, the zone of hemolysis is enhanced around the staphylococcal colonies. Which of the following is the most likely pathogen infecting this patient? Escherichia coli Enterococcus faecalis Staphylococcus epidermis Streptococcus agalactiae Streptococcus pneumoniae

ID-15578 Streptococcus agalactiae (group B Streptococcus [GBS]) is often a normal commensal of the genitourinary and gastrointestinal tracts of pregnant women. Those who are colonized and do not receive intrapartum antibiotic prophylaxis can transmit the bacteria to the fetus during passage through the vaginal canal, leading to neonatal sepsis (eg, irritability, lethargy, grunting, tachypnea, temperature instability, hypotension) within hours of birth. GBS is a leading cause of early neonatal sepsis and can be identified by the following laboratory features: Narrow zone of beta-hemolysis (clear) when plated on blood agar. Production of CAMP factor, a phospholipase that enhances the activity of beta-hemolysins secreted by certain strains of Staphylococcus aureus. Testing for CAMP factor (CAMP test) can confirm the presence of GBS; this test plates suspected GBS colonies perpendicular to an appropriate S aureus strain and evaluates for enhanced areas of hemolysis (arrow-shaped zones in the image above). GBS is also catalase-negative (as are all streptococci), contains the Lancefield group B antigen in its cell wall, and hydrolyzes hippurate. (Choice A) Escherichia coli, a gram-negative rod, is a common cause of early neonatal sepsis. Although E coli generates beta-hemolysins, it does not produce CAMP factor; therefore, enhancement of S aureus hemolysis does not occur. (Choice B) Enterococcus faecalis, a gram-positive cocci, is a common cause of sepsis in preterm neonates but is rare in those born at term. In addition, it does not hemolyze red blood cells (gamma hemolytic) or generate camp factor. (Choice C) Staphylococcus epidermidis, a gram-positive coccus that grows in clusters, is a common skin commensal that does not cause hemolysis when plated on blood agar. (Choice E) Streptococcus pneumoniae, a lancet-shaped, gram-positive bacterium, is alpha-hemolytic (partial, green hemolysis) in aerobic conditions and beta-hemolytic in anaerobic conditions. It does not generate CAMP factor. Educational objective: Streptococcus agalactiae (group B Streptococcus), one of the most common causes of early neonatal sepsis, is typically identified by the presence of a narrow zone of beta-hemolysis when plated on blood agar. It is also identified by the production of CAMP factor, which enhances beta-hemolysis by particular strains of Staphylococcus aureus.

A 62-year-old man is hospitalized with severe abdominal pain and diarrhea after a recent urinary tract infection. Six months ago, the patient had an episode of Clostridioides difficile colitis after being treated for pneumonia. Medical history is also significant for diverticulitis and upper gastrointestinal tract bleeding. The patient is allergic to penicillin. An appropriate work-up confirms C difficile colitis. He is administered an oral macrolide antibiotic that inhibits the sigma subunit of RNA polymerase. Which of the following agents was most likely initiated in this patient? Dxycycline Fidaxomicin Metronidazole Neomycin Vancomycin

ID=10401 Treatment options for Clostridioides difficile infection (CDI) typically include oral fidaxomicin or oral vancomycin. Fidaxomicin is a macrolide antibiotic that inhibits the RNA polymerase sigma subunit, the component involved in binding promoter DNA so that transcription can be initiated. Inhibition of transcription leads to impaired protein synthesis and eventual cell death (bactericidal activity against C difficile). Fidaxomicin is administered orally and has minimal systemic absorption, resulting in high concentrations in the colon and feces. It also has a narrow spectrum of activity, with a lesser effect on normal colonic flora than vancomycin. (Choice A) Doxycycline binds to the 30S ribosomal subunit and is effective in clostridial skin infections (eg, Clostridium perfringens, Clostridium septicum), but it is not effective for the treatment of CDI. (Choice C) Metronidazole interacts with DNA in anaerobic organisms, disrupting DNA structure and causing strand breakage. Oral metronidazole is no longer a first-line treatment for CDI. Intravenous metronidazole can be a useful adjunct in patients with fulminant CDI (eg, megacolon, ileus, hypotension) because passage of oral antibiotics to the colon is often delayed. (Choice D) Neomycin is a bactericidal aminoglycoside that inhibits the 30S ribosomal subunit. It can be used to treat hepatic encephalopathy or diarrhea due to Escherichia coli and as surgical prophylaxis for gastrointestinal procedures. However, it is not effective against C difficile. (Choice E) Oral vancomycin has minimal systemic absorption and is bacteriostatic at the concentrations typically used for CDI treatment. Although it is a first-line treatment option for CDI, its mechanism of action is inhibition of cell wall synthesis, not inhibition of RNA polymerase. Educational objective: Clostridioides difficile infection can be treated with oral fidaxomicin or oral vancomycin. Fidaxomicin is a macrolide antibiotic that inhibits RNA polymerase. It is bactericidal against C difficile.

A 29-year-old woman comes to the emergency department due to high fever and a rash. The illness began 5 days ago with acute-onset high fever, headache, retroorbital pain, and joint and muscle pain. Yesterday, the patient developed a diffuse rash, and this morning she had a nosebleed. The patient returned from a trip to Brazil 10 days ago. Her boyfriend traveled with her and is asymptomatic. Examination shows a diffuse maculopapular rash and scattered petechiae over her extremities. Laboratory evaluation reveals thrombocytopenia. Transmission of the pathogen causing this patient's current symptoms occurs in a manner that is most similar to which of the following diseases? Chikungunya Lymphogranuloma venerum Meningococcal meningitis Pulmonary tuberculosis Rocky Mountain spotted fever Typhoid fever

ID=11377 This patient traveled to Brazil and subsequently developed a high fever and hemorrhagic manifestations (eg, petechiae, thrombocytopenia, mucosal bleeding). The likely diagnosis is dengue, a Flavivirus infection transmitted by female Aedes mosquitoes. Dengue virus causes approximately 400 million infections per year and is especially prevalent in tropical and subtropical regions of Asia and South America (eg, Brazil). Manifestations typically arise 4-7 days after transmission and include sudden-onset high fever, headache, retroorbital pain, arthralgia, and myalgia. Hemorrhagic manifestations such as petechiae, purpura, or mucosal bleeding (eg, epistaxis, melena) often occur within days of symptom onset and can be life-threatening. Laboratory evaluation frequently reveals thrombocytopenia, leukopenia, and hemoconcentration. Because Aedes mosquitoes also transmit the viruses that cause yellow fever, Zika, and chikungunya, there are often simultaneous outbreaks in endemic areas during the rainy seasons when mosquito populations peak. Patients are also at high risk for coinfection with multiple mosquito-borne viral pathogens during these periods. (Choice B) Lymphogranuloma venereum is an ulcerative genital disease caused by specific serovars for Chlamydia trachomatis, which is transmitted by direct sexual contact with an infected individual. (Choice C) Meningococcal meningitis is a life-threatening bacterial infection that typically starts with acute-onset myalgia, high fever, and vomiting and rapidly progresses to petechial rash and circulatory collapse. The pathogen (Neisseria meningitidis) is transmitted by respiratory droplets. (Choice D) Pulmonary tuberculosis typically causes asymptomatic initial infection but may reactivate and lead to cough, hemoptysis, and cavitary pulmonary disease. Mycobacterium tuberculosis is spread primarily through airborne droplet nuclei. (Choice E) Rocky Mountain spotted fever is a tick-borne illness caused by Rickettsia rickettsii, a gram-negative obligate intracellular pathogen. It typically presents with fever and nonspecific symptoms followed a few days later by a petechial rash. (Choice F) Typhoid fever is a Salmonella infection that typically causes a stepwise, progressive illness marked by fever in week 1, abdominal pain and a rose-colored rash in week 2, and hepatosplenomegaly and intestinal perforation in week 3. It is transmitted primarily by contaminated food or water ingestion in areas with poor sanitation. Educational objective: Aedes mosquitoes transmit the viruses that cause dengue, chikungunya, Zika, and yellow fever; simultaneous outbreaks and coinfections are common. Dengue usually presents as an acute febrile illness with headache, retroorbital pain, arthralgia, and myalgia; hemorrhagic signs (eg, thrombocytopenia, petechiae, mucosal bleeding) often occur within days.

A 66-year-old postmenopausal woman comes to the office to discuss her bone densitometry results. She has a history of hypertension and has smoked a pack of cigarettes daily for 30 years. BMI is 20.1 kg/m2. Physical examination is unremarkable. Bone densitometry reveals low bone density consistent with osteoporosis. The patient is prescribed risedronate. Repeat bone densitometry 2 years later reveals no further loss of bone mineral density. Which of the following mechanisms most likely explains the drug's effect on bone mineral density? Decreased osteocyte-mediated inhibition of bone formation Decreased stimulation of receptor activator of nuclear factor kappa-B Decreased urinary calcium excretion Increased intestinal calcium absorption Inhibition of osteoclast-mediated bone resorption Stimulation to preosteoblasts to mature osteoblasts

ID=11564 Review Table Bisphosphonates (eg, alendronate, risedronate) treat osteoporosis by inhibiting osteoclast-mediated bone resorption. They have a chemical structure similar to that of pyrophosphate and attach to hydroxyapatite binding sites on bony surfaces. Osteoclasts that resorb the bone take up the bisphosphonate and are unable to adhere to the bony surface to continue resorption. Bisphosphonates also decrease osteoclast proton production, induce osteoclast apoptosis, and decrease development/recruitment of osteoclast precursor cells. The net result is to slow the rate of bone loss, and some patients may experience a small increase in bone mineral density. (Choice A) Sclerostin is a glycoprotein produced by osteocytes that inhibits osteoblast bone formation. Monoclonal antibody preparations (eg, romosozumab) that bind sclerostin reverse this effect and facilitate increased osteoblast activity. (Choice B) Denosumab is a monoclonal antibody that decreases bone resorption by binding to the receptor activator of nuclear factor kappa-B ligand (RANK-L) and blocking the interaction between RANK-L and RANK (a receptor located on osteoclast surfaces), which is required for preosteoclast maturation to osteoclasts. (Choices C, D, and F) Teriparatide is a recombinant formulation identical to the 34-amino-acid sequence at the N-terminal portion of endogenous parathyroid hormone (PTH). Similar to PTH, teriparatide stimulates maturation of preosteoblasts into bone-forming osteoblasts that lay down collagen and eventually mineralize the matrix. Teriparatide also increases gastrointestinal calcium absorption and renal tubular calcium reabsorption, which leads to decreased urinary calcium excretion. Educational objective: Bisphosphonates have a chemical structure similar to that of pyrophosphate and attach to hydroxyapatite binding sites on bony surfaces to inhibit bone resorption by osteoclasts.

A 3-year-old girl is brought to her pediatrician for a well child checkup. She has met all of the appropriate developmental milestones. Her height corresponds to the 60th percentile. Osteoblasts near the growth plates of her long bones secrete matrix material, and when they become trapped in the ossified matrix, they become known as osteocytes. These osteocytes remain connected to each other by: Tight junctions Hemidesmosomes Intermediate junctions Gap junctions Desmosomes

ID=1159 Within a single Haversian system, the central canal is encircled by multiple concentric lamellae of bony matrix that each contains lacunae filled with osteocytes and extracellular bone fluid. Delicate canaliculi radiate from each lacuna to create a reticular network with adjacent lacunae, and the cytoplasmic processes of the osteocytes lie within these canaliculi. These cytoplasmic processes send signals to and exchange nutrients and waste products with the osteocytes within neighboring lamellae via gap junctions. The osteocytes serve to maintain the structure of the mineralized matrix and control the short-term release and deposition of calcium (i.e., calcium homeostasis). The plasma calcium concentration directly dictates the metabolic activity of osteocytes, while parathyroid hormone and calcitonin indirectly influence their metabolic activity. Osteocytes can also sense mechanical stresses and send signals to modulate the activity of surface osteoblasts, thereby helping to regulate bony remodeling. (Choice A) Tight junctions (zonula occludens) are observed at the apices of glandular cells and consist of two closely adherent cytoplasmic membranes without an intervening space. Tight junctions are the first component of the junctional complex. (Choice B) Hemidesmosomes are half desmosomes that extend from the basal surfaces of keratinocytes in the stratified squamous epithelium to attach to the basal lamina. (Choice C) Intermediate junctions (zonula adherens) are a delicate network of cytoplasmic filaments that radiate from the cell membrane to hold adjacent cells together. Intermediate junctions are the second component of the junctional complex. (Choice E) Desmosomes are small, circular, adherent patches circumferentially placed around cells that comprise the third component of the junctional complex. These patches are particularly common in stratified squamous epithelium and contribute significantly to the structural cohesiveness of tissues subject to mechanical stressors. Educational objective: Osteocytes have long intracanalicular processes that extend through the ossified bone matrix. These cytoplasmic processes send signals to and exchange nutrients and waste products with the osteocytes within neighboring lamellae via gap junctions. Osteocytes can sense mechanical stresses and send signals to modulate the activity of surface osteoblasts, thereby helping to regulate bony remodeling.

A 42-year-old homeless man comes to the emergency department due to painful muscle spasms. The patient has had frequent involuntary contractions of the jaw, neck, and trunk muscles, which are triggered by sensory stimuli. He has a history of injection drug use and has had several injection site infections. Physical examination shows neck stiffness and decreased opening of the jaw. Which of the following is the most likely path taken by the agent responsible for this patient's condition? Bite wound>peripheral nerves>CNS neurons Contaminated food>systemic circulation>presynaptic nerve terminals Infected wound>systemic circulation>brain parenchyma Nasopharyngeal exudate>systemic circulation>neurons Oral mucosa>trigeminal nerve>temporal lobe Puncture wound>motor neurons>spinal cord interneurons

ID=1402 This patient with painful muscle spasms and neck/jaw contraction likely has tetanus, which is caused by the spore-forming anaerobic bacterium Clostridium tetani. C tetani spores are found in soil worldwide and are typically transmitted to humans during a puncture injury (eg, dirty needle use, splinter). The low oxygen tension of a puncture wound allows spores to germinate into vegetative rod-shaped bacteria that proliferate locally and produce tetanospasmin (tetanus toxin). Although the pathogen is noninvasive (it stays localized to the wound site), the toxin enters the presynaptic terminals of the lower motor neurons and travels by retrograde axonal transport to the CNS. In the spinal cord, at the level of the anterior horn cells, the toxin blocks inhibitory interneurons, leading to spasmodic muscle contraction. Classic findings include jaw stiffness due to masseter muscle spasm (trismus/lockjaw) and sustained contraction of the facial muscles, producing a bizarre "smiling" appearance (risus sardonicus). Spasmodic contraction of the back and neck can cause opisthotonos, and respiratory muscle involvement that can lead to respiratory failure. (Choice A) Rabies virus is transmitted via the saliva of an infected animal during a bite wound. The virus spreads by retrograde axonal transport through peripheral nerves and enters the spinal cord/CNS. Patients with rabies usually develop painful pharyngeal spasms, hydrophobia, fever, and hyperactivity. However, this patient has no history of an animal bite; he likely developed tetanus due to a puncture wound with a contaminated needle. (Choice B) Food-borne botulism is marked by the ingestion of preformed botulism toxin with subsequent spread through the systemic circulation. The toxin blocks cholinergic presynaptic nerve terminals, which results in symmetric, descending weakness. (Choice C) Bacteria (eg, Staphylococcus aureus) from a wound can spread through the bloodstream to the brain leading to brain abscess. Symptoms vary but the most common manifestation is unilateral headache. (Choice D) Diphtheria is associated with the formation of a pseudomembranous pharyngeal exudate and the elaboration of diphtheria toxin into the systemic circulation. The toxin causes cardiac (myocarditis) and neurologic toxicity. (Choice E) Herpes simplex virus type 1 causes orolabial lesions and is the most frequent cause of sporadic encephalitis, which occurs when the virus spreads via the olfactory tract to the temporal lobe (olfactory cortex). Patients usually present with confusion, fever, focal neurologic deficits, and seizures. Educational objective: Tetanus is caused by infection with toxigenic strains of the anaerobic bacterium Clostridium tetani. Transmission typically occurs when spores are inoculated into the skin via a puncture wound. The bacteria germinate, replicate locally, and elaborate tetanospasmin (tetanus toxoid). The toxin spreads in a retrograde fashion through the lower motor neurons to the spinal cord, where it blocks inhibitory interneurons and causes spasmodic muscle contraction (eg, trismus, risus sardonicus).

A 31-year-old woman comes to the office due to a 4-month history of left hip and groin pain. She has a constant, dull, achy pain at rest that is exacerbated by movement of the hip or weight bearing. The patient has had no recent falls or other trauma. Medical history is significant for sickle cell disease, pneumococcal pneumonia, and acute chest syndrome. There is no redness, warmth, or tenderness over the hip joint, but decreased active and passive internal and external rotation, extension, and abduction are noted. Imaging of the hip is shown in the exhibit. Which of the following is the most likely cause of this patient's pain? Absence of cartilage in the epiphyseal plate Cytokine-mediated increased bone resorption Cytokine-mediated new bone formation Necrosis of bone and marrow cells Synovial pannus formation

ID=316 This patient has chronic hip pain that is worse with weight bearing and decreased range of motion in multiple axes. X-ray reveals flattening and patchy sclerosis of the femoral head consistent with osteonecrosis (avascular necrosis). Biopsy is not generally required for diagnosis, but microscopic analysis of the affected bone may show dead bony trabeculae (empty lacunae) and fat necrosis. Osteonecrosis occurs due to disruption of the macrovasculature or microvasculature of bone. The femoral head is the most common location, although other bones may be affected. Conditions associated with osteonecrosis include the following: Sickle cell disease can cause thrombotic occlusion of arteries. Embolic disorders (eg, fat emboli, decompression sickness) can directly occlude small arteries. High-dose systemic corticosteroids and excessive alcohol intake can cause fat embolism and adipocyte hypertrophy in the marrow. Vasculitis (eg, systemic lupus erythematosus) can injure the vessel wall, resulting in distal ischemia. Femoral neck fracture can disrupt the medial circumflex femoral artery, which is the main arterial supply for the femoral head. (Choice A) Achondroplasia is an autosomal dominant disorder characterized by severely restricted chondrocyte proliferation in growth plate cartilage and decreased endochondral ossification. The shortened limbs and other features of this disorder are usually apparent in childhood. (Choices B and E) The chronic joint destruction in rheumatoid arthritis (RA) is associated with the formation of synovial pannus, an invasive mass composed of fibroblast-like synovial cells, granulation tissue, and inflammatory cells. At the same time, the affected joints undergo a cytokine-mediated increase in bone resorption with loss of subchondral bone that is visible on x-ray as periarticular erosions. Although RA can affect the hip, it typically affects the small joints of the hand and wrist; hip monarthritis is rare. (Choice C) Ankylosing spondylitis (AS) is characterized by bony erosions with excessive bone regrowth driven by overproduction of cytokines (eg, IL-17, IL-22, TNF-alpha). AS can affect the hip, but most patients have chronic back and sacroiliac pain; large-joint monarthritis is rare. Educational objective: Osteonecrosis (avascular necrosis) occurs due to disruption of the macrovasculature or microvasculature of bone. The femoral head is the most common location. Sickle cell disease can cause osteonecrosis due to the thrombotic occlusion of arteries. Other major risk factors include embolic disorders, high-dose systemic corticosteroids, excessive alcohol use, vasculitis (eg, systemic lupus erythematosus), and femoral neck fracture. References

A 34-year-old woman comes to the physician for a follow-up visit. She was diagnosed with rheumatoid arthritis 3 months ago and started on methotrexate therapy. Despite treatment, she continues to have several hours of morning stiffness daily and frequently awakens at night due to joint pain. Physical examination shows swelling and tenderness in the joints of her hands and wrists. Etanercept is subsequently added to her treatment regimen. This medication is best characterized as which of the following? Cell surface receptor antibody Chimeric monoclonal antibody Humanized monoclonal antibody Small-molecule receptor inhibitor Soluble receptor decoy protein

ID=8523 Etanercept is a tumor necrosis factor-alpha (TNF-α) inhibitor added to methotrexate to treat moderate-to- severe rheumatoid arthritis in patients who have failed methotrexate alone. It is a fusion protein linking a soluble TNF-α receptor to the Fc component of human immunoglobulin G1 (IgG1). Etanercept reduces the biological activity of TNF-α by acting as a decoy receptor: the TNF-α receptor component acts like a sponge to bind TNF-α and keep it away from functional TNF-α receptors, while the Fc component stabilizes the complex. Pharmaceutical companies provide the prefix of the names for biological agents; the suffix indicates whether the medication is a monoclonal antibody (mab), a receptor molecule (cept), or a kinase inhibitor (nib). Monoclonal antibodies, which are the largest group of biological agents, also include in their names the type of target (eg, bacterial or immune system) and their origin (eg, human or mouse). (Choice A) Rituximab (Ri-tu-xi-mab) is a chimeric monoclonal antibody targeted against CD20, a cell receptor found on the surface of B cells. It is used to treat CD20+ non-Hodgkin's lymphomas and other diseases related to excessive B-cell function. (Choice B) Infliximab (Inf-li-xi-mab) is a chimeric monoclonal antibody targeted against TNF-α. It is used in the treatment of a number of autoimmune diseases (eg, rheumatoid arthritis and Crohn's disease). (Choice C) Certolizumab pegol (Certo-li-zu-mab) is a pegylated (pegol) humanized monoclonal antibody that targets TNF-α. It lacks the Fc region, which helps minimizes complement activation and cell-mediated cytotoxicity that can occur with other anti-TNF-α medications that contain Fc domains. (Choice D) Imatinib (Imati-nib) mesylate is used to treat specific cancers, including Philadelphia chromosome-positive chronic myelogenous leukemia and kit-positive gastrointestinal stromal tumors. It is an example of a small-molecule tyrosine kinase receptor inhibitor. Educational objective: The suffix of a biological agent indicates whether a medication is a monoclonal antibody (mab), a receptor molecule (cept), or a kinase inhibitor (nib). Monoclonal antibodies also include in their names the type of target (eg, bacterial or immune system) and their origin (eg, human or mouse).

A 36-year-old man is evaluated due to swelling and severe pain of the right foot. Medical history is significant for kidney transplant due to focal segmental glomerulosclerosis. The patient is diagnosed with gout and treated with prednisone. Six weeks later, the patient is seen at follow-up. He has no foot pain and the swelling has subsided. Serum uric acid level is 13.5 mg/dL and creatinine level is 0.9 mg/dL. Allopurinol is prescribed. Initiation of this drug would most likely increase the activity of which of the following medications? Azathioprine Ganciclovir Prednisone Tacrolimus Trimethorprim

In the purine degradation pathway, purine bases (ie, hypoxanthine, guanine) can follow 1 of 2 routes: Approximately 10% are converted by xanthine oxidase to uric acid, which is released into the bloodstream and excreted in the urine. The other 90% are recycled into purine nucleotides by hypoxanthine-guanine phosphoribosyltransferase (HGPRT) to be reused in the formation of DNA and RNA (ie, purine salvage). Gout occurs due to elevated serum uric acid levels, leading to the formation of inflammation-inducing uric acid crystals within joints (eg, metatarsals). Xanthine oxidase inhibitors (eg, allopurinol, febuxostat) treat gout by reducing uric acid levels and in the process shunt additional purine bases toward the salvage route. Azathioprine is an immunosuppression drug that is metabolized via the purine degradation pathway. Following ingestion, azathioprine is converted to 6-mercaptopurine (6-MP), which is a false purine base that, like hypoxanthine and guanine, can follow 1 of 2 routes. Some 6-MP is converted by xanthine oxidase (or thiopurine methyltransferase) into inactive metabolites that are excreted in the urine. The remainder is converted by HGPRT into active metabolites (6-thioguanines), which act as false nucleotides that incorporate into DNA and RNA and inhibit hematopoietic cell proliferation. Therefore, when a xanthine oxidase inhibitor is combined with azathioprine, 6-MP is shunted toward conversion into active metabolites, leading to increased immunosuppression and risk of cellular toxicity. (Choice B) Ganciclovir is similar in structure to guanosine and forms false purine nucleotides that disrupt DNA synthesis. It is relatively specific for viral DNA replication enzymes; it does not interact with allopurinol because it is not catabolized by xanthine oxidase. (Choices C and D) Prednisone exerts a nonspecific immunosuppressive effect by inhibiting NF-kB to regulate the transcription of leukoproliferative cytokines. Tacrolimus is a calcineurin inhibitor that suppresses T cells by preventing IL-2 transcription. Neither drug targets the purine degradation pathway and therefore is not significantly influenced by xanthine oxidase inhibition. (Choice E) Trimethoprim inhibits dihydrofolate reductase to disrupt folate metabolism and inhibit the production of pyrimidine nucleotides. It is not affected by the inhibition of xanthine oxidase in the purine degradation pathway. Educational objective: Azathioprine is an immunosuppression drug that is metabolized into active metabolites by HGPRT and, conversely, inactivated by xanthine oxidase. Coadministration with a xanthine oxidase inhibitor (eg, allopurinol, febuxostat) shunts azathioprine metabolism toward the production of active metabolites, resulting in increased immunosuppression and risk of cellular toxicity.

A group of investigators is studying osteoprotegerin function in patients with postmenopausal osteoporosis. Using an engineered mouse strain, they develop a chimeric monoclonal antibody that blocks binding of receptor activator of nuclear factor kappa B (RANK) to RANK ligand (RANK-L). The investigators inject this monoclonal antibody into human subjects. Eight weeks later, a bone biopsy is obtained from all subjects. Which of the following is the most likely observed finding? Decreased bone mineralization Decreased bone resorption Increased osteoblast activity Increased osteoclast activity Increased survival of osteocytes

Interaction between the receptor for activated nuclear factor kappa B (RANK) and its ligand (RANK-L), is critical for the development of mature, multinucleated osteoclasts. This binding is blocked by osteoprotegerin (OPG), which acts as a decoy receptor for RANK-L (preventing it from interacting with RANK). By binding RANK-L, OPG reduces the differentiation and survival of osteoclasts, resulting in decreased bone resorption and increased bone density. Estrogen maintains bone mass in premenopausal women by inducing production of OPG by osteoblasts and stromal cells and decreasing expression of RANK on osteoclast precursors. The loss of OPG at menopause leads to increased osteoclast activity that predisposes to osteoporosis. Denosumab is a monoclonal antibody used in the treatment of postmenopausal osteoporosis. It works similar to OPG in that it binds RANK-L and prevents its interaction with RANK receptor. Denosumab therefore causes decreased osteoclast differentiation and activity (Choice D) as well as decreased bone resorption. (Choices A and C) RANK (receptor) is not present on osteoblasts, and inhibition of binding of RANK-L to RANK does not affect osteoblast activity or bone mineralization. (Choice E) Osteocytes are derived from osteoblasts. Osteocyte survival is regulated by a variety of factors including mechanical stress, local growth factors, and steroid hormone levels. Although osteocytes produce RANK-L, which acts on osteoclasts, osteocyte survival is not dependent on the RANK signaling pathway. Educational objective: The nuclear factor kappa B (RANK)/RANK-ligand (RANK-L) interaction is essential for the formation and differentiation of osteoclasts. Osteoprotegerin (OPG) blocks binding of RANK-L to RANK and reduces formation of mature osteoclasts, leading to decreased bone resorption. Denosumab is a monoclonal antibody used in the treatment of postmenopausal osteoporosis that works in a manner similar to OPG.

A married couple comes to the physician for routine prenatal counseling. The husband is 120 cm (3 ft 11 in) tall with disproportionately short upper and lower extremities, a large head, and a prominent forehead. He is unable to provide a biological family history as he was adopted. His spouse is of average height with normal constitutional features, and her family history is insignificant. They are concerned about their unborn child's height. Which of the following is the best response to their concerns? The condition is not inheritable The risk depends on the child's biological sex The risk depends on the mother's carrier status The risk for the child to be short is about 25% The risk for the child to be short is about 50%

The husband's short stature and morphologic features are suggestive of achondroplasia. Achondroplasia is the most common form of short-limbed dwarfism and is caused by a mutation that results in constitutive activation of fibroblast growth factor receptor 3 (FGFR3). Achondroplasia occurs as a sporadic mutation (due to advanced paternal age) in 85% of cases. However, once a mutation occurs, it can be transmitted as an autosomal dominant trait (responsible for the remaining 15% of cases) (Choice A). Only 1 mutant copy of the FGFR3 gene is sufficient to cause the disorder; 2 copies of the mutant gene (ie, homozygosity) are lethal. As a result, the husband must be heterozygous for the achondroplasia mutation. A heterozygous parent has a 50% chance of transmitting an autosomal dominant mutation. Therefore, the unborn child has a 50% chance of inheriting achondroplasia. Because achondroplasia is a rare condition, the chance of the unborn child having a sporadic (de novo) mutation does not significantly add to the 50% risk of inheriting the disease. (Choice B) In sex-linked disorders, the responsible gene is located on a sex chromosome (either X or Y). Most sex-linked disorders are X-linked recessive (XLR). In XLR disorders, women with a copy of the defective gene will not have the disorder but will be carriers; men who inherit the defective gene will be affected. In X-linked dominant disorders, both males and females are affected following inheritance of the defective gene. (Choice C) Inheritance of an AD trait does not always result in the disease phenotype in conditions with incomplete penetrance. However, achondroplasia is a fully penetrant genetic disorder; if the mother carried the trait, she would have short stature. (Choice D) About 25% of children are affected in autosomal recessive (AR) disorders if both parents carry 1 copy of the defective gene (most common scenario). Many AR disorders are the result of enzymatic deficiencies (eg, phenylketonuria) that require both copies of the gene to be knocked out, as 1 functional copy usually provides sufficient activity to prevent occurrence of the disease. Educational objective: Achondroplasia is an autosomal dominant (AD) disorder that results in a gain-of-function mutation in the FGFR3 gene. Most individuals affected by AD disorders are heterozygous and have a 50% chance of transmitting the mutation to their offspring.

A 22-year-old woman participates in a research study. She is asymptomatic and has not had a respiratory illness in the last 8 weeks. She reports no chronic medical problems and takes no medications. A nasopharyngeal specimen is obtained and cultured on a selective medium containing vancomycin, colistin, nystatin, and trimethoprim. The organism most likely to be isolated is typically associated with which of the following conditions? Cystitis Dental caries Diphtheria Meningitis Otitis media Pneumonia Rheumatic heart disease

The nasopharynx is colonized with a wide range of gram-positive and gram-negative bacteria. Therefore, nasopharyngeal swabs are often cultured on media containing antibiotics to isolate a specific organism. The agar described in this case is the Thayer-Martin medium, used to identify pathogenic Neisseria organisms such as N meningitidis (a common cause of meningitis) and N gonorrhoeae (a common cause of urethritis). The Thayer-Martin medium is a chocolate sheep blood agar infused with vancomycin to inhibit gram-positive bacteria; with colistin and trimethoprim to inhibit gram-negative bacteria (other than pathogenic Neisseria species); and with nystatin to inhibit yeast. (Choice A) The most common cause of cystitis is Escherichia coli, a gram-negative rod inhibited by colistin and trimethoprim. E coli is not a common nasopharyngeal organism. (Choice B) Dental carries are caused primarily by Streptococcus (eg, S mutans) and Lactobacillus species, gram-positive organisms inhibited by vancomycin. (Choice C) Corynebacterium diphtheriae is the cause of diphtheria. It usually presents with slow-onset sore throat, malaise, and low-grade fever. C diphtheriae is a gram-positive organism inhibited by vancomycin. (Choices E and F) The most common cause of both otitis media and community-acquired pneumonia in adults is Streptococcus pneumoniae. Although S pneumoniae can colonize the nasopharynx, this gram-positive organism is inhibited by vancomycin. (Choice G) Rheumatic heart disease is caused by Streptococcus pyogenes (group A streptococci), a common gram-positive pharyngeal pathogen. S pyogenes is inhibited by vancomycin. Educational objective: The Thayer-Martin medium is used to isolate pathogenic Neisseria species such as N meningitidis and N gonorrhoeae. It is a chocolate sheep blood agar that contains vancomycin to inhibit the growth of gram-positive organisms; colistin and trimethoprim to inhibit gram-negative bacteria (other than pathogenic Neisseria); and nystatin to inhibit yeasts.

A 54-year-old man comes to the office due to severe foot pain. The patient attended a wedding reception where he drank several alcoholic beverages, then woke the following morning with pain. He has a long history of gouty arthritis, and his current symptoms are similar to previous flares of the disease. Past medical history includes type 2 diabetes mellitus and recently diagnosed peptic ulcer disease. Physical examination shows erythema, warmth, and swelling at the left first metatarsophalangeal joint. The patient is started on a new medication for gout that provides significant relief of his symptoms, but he returns to the clinic a week later with diarrhea and persistent nausea. The drug used in this patient most likely affects which of the following cell structures? Cytoskeleton Golgi apparatus Microsomes Nucleus Peroxisomes Rough endoplasmic reticulum Smooth endoplasmic reticulum

The preferred first-line treatment for acute gouty arthritis is a nonsteroidal anti-inflammatory drug, but these are contraindicated in patients with a recent history of peptic ulcer. Other treatment options include colchicine and oral or intraarticular glucocorticoids. Colchicine binds to the intracellular protein tubulin and inhibits its polymerization into microtubules. This, in turn, disrupts cytoskeletal-dependent functions such as chemotaxis and phagocytosis. Colchicine is administered initially at the first signs of a gout flare and can be repeated an hour later. It may also be used for prophylaxis while initiating urate-lowering therapy (eg, allopurinol). Important adverse effects of colchicine include nausea, abdominal pain, and diarrhea, which are most common at higher doses. Colchicine should be avoided in patients who are elderly or have severe renal dysfunction. Educational objective: Colchicine inhibits tubulin polymerization into microtubules and can be used for acute treatment and prophylaxis of gout. Important side effects of colchicine include nausea, abdominal pain, and diarrhea.

A 34-year-old man comes to the office due to progressive low back pain for several years. The patient reports that the stiffness and pain are worst in the morning but gradually improve during the day. Lately, he has been taking over-the-counter ibuprofen, which provides some relief. Laboratory evaluation shows a positive HLA-B27 antigen. X-ray imaging reveals fusion of the sacroiliac joints. Which of the following is most helpful to monitor disease progression in this patient? Degree of chest expansion Hand joint mobility Peak expiratory flow rate Rheumatoid factor level Urine protein level

The symptoms of low back pain and morning stiffness in a young man suggest ankylosing spondylitis (AS), a chronic inflammatory condition associated with the HLA-B27 serotype. AS is characterized by stiffness and fusion (ankylosis) of the axial joints. The sacroiliac and apophyseal joints of the spine are the most commonly affected, leading to restricted spinal mobility. Many patients also develop peripheral arthritis and enthesitis, which is defined as pain, tenderness, and swelling at the sites of tendon insertion into bone (eg, Achilles tendon insertion). In addition, AS can cause complications in extraskeletal systems: Respiratory: Involvement of the thoracic spine and enthesopathies of the costovertebral and costosternal junctions can limit chest wall expansion, leading to hypoventilation. Chest expansion should be monitored regularly in patients with AS. Cardiovascular: The most common cardiovascular complication of AS is ascending aortitis, which can lead to dilation of the aortic ring and aortic insufficiency. Eye: Anterior uveitis develops in some patients with AS and presents with pain, blurred vision, photophobia, and conjunctival erythema. (Choice B) Hand joint mobility testing is part of the assessment of rheumatoid arthritis severity. However, the small joints of the hands are not prominently affected by AS. (Choice C) Peak expiratory flow is determined by airway resistance (obstruction) and chest and abdominal wall muscular function. These variables are not affected by chest wall restriction and are normal in AS. Regular measurement of expiratory flow rates is important in patients with asthma. (Choice D) Rheumatoid factor can be found in a variety of rheumatic (eg, rheumatoid arthritis, Sjögren's syndrome, systemic lupus erythematosus [SLE]) and nonrheumatic disorders (eg, hepatitis C) and in up to 4% of young and healthy individuals. However, it is not associated with AS. (Choice E) Urine protein excretion should be monitored in SLE and diabetes mellitus. Renal involvement is rare in AS. Educational objective: Ankylosing spondylitis is characterized by stiffness and fusion of axial joints (ankylosis) and inflammation at the site of insertion of tendons into bone (enthesitis). Involvement of the thoracic spine and costovertebral and costosternal junctions can limit chest wall expansion, leading to hypoventilation.

A 5-month-old boy is brought to the emergency department due to poor feeding, weakness, and complete loss of extremity muscle tone. Vaccinations are up to date, and there is no significant medical history. The patient receives formula as his sole source of nutrition except for occasional fruit juice and honey. Which of the following is most likely to confirm the diagnosis in this patient? Blood test for liver enzymes Blood test for viral titers Stool test for bacterial toxins Stool test for occult blood Urine test for amino acids Urine test for glucose and ketones

This 5-month-old patient has consumed honey, a food notorious for contamination with Clostridium botulinum spores. When an infant consumes C botulinum spores, the bacteria can germinate in the gastrointestinal tract. Bacteriolysis releases botulinum toxin, which is absorbed systemically and blocks release of acetylcholine from cholinergic presynaptic terminals. Constipation is usually the first manifestation of infant botulism, followed days to weeks later by mild weakness, lethargy, and reduced feeding. In rare, severe cases, infants can have weakened suckling and crying, diminished gag reflex, and symmetric, descending flaccid paralysis with loss of head control that can cause the infant to appear "floppy." In contrast, adult botulism usually results from ingestion of preformed toxin (eg, contaminated canned foods) and is almost always very severe. While infant botulism can be diagnosed based on the clinical presentation and food consumption history, the diagnosis is usually confirmed through identification of C botulinum spores or toxin in stool samples. (Choice A) Measurement of blood liver enzyme levels can indicate damage to hepatic cells in hereditary fructose intolerance (genetic deficiency in aldolase B). Although patients with fructose intolerance can have poor feeding shortly after juice and honey are added to the diet, other classic manifestations include hypoglycemia, vomiting, and hepatomegaly rather than loss of extremity muscle tone. (Choice B) Serum viral titers are frequently used in the evaluation of patients with viral hepatitis or suspected Epstein-Barr or cytomegalovirus infections. Poor feeding with loss of extremity muscle tone is more characteristic of infant botulism. (Choice D) Allergic proctocolitis in infants can present after introduction of different food groups, and the diagnosis can be supported by stool studies that are positive for gross or occult blood. However, these infants usually appear well despite persistent diarrhea and/or rectal bleeding. (Choice E) Patients with amino acids in their urine may have an inborn error of metabolism (eg, maple syrup urine disease) with symptoms usually manifesting in the neonatal period or early infancy. (Choice F) High levels of urine glucose and ketones, together with hyperglycemia and metabolic acidosis, can support a diagnosis of diabetic ketoacidosis. Educational objective: Infant botulism can result from consumption of honey, which frequently contains C botulinum spores that can germinate and produce botulinum toxin. Symptoms of infant botulism include constipation, mild weakness, lethargy, poor feeding, and, in severe cases, flaccid paralysis. The diagnosis can be confirmed by identification of C botulinum spores or toxins in the stool.

A 15-year-old boy is brought to the emergency department due to hemoptysis. He has a history of amputation of the right lower extremity due to bone cancer. Chest imaging reveals a lung mass. Excisional biopsy of the mass shows sheets of uniform, round, small cells slightly larger than lymphocytes with scant, clear cytoplasm. The cellular deposits are interrupted by vascular fibrous septa, with areas of hemorrhage and an abrupt transition from viable to necrotic cells. A representative sample is shown in the image below. ID=15635 Which of the following is the most likely diagnosis? Adenocarcinoma Chondrosarcoma Ewing sarcoma Multiple myeloma Osteosarcoma

This boy with a history of bone cancer has a lung mass composed of uniform, small, round cells, findings consistent with metastatic Ewing sarcoma (ES). ES is the second most common primary malignant bone tumor in children (after osteosarcoma). It typically occurs in the diaphysis of long bones (eg, femur) or in the pelvis and presents with localized pain and swelling. It often metastasizes to the lungs, and pulmonary complications (eg, hemoptysis) are a leading cause of death. Imaging classically reveals a poorly defined, lytic, "moth-eaten" lesion that infiltrates the bony cortex and neighboring soft tissue. The associated periosteal reaction leads to concentric layers of new, reactive bone ("onion skin" appearance). ES is a primitive tumor of unclear origin (likely arising from neuroectodermal or mesenchymal stem cells). Histopathology reveals sheets of uniform, small, round, blue cells with scant, clear cytoplasm separated by fibrous septa and patches of necrosis. Most cases are associated with a translocation involving EWSR1 on chromosome 22 and FLI1 on chromosome 11. This translocation produces a fusion protein that functions as an abnormal transcription factor, leading to uncontrolled cell proliferation. (Choice A) Adenocarcinoma, the most common primary lung malignancy, often metastasizes to bone. Histopathology shows neoplastic glands lined by mucin-producing cells. (Choice B) Chondrosarcoma typically occurs in the axial skeleton (eg, pelvis) of older adults (age >40). Histopathology shows neoplastic chondrocytes in a cartilaginous matrix. (Choice D) Multiple myeloma is a plasma cell malignancy that is extremely rare in patients age <40. Imaging often reveals multiple lytic lesions, osteopenia, and pathologic fractures. Microscopic examination shows numerous plasma cells, characterized by eccentric nuclei with "clock-face" chromatin, perinuclear clearing, and abundant cytoplasm. (Choice E) Osteosarcoma is the most common primary bone malignancy in children. However, histopathology shows pleomorphic malignant cells that produce new osteoid and bone. Educational objective: Ewing sarcoma is a malignant bone tumor that typically arises in the long bones or pelvis in children and often metastasizes to the lungs. Imaging classically reveals a poorly defined, lytic lesion with an "onion skin" periosteal reaction. Histopathology shows sheets of uniform, small, round, blue tumor cells.

A 2-year-old boy is brought to the emergency department due to high fever and malaise for the past 4 days. His parents say that he began limping yesterday and now refuses to walk. The patient has had no recent travel or exposure to anyone with similar symptoms. Temperature is 39.4 C (103 F). Passive range of motion of the hips, knees, and ankles does not elicit pain. There are no joint effusions. The patient refuses to bear weight. An intravenous radiotracer that localizes to areas of increased osteoblastic activity is administered, and several images are obtained to find areas with atypical activity. Abnormally increased uptake of the radiotracer is most likely to be seen in which of the following areas? Flat bone Long bone diaphysis Long bone epiphysis Long bone metaphysis Vertebral body

This child has acute-onset fever, malaise, and refusal to bear weight, findings concerning for osteomyelitis. Older children often have focal bony tenderness, but children age ≤2 may be unable to localize pain. Passive range of motion of the joints typically is normal and helps distinguish osteomyelitis from septic arthritis. The most common cause of osteomyelitis in children is hematogenous spread of bacteria, which may be introduced by minor (often unnoticed) trauma to a distant site. Infection in children classically occurs in the metaphyses of long bones (eg, femur, tibia) because this region is highly vascular yet contains slow-flowing sinusoids that are conducive to bacterial seeding. Bacterial seeding within the metaphysis initially causes acute inflammation of the bone marrow, with focal hyperperfusion and increased radiotracer uptake within the affected region. Persistent inflammation within the confined bony space leads to increased intramedullary pressure, which compromises blood flow and forces infectious exudate into the cortex and periosteum. Without treatment, the infection can progress to chronic suppurative osteomyelitis, in which necrotic bone (ie, sequestrum) serves as an infectious reservoir and sinus tracts develop to drain away the purulent material. (Choice A) Osteomyelitis of flat bones (eg, skull, pelvis) is uncommon in children and is typically caused by contiguous spread of infection (eg, open wound, dental abscess). Although pelvic bone involvement may lead to refusal to bear weight, this patient has no gross abnormality of the hips or lower extremities suggesting adjacent soft tissue infection. (Choice B) Ewing sarcoma typically arises in diaphysis of the long bones, especially the femur. It is the second most common malignant bone tumor in children after osteosarcoma. However, Ewing sarcoma typically presents in children age >10 with an insidious onset of progressive pain, not in toddlers with acute refusal to bear weight. (Choice C) The physis (growth plate) in children limits vascular connections between the metaphysis and epiphysis, which is relatively less vascularized, making epiphyseal involvement uncommon in hematogenous osteomyelitis. Although avascular necrosis of the proximal femoral epiphysis (eg, Legg-Calve-Perthes) can present with a limp, the pain is typically more mild and develops gradually without fever. (Choice E) Hematogenous osteomyelitis in adults occurs more commonly in the vertebral bodies due to increasing vertebral vascularity with age as well as epiphyseal closure during puberty (improves capillary flow within the metaphyses of long bones). Educational objective: Hematogenous osteomyelitis occurs most commonly in children. It usually affects the metaphysis of long bones due to the presence of slow-flowing sinusoids that are conducive to bacterial seeding. Fever and refusal to bear weight are common manifestations in young children who are unable to localize the pain.

A 70-year-old man comes to the clinic due to intermittent leg pain and difficulty walking. He describes the pain as mild-to-moderate, deep, and lasting throughout the day. The patient has a history of osteoarthritis in his hands for which he takes over-the-counter nonsteroidal anti-inflammatory drugs, but says that he never has had problems with his knees. Physical examination shows point tenderness over the right tibia. Laboratory studies show elevated serum alkaline phosphatase. Bone biopsy shows haphazardly oriented segments of lamellar bone with prominent cement lines. The initial phase of this patient's disorder involves increased activity of which of the following cell types? Chondrocytes Endothelial cells Fibroblasts Osteoblasts Osteoclasts

This elderly patient's bone pain, increased alkaline phosphatase, and biopsy findings are characteristic of Paget disease of bone (PDB), a condition that results in accelerated bone remodeling with eventual bony overgrowth. The disease is thought to be caused by environmental factors and gene mutations (eg, affecting RANK, osteoprotegerin) that result in excessive RANK and NF-κB activation. This leads to increased osteoclast differentiation and activity. The disease typically progresses through 3 phases: Osteolytic (osteoclast-dominant) phase - characterized by increased numbers of osteoclasts that appear abnormally large with an excessive number of nuclei. Increased resorption activity is prominent. Mixed (osteoclastic-osteoblastic) phase - defined by a rapid increase in osteoblastic bone formation with persistent osteoclastic activity. The newly made bone is abnormal, with interspersed areas of disorganized lamellar and woven bone. Osteosclerotic (osteoblast-dominant) phase - characterized by continued osteoblastic bone formation and remodelling that result in a dense, hypovascular, mosaic pattern of lamellar bone with irregular, haphazardly oriented sections separated by prominent cement lines. The end result is a thickened, deformed bone that is weaker than normal and prone to fracture. (Choice A) Chondrocytes are not involved in the pathogenesis of PDB. (Choices B and C) In the early stages of PDB, the adjacent marrow spaces are replaced by highly vascular stromal tissue as a result of increased endothelial cell and fibroblast proliferation due to cytokines secreted by osteoclasts. The increased vascularity causes arteriovenous shunting that can result in high-output heart failure. (Choice D) Although this patient's bone biopsy shows findings characteristic of the osteosclerotic (osteoblast-dominant) phase of Paget disease, the question is specifically asking about the initial (osteolytic) phase of Paget disease, where osteoclast activity is predominantly increased. Increased activity and dysregulation of osteoblasts and fibroblasts increase the risk of developing sarcomas (eg, osteosarcoma) in patients with PDB. Educational objective: Bone pain and elevated alkaline phosphatase level in an elderly patient can occur with osteoblast metastases and Paget disease of bone (PDB). Biopsy showing a mosaic pattern of lamellar bone is diagnostic for PDB. The initial phase in PDB is characterized by an increase in osteoclastic activity.

A group of researchers is developing new drugs for osteoporosis. They are testing a new drug, Substance X, that exhibits the following metabolic effects when given via an infusion in varying doses (as shown in the graphs below). ID=641 Which of the following most closely resembles the metabolic effects of Substance X? 1,25-dihydroxyvitamin D analog Fibroblast growth factor 23 inhibitor Pyrophosphate analog Receptor activator of nuclear factor kappa-B ligand (RANK-L) inhibitor Recombinant parathyroid hormone

This investigational drug causes a dose-dependent increase in serum calcium and urine phosphate excretion, which resembles the activity of parathyroid hormone (PTH). PTH receptors are located on osteoblasts (not osteoclasts), and activation causes osteoblasts to increase production of receptor activator of nuclear factor kappa-B ligand (RANK-L) and monocyte colony-stimulating factor. These factors stimulate osteoclastic precursors to differentiate into bone-resorbing osteoclasts. PTH also decreases the release of osteoprotegerin (OPG), a decoy receptor for RANK-L; therefore, lower levels of OPG allow for more interaction between RANK-L and the osteoclastic receptor, increasing bone resorption and releasing calcium and phosphate into circulation. In the kidney, PTH decreases tubular reabsorption of phosphorus while increasing reabsorption of calcium. Therefore, the combined effects are increased serum calcium and urine phosphate levels. Chronically high levels of PTH (ie, hyperparathyroidism) increase the risk of osteoporosis. However, intermittent administration of recombinant PTH analogs induces a greater increase in osteoblast activity in proportion to osteoclast activity and a net increase in new bone formation. Teriparatide is a recombinant PTH analog used to treat osteoporosis. (Choice A) PTH increases renal conversion of 25-hydroxyvitamin D to 1,25-dihydroxyvitamin D, which increases intestinal absorption of calcium and phosphate and decreases PTH secretion via negative feedback. 1,25-dihydroxyvitamin D analogs (eg, calcipotriol) can raise serum calcium to some extent, but suppression of PTH limits the degree of phosphate excretion in the urine. (Choice B) PTH increases renal phosphate excretion by inducing internalization and destruction of type IIa sodium/phosphate cotransporters (NPT2) in the proximal renal tubule. Fibroblast growth factor 23 (FGF23) is a hormone produced by osteocytes that acts synergistically with PTH to increase phosphate excretion by downregulating NPT2 gene expression. Investigational FGF23 inhibitors reduce renal phosphate excretion. (Choices C and D) Bisphosphonates (eg, alendronate, risedronate) are pyrophosphate analogs that attach to hydroxyapatite binding sites on bone surfaces and inhibit osteoclast-mediated bone resorption. Denosumab is a monoclonal antibody that decreases bone resorption by binding to RANK-L and blocking the interaction between RANK-L and RANK on osteoclast surfaces. These agents generally lower, rather than raise, serum calcium levels. Educational objective: Parathyroid hormone (PTH) causes increased bone resorption, increased serum calcium levels, and increased renal phosphate excretion. Chronically high levels of PTH increase the risk of osteoporosis. However, intermittent administration of recombinant PTH analogs (eg, teriparatide) induces a greater increase in osteoblast activity in proportion to osteoclast activity and a net increase in bone formation.

A 60-year-old man comes to the office due to a 4-month history of increasing midthoracic back pain, which is aggravated by coughing. The patient has also had right thigh pain but no numbness or weakness in the legs and no bowel or bladder incontinence. He has no other medical conditions. Vital signs are within normal limits. On examination, there is tenderness over the eighth and ninth thoracic vertebrae. Neurologic examination is normal. The prostate is normal in size and has no palpable nodules. Imaging studies reveal enlarged vertebral bodies with cortical thickening. Serum testing for which of the following would most likely help establish a diagnosis in this patient? 25-hydroxyvitamin D level Alkaline phosphatase level Erythrocyte sedimentation rate Parathyroid hormone level Prostate-specific antigen level

This older patient with multifocal bone pain has enlarged vertebral bodies with cortical thickening on imaging, which is consistent with Paget disease of bone (PD). PD is characterized by excessive and disordered bone formation. It commonly affects the skull, long bones of the extremities, and vertebral column. In addition to pain and deformity, vertebral involvement can lead to spinal stenosis, nerve compression, and compression fracture. The increased formation of new bone in PD is associated with an elevated serum alkaline phosphatase level, but because the changes occur slowly, serum calcium and phosphorus levels are often normal. Radiographs typically reveal lytic or mixed lytic-sclerotic lesions, thickening of cortical and trabecular bone, and bony deformities. In the vertebrae, bony enlargement and cortical thickening may create an appearance resembling a picture frame. (Choice A) The level of 25-hydroxyvitamin D reflects total body vitamin D stores and is useful in the diagnosis of osteomalacia. In addition to bone pain, osteomalacia causes muscle weakness and difficulty walking. X-ray reveals diffuse demineralization, often with insufficiency fractures, rather than cortical thickening. (Choice C) The erythrocyte sedimentation rate reflects circulating levels of inflammatory proteins and is elevated in many inflammatory and malignant conditions. It is classically elevated in polymyalgia rheumatica; this condition causes pain in the shoulders, hips, and proximal extremities, but x-rays are normal. It can also be strikingly elevated in multiple myeloma, which can cause osteolytic lesions resembling early pagetic lesions but does not cause cortical thickening. (Choice D) Hyperparathyroidism can present with bone pain, typically in association with hypercalcemic manifestations (eg, constipation, renal stones). X-ray of the vertebral column reveals cortical thinning, not thickening. (Choice E) Prostate-specific antigen is used in the diagnosis of prostate cancer. Bone metastasis in prostate cancer appears on x-ray as focal sclerotic lesions, and most patients have palpable nodules on prostate examination. Educational objective: Paget disease of bone is characterized by excessive and disordered bone formation. It commonly affects the skull, long bones, and vertebral column. The increased formation of new bone is associated with an elevated serum alkaline phosphatase level. Radiographs shows lytic or mixed lytic-sclerotic lesions, thickening of cortical and trabecular bone, and bony deformities.

A 29-year-old woman comes to the office due to 2 weeks of severe, progressive joint pain in her hands, wrists, and ankles. The pain started in several fingers and spread over the course of a day to more joints. It is so severe that she is unable to perform daily activities, and ibuprofen provides only partial relief. The patient has no medical conditions but developed a high fever and diffuse rash on her trunk and extremities during a trip to Puerto Rico 2 weeks ago. Evaluation at a local medical facility was negative for malaria, and her fever and rash resolved spontaneously after a few days. She does not use tobacco, alcohol, or illicit drugs and is in a monogamous relationship with her husband. Temperature is 37 C (99 F), blood pressure is 122/70 mm Hg, and pulse is 86/min. On physical examination, the patient has pink, moist oropharyngeal mucosa; no rash; clear lungs; normal heart sounds; and no abdominal organomegaly. Periarticular swelling and tenderness are noted in the wrists, ankles, and interphalangeal joints of her hands. Range of motion in these joints is decreased due to pain. Laboratory testing reveals lymphopenia. Which of the following is the most likely cause of this patient's joint pain? Acute joint inflammation from crystal deposition Autoantibody-mediated inflammatory arthritis Spondyloarthritis triggered by bacterial infection Tick bite-borne spirochetal joint infection Viral infection transmitted by mosquito bite

This patient developed a transient febrile illness and rash while traveling in Puerto Rico and now has severe, persistent polyarthritis. The likely cause is chikungunya, an alphavirus infection transmitted by Aedes mosquitoes. Outbreaks are common in tropical and subtropical regions (eg, Caribbean, Africa, Asia, Indian and Pacific Oceanic islands) during the rainy seasons; travelers may bring the illness home. Manifestations generally begin a few days after transmission and include high fever; diffuse maculopapular rash; and severe, symmetric polyarticular arthralgia that begins in the small joints of the hands and feet and may move proximally. Transient lymphopenia and thrombocytopenia are common. Fever and rash usually abate within a few days, but many (>70%) patients have persistent arthralgia, polyarticular arthritis (particularly of the wrists, hands, or ankles), or tenosynovitis for weeks or months. Because Aedes mosquitoes also transmit the viruses that cause dengue, yellow fever, and Zika, coinfection can occur. (Choice A) Gout causes recurrent, acute joint inflammation due to monosodium urate crystal deposition. However, most cases are monoarticular and occur in lower extremity joints. (Choice B) Rheumatoid arthritis (RA) and systemic lupus erythematosus (SLE) are autoantibody disorders that cause symmetric, inflammatory polyarthritis. Patients generally develop morning stiffness followed by joint swelling in the fingers, wrists, and toes; this migrates in SLE and progresses to eventual joint destruction in RA. The presence of a febrile illness while traveling and persistent nonmigratory joint pain makes an infection like chikungunya more likely than an autoantibody-mediated inflammatory arthritis. (Choice C) Reactive arthritis is a rare condition that primarily follows a gastrointestinal (eg, Salmonella, Shigella, Campylobacter) or genital (eg, Chlamydia) bacterial infection. Manifestations typically include an asymmetric oligoarthritis that often affects the knees. This patient's febrile illness while traveling and symmetric polyarthritis make chikungunya more likely. (Choice D) Lyme disease is caused by the spirochete Borrelia burgdorferi. Initial symptoms usually include fever; malaise; and a spreading, annular rash (erythema migrans). Several months or years later, late manifestations such as monoarticular arthritis of the knee or asymmetric oligoarthritis can develop. Educational objective: Chikungunya is an alphavirus transmitted by mosquitoes in tropical and subtropical areas. Initial manifestations include fever, diffuse rash, and severe polyarthralgia. Although fever and rash quickly resolve, many patients have chronic arthralgia, polyarthritis (eg, wrist, fingers, ankles), or tenosynovitis.

A 61-year-old man comes to the office to follow up acute arthritis. He was seen at a nearby urgent care center 6 weeks ago for sudden-onset pain and swelling in the ankle. The patient was treated with an unknown analgesic with rapid relief of his pain. Since that time, his pain has resolved completely and he is now feeling well. Past medical history is notable for hypercholesterolemia, for which he takes atorvastatin. He also has a history of recurrent renal colic. On examination, his ankle appears normal with no redness, warmth, or swelling. Serum uric acid level is 9.8 mg/dL. Which of the following is the best agent for long-term management of this patient? Antibiotics Cyclooxygenase inhibitor Glucocorticoid Lipoxygenase antagonist Uricosuric agents Xanthine oxidase inhibitors

This patient had an episode of acute gouty arthritis that has responded to acute therapy. The decision now needs to be made whether to initiate long term preventive therapy. Patients who have only occasional episodes of gout may not need prophylactic treatment. However, uric acid-lowering medications can be helpful in patients with frequent episodes of gouty arthritis, chronic joint destruction, uric acid renal stones, or tophi. In addition to acute gout, this patient has had recurring episodes of likely uric acid stones and should be considered for prophylactic therapy. The preferred treatment to prevent recurrent attacks is xanthine oxidase inhibitors (eg, allopurinol, febuxostat), which decrease uric acid production. Prophylactic therapy is usually initiated during intercritical (between attacks) periods as acute changes in serum uric acid levels can worsen an acute attack. (Choice A) Septic arthritis can cause an acute monoarthritis but would be unlikely to respond to analgesics. (Choice B) Nonsteroidal antiinflammatory drugs (NSAIDs) act by inhibiting cyclooxygenase. NSAIDs are the first-line therapy for acute gouty arthritis and can prevent an acute flare while urate-lowering therapy is started. However, they are not normally used for long-term therapy due to risk of peptic ulcer. (Choice C) Oral or intraarticular glucocorticoids can be used for acute gout in patients who cannot tolerate NSAIDs or colchicine. However, they have significant long-term side effects. (Choice D) Lipoxygenase inhibitors (eg, zileuton) are oral anti-inflammatory medications used in the long-term management of asthma. They are not useful for treatment of gout. (Choice E) Uricosuric agents (eg, probenecid) are second-line drugs for chronic management of gout. These drugs increase renal excretion of uric acid and are therefore contraindicated in patients with a history of renal stones. Educational objective: Chronic uric acid-lowering therapy is recommended for patients with gout who have frequent gouty attacks, uric acid kidney stones, tophi, or chronic joint destruction from gout. Xanthine oxidase inhibitors are the preferred treatment.

A 68-year-old man comes to the office due to difficulty performing normal day-to-day activities with his hands. He has had pain in his finger joints, wrists, and knees for many years and has been taking over-the-counter analgesics. The patient has been evaluated by doctors for "joint problems" in the past but declined their treatment recommendations. He has smoked a pack of cigarettes daily for 30 years. Vital signs are normal. Examination findings of the patient's hands are shown in the image below. Swan Neck ID=15627 Which of the following pathogenic mechanisms is most likely responsible for this patient's condition? Calcific degeneration of the cartilage Cartilage thinning and subchondral sclerosis DNA/anti-DNA immune complex deposition Synovial pannus formation Uric acid crystal deposition

This patient has a chronic, symmetric, deforming arthritis consistent with advanced rheumatoid arthritis (RA). His examination shows classic features including joint enlargement, ulnar deviation at the metacarpophalangeal joints, and swan neck deformities (hyperextension at the proximal interphalangeal joints with flexion at the distal interphalangeal joints). The joint destruction in RA is thought to be initiated by CD4+ T helper cells and is characterized by synovial hyperplasia and an inflammatory infiltrates (often containing lymphoid follicles). The joint space often becomes replaced by a synovial pannus, an invasive mass composed of fibroblast-like synovial cells, granulation tissue, and inflammatory cells. Release of proteinases (eg, matrix metallopeptidase 13) causes destruction of the articular cartilaginous matrix, facilitating erosion of the surrounding articular cartilage and underlying bone. Ossification of the pannus can lead to fusion of the bones across the affected joint (bony ankylosis). (Choices A and E) Calcific degeneration of the articular cartilage (chondrocalcinosis) is a characteristic feature of calcium pyrophosphate dihydrate deposition disease (pseudogout). Deposition of uric acid crystals results in gout. Both gout and pseudogout are inflammatory arthritides that can cause pain and joint deformation but present with acute, episodic arthritis and are more common in the lower extremities. (Choice B) Osteoarthritis is characterized by progressive fracturing and erosion of the articular cartilage. Periarticular findings include osteophyte formation and subchondral sclerosis. When osteoarthritis affects the hands, it typically presents with bony enlargement at the proximal and distal interphalangeal joints (Bouchard and Heberden nodes, respectively) rather than ulnar deviation at the metacarpophalangeal joints. (Choice C) Autoimmune complex deposition (type III hypersensitivity) plays a significant role in the arthritis associated with systemic lupus erythematosus. Although lupus may cause a chronic destructive arthritis resembling RA in a minority of cases, lupus arthritis is typically migratory, nondeforming, and much more common in younger women than in older men. Educational objective: Joint destruction in rheumatoid arthritis is characterized by synovial hyperplasia, an inflammatory infiltrate, and synovial angiogenesis. The joint space often becomes replaced by pannus, an invasive mass composed of fibroblast-like synovial cells, granulation tissue, and inflammatory cells that can erode into the articular cartilage and underlying bone.

A 48-year-old woman comes to the office with a 4-month history of pain, stiffness, and swelling of her hands, feet, and knees that is associated with daily fatigue. She has taken over-the-counter nonsteroidal anti-inflammatory agents, but they only temporarily relieve the pain. The patient has no other medical issues. She does not use tobacco, alcohol, or illicit drugs and is not sexually active. Vital signs are within normal limits. BMI is 35 kg/m2. The patient walks with a limp due to pain in the foot. There is symmetric swelling, tenderness, and restricted range of motion involving the wrists, ankles, forefeet, and knees. There is no skin rash. Laboratory studies show anemia. Which of the following is most likely to be the earliest finding during the development of this patient's condition? Fibrillation of articular cartilage Joint subluxation Marginal bone erosions Osteophyte formation Synovial neovascularization

This patient has a chronic, symmetric, polyarticular arthritis with joint swelling and stiffness consistent with early rheumatoid arthritis (RA). RA is a progressive autoimmune disorder that is often associated with signs of systemic inflammation (eg, fatigue, fever, anemia). It has a peak incidence at age 50-75 but can occur at any age; women are affected more commonly than men. The pathogenesis of RA begins with activation of T lymphocytes in response to rheumatoid antigens (eg, citrullinated peptides, type II collagen). Activated T cells release cytokines that cause synovial hyperplasia with recruitment of additional mononuclear cells. The accelerated metabolic rate of the inflamed synovial tissue leads to local hypoxia and increased production of hypoxia-inducible factor 1 and vascular endothelial growth factor by local macrophages and fibroblasts, resulting in synovial angiogenesis (neovascularization). As the disease progresses, new blood vessels provide nutrients that facilitate expansion of inflammed synovium into a rheumatoid pannus, an invasive mass composed of fibroblast-like synovial cells, granulation tissue, and inflammatory cells. Over time, the pannus encroaches into the joint space and can destroy the articular cartilage and erode the underlying subchondral bone (Choice C). Ossification of the pannus can lead to fusion of the bones across the affected joint (bony ankylosis). (Choices A and D) Osteoarthritis is characterized by progressive fibrillation (ie, fissuring, fracturing) and erosion of articular cartilage due to increased biomechanical stress. Periarticular findings include osteophyte formation and subchondral sclerosis due to extensive bone remodelling. In contrast, RA is more frequently associated with periarticular bone erosions. (Choice B) Late-stage RA is characterized by widespread joint deformities. Characteristic findings include ulnar deviation at the metacarpophalangeal joints, swan-neck deformities (hyperextension at the proximal interphalangeal joints with flexion at the distal interphalangeal joints) in the digits, and additional deformities (eg, volar subluxation of the carpus, radial drift) at the wrists. Educational objective: Rheumatoid arthritis is characterized by synovial hyperplasia with inflammatory infiltrates. The accelerated metabolic rate of the inflamed synovium causes local hypoxia, which leads to synovial angiogenesis. As the disease progresses, the joint space is replaced by a rheumatoid pannus (an invasive mass of fibroblast-like synovial cells, granulation tissue, and inflammatory cells) which can destroy the articular cartilage and underlying subchondral bone.

A 70-year-old white woman is brought to the emergency department due to acute right hip pain after a fall. She was walking in her house when she tripped on a floor rug and fell, landing on her hip. The patient has been unable to bear weight on the right leg since the fall and had to call for emergency transport to the hospital. She has no significant medical history and does not use tobacco, alcohol, or illicit drugs. The patient's only medication is an over-the-counter multivitamin. Family history is notable for a hip fracture in her mother. Blood pressure is 150/90 mm Hg and pulse is 112/min. The patient is in moderate pain and her right leg appears shortened and externally rotated. There is tenderness and swelling of the right hip. Radiographs reveal a right femoral neck fracture. Which of the following changes in bone structure is most likely responsible for this patient's condition? Lamellar bone structure resembling a mosaic Osteoid matrix accumulation around trabeculae Persistence of primary spongiosa in the medullary canal Subperiosteal bone resorption and cystic degeneration Trabecular thinning with fewer interconnections

This patient has a fragility fracture (ie, due to low-intensity trauma that would not cause a fracture in normal bone). In light of her white ethnicity and post-menopausal age, this is likely due to osteoporosis. After menopause, declining estrogen levels accelerate the loss of bone mass with a decrease in osteoblastic and an increase in osteoclastic activity. Other common risk factors include low body weight, smoking, heavy alcohol intake, and sedentary lifestyle. The 2 major types of bone are trabecular (also called cancellous or spongy bone) and cortical. Trabecular bone composes only 15% of the skeleton by weight but is metabolically more active due to its large surface area. Initially following menopause, bone loss predominantly affects trabecular bone, especially in the dorsolumbar vertebral bodies. Morphologic characteristics include trabecular thinning and perforation with loss of interconnecting bridges. With continued aging, cortical bone, which composes most of the appendicular skeleton, also becomes involved. The neck of the femur has components of both trabecular and cortical bone, and is a common site of osteoporotic fracture. (Choice A) Paget disease of bone is caused by defective osteoid formation and increased bone remodeling. Collagen is laid down in a haphazard manner, resulting in a mosaic pattern of bone with irregular sections of lamellar bone linked by prominent cement lines. Paget disease typically presents with bone pain and deformity with osteolytic or mixed lytic/sclerotic lesions on x-ray. (Choice B) Osteomalacia is commonly due to vitamin D deficiency, and is characterized by unmineralized osteoid deposits on trabecular surfaces. The bone is weak and prone to fracture, but symptomatic patients typically have bone pain, muscle weakness, and impaired ambulation. (Choice C) Osteopetrosis is a group of disorders characterized by impaired osteoclast function. This leads to persistence of the primary spongiosa in the medullary cavity. These rare disorders typically present early in life with fractures, deformities, and hematologic cytopenias (due to obliteration of the marrow space). (Choice D) Bone disease in hyperparathyroidism is characterized by increased bone resorption in cortical bone with subperiosteal thinning and cystic degeneration (osteitis fibrosa cystica). However, this is typically seen in advanced cases along with diffuse bone pain and manifestations of hypercalcemia (eg, constipation, neuropsychiatric symptoms). Educational objective: Initially in osteoporosis, bone loss predominantly affects trabecular bone, leading to trabecular thinning and perforation with loss of interconnecting bridges. Over time, cortical bone, which composes most of the appendicular skeleton, also becomes involved.

A 48-year-old woman comes to the emergency department due to back pain after minor trauma. She has no leg weakness or numbness. Review of systems is positive for recent-onset hot flashes and irregular menses; her most recent menstruation was 2 weeks ago. The patient has a long history of hypothyroidism treated with levothyroxine and rheumatoid arthritis treated with methotrexate. She has also taken oral prednisone for frequent symptom flares. The patient does not use tobacco, alcohol, or illicit drugs, and she consumes a balanced diet. Physical examination shows point tenderness over the midthoracic spine. The straight-leg raising test is negative, and neurologic examination of the lower extremities shows no abnormalities. Spine imaging is shown below: ID=1773 Which of the following contributed most to this patient's current condition? Inadequate levothyroxine replacement Medication adverse effect Nutritional vitamin deficiency Ovarian hormone deficiency Vertebral joint inflammation

This patient has a fragility fracture of a thoracic vertebra (ie, fracture due to a force significantly less than that required to fracture a normal bone). A fragility fracture suggests underlying bone pathology, often due to metastatic malignancy or intrinsic bone disease. In this patient, who has had frequent exposure to systemic glucocorticoids (eg, prednisone), this likely represents osteoporosis. Chronic or recurrent glucocorticoid use, as is occasionally needed for patients with rheumatoid arthritis, is associated with an increased risk for osteoporosis. Osteoporosis can also occur due to systemic absorption of topical glucocorticoids (eg, inhaled glucocorticoids used in the treatment of asthma). Glucocorticoids promote osteoporosis by inhibiting proliferation and differentiation of osteoblast precursor cells, promoting osteoclast differentiation and activity, and suppressing intestinal calcium absorption and renal calcium reabsorption. (Choice A) Thyroid hormone stimulates osteoclast differentiation and activity, bone resorption, and release of calcium into circulation. Thyrotoxic states (eg, excessive levothyroxine dose, endogenous hyperthyroidism) cause increased bone turnover with net bone loss. However, inadequate levothyroxine dosing is not a significant contributor to bone loss. (Choice C) Deficiency of vitamin D can lead to osteoporosis. Risk factors include malabsorption syndromes (eg, celiac disease) and lack of sun exposure, but nutritional deficiency is not a common cause of vitamin D deficiency. Glucocorticoid use is likely a more significant factor in this patient. (Choice D) Loss of ovarian estrogen production after menopause leads to an increased risk of osteoporosis. However, postmenopausal osteoporosis develops slowly, and fracture is uncommon until at least 10-15 years after cessation of menses; this patient is still perimenopausal, and loss of estrogen is unlikely to have caused her osteoporosis. (Choice E) Involvement of the cervical spine is common in rheumatoid arthritis and, over time, can lead to atlantoaxial instability, subluxation with possible cord compression, and odontoid fracture. However, involvement of the thoracic and lumbar spine is uncommon. Educational objective: Osteoporosis is a common cause of fragility fractures, which occur in the absence of significant trauma. Chronic or recurrent use of glucocorticoids (eg, prednisone) promotes osteoporosis and increases the risk of fractures.

A 24-year-old man comes to the emergency department with leg swelling. His dog, which is fully vaccinated and has lived with him for the past year, bit him on the left leg last night, leading to a break in the skin and bleeding. The patient irrigated the wound with bottled water and soap and managed his pain with acetaminophen. This morning, the wound area became warm, red, and swollen. On examination, temperature is 36.4 C (97.5 F). There is an open wound on the left leg, with surrounding erythema, warmth, and tenderness. No wound drainage or crepitus is present. The distal pulses are palpable. Wound cultures grow gram-negative coccobacilli; the culture has a mouse-like odor. Which of the following organisms is most likely responsible for this patient's current presentation? Bartonella henselae Campylobacter jejuni Clostridium perfringens Coxiella burnetii Erysipelothrix rhusiopathiae Francisella tularensis Fusobacterium Pasteurella multocida Proteus mirabilis

This patient has a skin and soft-tissue infection (SSTI) that developed at the site of a dog bite relatively soon following the bite. Pasteurella multocida is an organism found in the mouths of dogs and is responsible for the majority of acute skin infections following a dog bite. Infection typically occurs within 24 hours of inoculation and has a characteristic mouse-like odor (indole-positive species). P multocida SSTI can also occur following a cat bite. Other organisms associated with dog bites include staphylococci, streptococci, and Capnocytophaga canimorsus. Management includes wound care and antibiotics (eg, amoxicillin-clavulanate). (Choice A) Bartonella henselae is associated with cat-scratch disease, which can develop following inoculation from an infected cat and presents with lymphadenopathy that is often self-limiting. It is not associated with dog bites. (Choices B and I) Campylobacter jejuni typically causes gastrointestinal (diarrheal) illness, and Proteus mirabilis is a common cause of urinary tract infection. (Choices C and E) Clostridium perfringens is an anaerobe that can cause necrotizing SSTI, which occurs most commonly after a traumatic wound and leads to myonecrosis and gas gangrene. Erysipelothrix rhusiopathiae is a gram-positive rod (not a gram-negative coccobacillus) that causes erysipeloid. (Choices D and G) Coxiella burnetii can cause Q fever, a mild form of pneumonia. Fusobacterium is part of the anaerobic oral flora potentially causing aspiration pneumonia or pharyngitis (Lemierre disease). (Choice F) Francisella tularensis causes tularemia, a zoonotic infection that can occur following contact with lagomorphs and rodents (eg, rabbits, beavers, squirrels). The presentation is variable (eg, severe febrile pulmonary infection, ulcerative disease at inoculation site). Educational objective: Pasteurella multocida is a cause of soft-tissue infection that develops within 24 hours following a dog or cat bite. Management includes wound care and antibiotics targeted against this organism.

A 27-year-old man comes to the emergency department due to severe left knee pain and swelling for the past 2 days. He has no recent injury or significant medical problems. The patient drinks 3 or 4 cans of beer daily but does not use tobacco or illicit drugs. He works as a groundskeeper at a nearby golf course and has had to take the last couple of days off because of the pain. Temperature is 38.3 C (101 F), blood pressure is 115/70 mm Hg, and pulse is 92/min. On examination, the left knee is swollen, erythematous, warm, and tender with restricted range of active and passive motion. The remainder of the examination is unremarkable. Which of the following is the best next step in management of this patient? Ibuprofen and follow up in 3 days MRI of the knee joint Radionuclide bone scan Serum uric acid level Synovial fluid analysis

This patient has acute joint pain, swelling, and erythema with restricted range of motion, consistent with synovitis. Acute undiagnosed synovitis, especially when accompanied by fever or leukocytosis, represents a true emergency and warrants an expedited evaluation. Potential causes include septic arthritis (gonococcal, nongonococcal), crystal arthropathy (eg, gout), hemarthrosis, or rheumatic disease. Delayed diagnosis of septic arthritis may lead to loss of the joint and long-term disability, and may be fatal (Choice A). Acute synovitis is best evaluated with diagnostic arthrocentesis and synovial fluid analysis. Gross inspection may assist the diagnosis, with purulent or cloudy fluid suggesting an infectious or inflammatory process. Fluid should be sent for crystal analysis, cell count, Gram stain, and culture. Blood cultures should also be drawn if septic arthritis is suspected. (Choice B) MRI is useful to evaluate menisci, ligaments, and other soft tissues around the joint. However, MRI can be time-consuming and would not provide a definitive diagnosis of septic arthritis. (Choice C) Radionuclide bone scans are useful in the evaluation of metastatic and infectious bone disorders. However, the findings are sensitive but very nonspecific for potential causes of synovitis. (Choice D) Synovial fluid crystal analysis is the definitive test for gout. Serum uric acid levels are less useful for diagnosis and may be normal during an acute gout attack. In addition, serum uric acid levels do not rule out other important causes of synovitis such as septic arthritis. Educational objective: Synovitis is characterized by pain, erythema, swelling, and reduced range of motion in a joint. Acute synovitis may represent serious pathology (eg, septic arthritis), especially if accompanied by fever or leukocytosis; it should be evaluated urgently with synovial fluid analysis.

A 36-year-old man comes to the emergency department due to severe, acute knee pain that awoke him from sleep. He has a history of end-stage renal disease for which he underwent a renal transplant 8 weeks ago. The patient takes an immunosuppressive regimen that includes cyclosporine and low-dose prednisone. He visited his nephrologist last week and serum creatinine levels were normal with no proteinuria. Temperature is 37.5 C (99.5 F), blood pressure is 136/78 mm Hg, and pulse is 88/min. Examination shows redness, warmth, and a small effusion in the left knee. Passive range of motion at the knee causes severe pain. Microscopic examination of joint fluid from the knee shows numerous neutrophils as well as both intra- and extracellular crystals as shown in the exhibit. Which of the following mechanisms is most likely responsible for this patient's acute symptoms? Decreased urinary excretion of uric acid Increased production of uric acid Rapid drop in blood uric acid levels Release of nucleic acids from apoptotic cells Volume depletion and decreased renal perfusion

This patient has acute monoarticular arthritis of the knee with synovial fluid evidence of inflammation and needle-shaped (monosodium urate) crystals, indicating a diagnosis of gout. Gout develops in patients with hyperuricemia and is associated with several risk factors including excessive alcohol intake, meat/seafood consumption, chronic kidney disease, and certain medications. Cyclosporine is a common cause of gout exacerbations in patients who have undergone solid organ transplantation, particularly renal transplantation. This medication impairs renal excretion of uric acid, resulting in hyperuricemia. Although high doses of systemic glucocorticoids (eg, prednisone) can be used to treat acute gout, the low doses used for immunosuppression in patients who have undergone transplantation are usually not sufficient to prevent gout flares. (Choice B) Increased production of uric acid is commonly due to a purine-rich diet (eg, red meat, seafood), excessive alcohol intake, fructose-sweetened beverage intake, or disorders associated with increased cell turnover (eg, myeloproliferative disorders, cancers). (Choice C) Urate-lowering agents (eg, allopurinol) are used to prevent recurrent gout but can occasionally precipitate a gout flare due to rapid declines in blood uric acid levels, which can disrupt preexisting crystal deposits and increase their immunogenicity. This patient is not taking a medication that lowers uric acid levels. (Choice D) Patients who have malignancy associated with high tumor burdens (eg, acute lymphoblastic leukemia) can develop tumor lysis syndrome when cytotoxic chemotherapy is initiated. The massive release of nucleic acids into the circulation (from lysed cells) increases serum uric acid levels (due to catabolism of purine nucleic acids) and can precipitate gout. (Choice E) Diuretics (eg, furosemide, hydrochlorothiazide) can be associated with gout as these medications reduce urate excretion in the proximal tubule and also cause volume depletion leading to decreased renal perfusion and filtration of uric acid. This patient has a normal serum creatinine level, indicating adequate renal function. Educational objective: Patients undergoing solid organ transplantation are at increased risk of gout due to medications that impair renal clearance of uric acid. Cyclosporine is particularly associated with gout in the post-transplantation period due to decreased uric acid excretion.

A 28-year-old man comes to the emergency department due to 12 hours of severe right knee pain and swelling. He has had no trauma. The patient drinks 1 or 2 cans of beer on weekends but does not use tobacco or illicit drugs. On physical examination, right knee effusion, erythema, and tenderness are present. Active and passive range of motion is markedly decreased. Other joints are normal. Arthrocentesis is performed, and synovial fluid analysis shows a white blood cell count of 110,000/mm3. Which of the following would provide the best treatment for this patient's condition? Allopurionl Antibiotics Colchicine Indomethacin Prednisone

This patient has acute monoarticular arthritis with an effusion, decreased range of motion of the affected joint, and a synovial fluid leukocyte count >100,000/mm3. Septic arthritis (very often due to gonococcus in this patient's age group) is a likely diagnosis, and he should receive antibiotics given the risk of joint destruction, osteomyelitis, and sepsis. Synovial fluid Gram stain and culture can help confirm the diagnosis (although cultures can sometimes be negative with gonococcus). Microscopy can exclude the presence of crystals suggestive of gout and pseudogout, conditions that can also cause acute monoarticular arthritis (gout is associated with alcohol use). However, they typically present at an older age, and the synovial fluid leukocyte count is usually lower (at the lower end of the 20,000-100,000/mm3 range). (Choice A) Allopurinol is a hypoxanthine isomer that lowers serum uric acid levels. It is prescribed to prevent attacks of acute gouty arthritis but is not useful in the treatment of acute flares. (Choices C, D, and E) Colchicine reduces the acute inflammation of gouty arthritis by inhibiting neutrophil migration into the inflamed areas. Indomethacin, a nonsteroidal anti-inflammatory drug (NSAID), may be prescribed for pain relief in noninfectious inflammatory arthritis (eg, gout) but would not treat infection. Prednisone is sometimes used in the treatment of gout when there are contraindications to colchicine and NSAIDs. Educational objective: A high synovial fluid leukocyte count (>100,000/mm3) and absent crystals on microscopic examination strongly suggest bacterial joint infection. Septic arthritis requires antibiotic treatment to prevent joint destruction, osteomyelitis, and sepsis.

An 8-year-old boy is brought to the office for rapid and irregular movements of his hands for one week. His parents say that he is also making unintentional "funny faces" and has trouble controlling the volume of his voice. His temperature is 38.9 C (102 F). On physical examination, the boy moves his hands frequently and erratically. He has a new III/VI systolic murmur and several circular, faintly erythematous lesions on his abdomen. Which of the following is the most likely mechanism for this patient's condition? Nonspecific T cell receptor activation Cross-reactivity of antibodies against bacterial and host antigens Embolization of an infected thrombus Injury from immune complex deposition Release of an erythrogenic toxin

This patient has acute rheumatic fever, an immune-mediated disease following an untreated group A streptococcal (GAS) infection. Antibodies against GAS cross-react with host tissues due to molecular mimicry between GAS antigens and cardiac and central nervous system antigens. Specifically, antibodies directed against GAS antigens, M protein and N-acetyl-beta-D-glucosamine, subsequently attack myosin, a cardiac protein, and lysoganglioside, a neuronal cell surface protein. The major manifestations of acute rheumatic fever include arthritis, pancarditis, Sydenham chorea, erythema marginatum, and subcutaneous nodules. This patient's murmur is likely due to acute mitral regurgitation from pancarditis. Sydenham chorea presents with non-rhythmic movements of the hands, feet, and face. Patients often have sudden changes in voice pitch and volume. Erythema marginatum presents as faintly erythematous, circular lesions with central clearing that come and go on the trunk and extremities. (Choice A) Superantigens cause a tremendous release of cytokines through nonspecific T cell receptor activation, leading to acute fever, hypotension, and erythroderma. This is the mechanism of action of the toxic shock syndrome exotoxins produced by both Staphylococcus aureus and Streptococcus pyogenes. (Choice C) An embolic stroke from infective endocarditis can present with fever, a new-onset murmur, and focal neurologic deficits. In addition, Janeway lesions, Osler nodes, and Roth spots are specific findings of infectious endocarditis. The most common pathogens are S aureus, viridans group streptococci, and enterococci. (Choice D) The deposition of streptococcal antigen immune complexes in glomeruli causes post-streptococcal glomerulonephritis, a type III hypersensitivity reaction. Patients have microscopic or gross hematuria, edema, hypertension, and proteinuria. (Choice E) Scarlet fever is caused by the body's response to an erythrogenic (pyrogenic) toxin released by group A Streptococcus. It presents with a diffuse, erythematous, "sandpaper"-textured rash most notable in the skin folds (eg, inguinal, axillary, antecubital areas). Scarlet fever can occur with pharyngitis and can lead to acute rheumatic fever if untreated. Educational objective: Acute rheumatic fever is an autoimmune reaction following an untreated group A streptococcal pharyngitis. Anti-group A Streptococcus antibodies (eg, anti-M protein, anti-N-acetyl-beta-D-glucosamine) cross-react and attack cardiac and central nervous system antigens.

A 46-year-old woman comes to the clinic to establish primary care as a new patient. She is a marathon runner but has been unable to train due to a vertebral stress fracture diagnosed via x-ray during a recent emergency department visit. She has a history of hypothyroidism for which she takes levothyroxine. The patient's last menstrual period was a year ago. Her diet consists mainly of vegetables and fruit, and she takes a daily multivitamin. Her mother died of breast cancer at age 52. The patient is concerned that her family history puts her at risk for breast cancer. X-ray absorptiometry studies demonstrate abnormally low bone density in the lumbar vertebrae. Which of the following drugs is the most appropriate option for decreasing the risk of both bone fractures and breast cancer in this patient? Alendronate Ethinyl estradiol Leuprolide Medroxyprogesterone Raloxifene Tamoxifen

This patient has amenorrhea and low bone density, suggesting a low-estrogen state. This may be due to menopause, although her exercise history suggests possible hypothalamic amenorrhea (excessive exercise suppresses secretion of GnRH, leading to decreased release of FSH and low estrogen levels). Because estrogen decreases bone resorption, estrogen deficiency in women increases the risk for osteoporosis. Although estrogen replacement therapy (eg, ethinyl estradiol) can mitigate this risk, it can increase the risk of breast cancer and should be avoided in patients with a family history of breast cancer (Choice B). In addition, unopposed estrogen (ie, without concurrent progesterone) can cause endometrial proliferation resulting in endometrial hyperplasia and cancer. Selective estrogen receptor modulators (eg, raloxifene, tamoxifen) are nonsteroidal compounds that bind estrogen receptors and exhibit antagonist and agonist properties in a tissue-specific manner. Raloxifene has estrogen agonist activity on bone, which decreases bone resorption, improves bone density, and decreases the risk of vertebral fractures. In addition, raloxifene has an estrogen antagonist effect on breast tissue and can decrease the risk of breast cancer. It also acts as an estrogen antagonist in the uterus and does not increase the risk of endometrial cancer. Tamoxifen is a selective estrogen receptor modulator that has strong estrogen antagonist activity in the breast and is used in the treatment of estrogen receptor-positive breast cancer. It has estrogen-like effects on bone and can reduce the risk of osteoporosis in postmenopausal women. However, its agonist activity on the uterus increases the risk of endometrial hyperplasia/cancer and it is not appropriate for routine use in osteoporosis (Choice F). (Choice A) Bisphosphonates (eg, alendronate) inhibit osteoclast-mediated bone resorption. They have no estrogen agonist or antagonist activity and do not lower the risk of breast cancer. (Choice C) Leuprolide is a GnRH analog that stimulates FSH and LH release (and subsequent estrogen production) when administered in a pulsatile manner (eg, via a programmable pump for treatment of anovulation) but suppresses release when administered continuously (eg, for endometriosis). Continuous use may reduce breast cancer risk but is associated with accelerated bone loss. (Choice D) Medroxyprogesterone reduces the incidence of endometrial hyperplasia and endometrial carcinoma in postmenopausal women on estrogen replacement therapy, but otherwise has no benefit in management of osteoporosis. In addition, long-term use of medroxyprogesterone alone is associated with increased bone loss. Educational objective: Selective estrogen receptor modulators exhibit estrogen antagonist and agonist properties in a tissue-specific manner. Raloxifene has estrogen agonist activity on bone, which decreases bone resorption and improves bone density. Raloxifene has an estrogen antagonist effect on breast tissue and can decrease the risk of breast cancer; it also acts as an estrogen antagonist in the uterus, and does not increase the risk of endometrial cancer.

A 52-year-old woman comes to the office due to pain in the right knee for the past several months. The pain is typically better in the morning but worsens by the end of the day. There is no history of trauma to the joint. Medical history is significant for hypertension, type 2 diabetes mellitus, and hypercholesterolemia. The patient is a lifetime nonsmoker. She works as an office clerk. Family history is significant for stroke in her mother and gout in her father. Temperature is 37.2 C (99 F), blood pressure is 145/95 mm Hg, and pulse is 90/min. BMI is 37 kg/m2. Physical examination shows crepitation on flexion and extension of the right knee; range of motion is otherwise normal, and there is no swelling, warmth, or redness. Which of the following pathologic processes is most likely occurring in this patient's knee joint? Calcium pyrophosphate crystal deposition Fissuring and flaking of articular cartilage Iron deposition in cartilage Synovial cell hyperplasia Virus-induced synovial inflammation

This patient has chronic knee pain associated with crepitus,which is consistent with osteoarthritis (the most common form of arthritis). Besides the knees, osteoarthritis also commonly affects the hips, lumbar spine, and distal joints of the hands. Patients typically have pain that worsens with activity and is relieved by rest. Osteoarthritis typically progresses with age and is accelerated in patients with prior joint injury or excessive mechanical stresses (eg, obesity, joint deformities). Osteoarthritis is characterized by progressive fissuring, flaking, and erosion (fibrillation) of articular cartilage. The etiology is multifactorial, with excessive biomechanical stress and increased intraarticular metalloproteinase activity as the major contributors to cartilage destruction. Although a number of proinflammatory mediators (eg, IL-6, macrophage chemotactic protein-1) have been linked to osteoarthritis, overt signs of synovitis (eg, redness, warmth) are less prominent than in the classic inflammatory arthritic disorders (eg, gout). Effusions, if present, are often small. (Choice A) Calcium pyrophosphate deposition disease presents with an acute inflammatory arthritis resembling gout (ie, pseudogout). Although this disorder is common in the knee, chronic arthritis without signs of acute synovitis is more consistent with osteoarthritis. (Choice C) Hemochromatosis can cause deposition of iron salts within the synovium and joint space, which can lead to a chronic arthritis resembling osteoarthritis. Although hereditary hemochromatosis is associated with diabetes mellitus, this patient lacks other characteristic findings (eg, skin pigmentation, liver dysfunction, cardiac enlargement); in addition, osteoarthritis is a much more common cause of arthritis in the general population. (Choice D) Rheumatoid arthritis is an inflammatory arthritis associated with synovial hyperplasia. It most commonly affects the proximal interphalangeal (PIP), metacarpophalangeal (MCP), wrist, knee, and ankle joints. However, patients typically have symmetric joint involvement and prolonged morning stiffness. (Choice E) Parvovirus B19 presents in adults as a nonspecific flulike illness, often followed by a symmetric arthritis most commonly affecting the hands. Symptoms are acute and resolve within a few weeks. Educational objective: Osteoarthritis is characterized by progressive fissuring and erosion of articular cartilage. Risk factors include advancing age, obesity, joint trauma, and repetitive stress. Patients may have mild effusion and crepitus on physical examination, but signs of synovitis (eg, redness, warmth) are less prominent than in the classic inflammatory arthritic disorders

A 61-year-old woman who recently immigrated to the United States has a long history of "joint problems." She began to have pain in her hands at age 30, and her symptoms have been intermittent but progressive since then. The symptoms were treated with nonsteroidal anti-inflammatory drugs when the pain was most severe. The patient also has a history of hypertension and chronic anemia. She does not smoke cigarettes or use illicit drugs. Family history is insignificant. On examination, there is gross deformity of multiple joints in both hands. X-ray findings are shown in the image below. ID=11821 The disease process responsible for the radiographic findings would most likely affect which of the following? Cervical spine Lumbar spine Sacroiliac joints Thoracic spine

This patient has chronic polyarticular arthritis, primarily involving the small joints of the hands. Her arthritis is progressive, leading to joint destruction and ulnar deviation of the digits, consistent with rheumatoid arthritis (RA). The symptoms of RA may be relieved temporarily with nonsteroidal anti-inflammatory drugs or systemic glucocorticoids, but long-term use of disease-modifying antirheumatic drugs is recommended to avoid progressive joint destruction. In addition to the metacarpophalangeal and proximal interphalangeal joints in the hands (and metatarsophalangeal joints in the feet), RA may also involve the wrists, elbows, knees, and ankles. Cervical spine involvement is also common in longstanding disease, and may lead to severe pain and disability due to spinal instability with potential radiculopathy/cord compression. The hips and lumbosacral joints are usually spared in RA. (Choice B) The lumbar spine is only rarely involved in RA but is commonly affected by osteoarthritis. (Choice C) Arthritis of the sacroiliac joints is characteristic of the seronegative spondyloarthropathies (ankylosing spondylitis, reactive arthritis, arthritis associated with inflammatory bowel disease, psoriatic arthritis). These conditions are seen most commonly in patients who carry the human leukocyte antigen B27 allele. (Choice D) Arthritis involving the thoracic spine is uncommon and is usually due to osteoarthritis or spondyloarthritis. Educational objective: Rheumatoid arthritis causes progressive joint destruction involving the hands, wrists, elbows, and knees. Cervical spine involvement can lead to spinal instability and cord compression.

A 62-year-old woman comes to the office due to a 3-month history of painful, swollen wrists and knees. She also has joint stiffness, which is worse upon wakening and limits her daily activities. Review of systems is positive for fatigue. The patient's only other medical condition is hypothyroidism for which she takes levothyroxine. She smoked a pack of cigarettes daily for 20 years and quit 15 years ago. Vitals signs are within normal limits. Physical examination shows symmetric, moderate swelling of the wrists and knees. The joints are tender and warm. Range of motion is intact but painful. Plain radiographs of the symptomatic joints show joint space narrowing and marginal erosions. Which of the following cytokines are primarily involved in the pathogenesis of this patient's joint destruction? IL-1 and tumor necrosis factor-alpha IL-2 and interferon gamma IL-4 and IL-5 IL-10 and transforming growth factor-beta IL-12 and IL-23

This patient has chronic, symmetric, inflammatory arthritis associated with significant morning stiffness and overt synovitis (swelling, warmth, tenderness). In association with the radiographic findings of joint space narrowing and marginal joint erosions, this presentation is typical for rheumatoid arthritis (RA). RA is a progressive autoimmune disorder that has a peak incidence at age 50-75 but can occur at any age; women are affected more often than men. Systemic symptoms (eg, fatigue, fever) are common. The pathogenesis of RA involves both humoral (eg, autoantibodies against citrullinated polypeptides) and cell-mediated immunity; activation of CD4+ T cells, especially Th1 and Th17, occurs early in the disease process. Macrophages release proinflammatory cytokines critical for the development and progressive articular destruction seen in RA. These include: Tumor necrosis factor-alpha (TNF-alpha) stimulates the proliferation of inflammatory cells and causes expression of inflammatory factors (eg, collagenase, prostaglandins) by synovial cells. IL-1 induces synthesis of matrix metalloproteinases and enhances T-cell immune responses. The proteases (eg, collagenase, metalloproteinase) contribute to cartilage destruction. In addition, both cytokines indirectly activate osteoclasts, resulting in bony erosions. Monoclonal antibodies that inhibit TNF-alpha (eg, adalimumab, etanercept) or IL-1 receptors (eg, anakinra) are widely used in the treatment of RA and can slow progression of the disease. (Choice B) Activated Th1 cells produce significant quantities of IL-2 and interferon gamma, among other cytokines. Although Th1 cells and interferon gamma play a role in the pathogenesis of RA, IL-2 is less prominent. These cytokines are present in higher quantities in granulomatous diseases (eg, tuberculosis, sarcoidosis). (Choice C) IL-4 and IL-5 are involved prominently in mast cell function and play a role in the pathogenesis of atopic disorders (eg, asthma). (Choice D) Transforming growth factor-beta and IL-10 are antiinflammatory cytokines that downregulate lymphocyte activation and proliferation and reduce the production of proinflammatory cytokines (eg, TNF-alpha). These cytokines are likely protective of the joint destruction seen in RA. (Choice E) IL-12 and IL-23 are produced by activated T cells and play a prominent role in the pathogenesis of psoriasis. Educational objective: The pathogenesis of rheumatoid arthritis involves early activation of CD4+ T cells (especially Th1 and Th17 subsets) with release of cytokines such as tumor necrosis factor-alpha and IL-1 that cause destruction of cartilage and bone. Monoclonal antibodies that inhibit tumor necrosis factor-alpha or IL-1 receptors can slow progression of the disease.

A 68-year-old woman comes to the office due to a burning sensation in her chest and throat for the past 2 weeks. Associated symptoms include trouble swallowing. Medical history is significant for osteoporosis, and she has no known drug allergies. The patient has smoked half a pack of cigarettes daily for 50 years, and does not use alcohol or illicit drugs. Temperature is 36.7 C (98 F), blood pressure is 110/70 mm Hg, and pulse is 70/min. BMI is 20 kg/m2. Cardiopulmonary examination shows clear lungs and normal S1 and S2. The abdomen is soft and nontender. Laboratory results are normal. It is determined that the patient's current symptoms are caused by one of her medications, which is discontinued. Her symptoms subsequently resolve. The medication responsible for this patient's presentation is also associated with which of the following side effects? Chronic kidney disease Hypercalcemia Osteonecrosis of the jaw Venous thromboembolism Vitamin B12 deficiency

This patient has esophagitis, with burning pain in the chest and dysphagia. Medication-induced esophagitis is a common adverse effect of bisphosphonates (eg, alendronate, risedronate) thought to be caused by disruption of the protective phospholipid barrier in the lower esophagus. This allows refluxing gastric acid to cause mucosal erosion and ulceration. Bisphosphonates are contraindicated in conditions that impair esophageal motility (eg, stricture, achalasia). When bisphosphonates are used, the risk of esophagitis can be lessened by taking the medication with a full glass of water (to ensure the pill is delivered fully into the stomach) and remaining upright for 30 minutes following administration (to avoid reflux of gastric contents). Bisphosphonates are also associated with osteonecrosis of the jaw (mandible or maxilla) and atypical bone fractures (eg, stress fractures of the subtrochanteric zone and femoral shaft). The precise pathophysiology is debated and may vary, but proposed mechanisms include suppression of bone remodeling, impaired healing of microfractures, and decreased angiogenesis. (Choice A) Medication-induced nephropathy can be due to interstitial nephritis (eg, cyclosporine), crystal nephropathy (eg, acyclovir), renal vasoconstriction (eg, amphotericin B, nonsteroidal anti-inflammatory drugs), or tubular injury (eg, aminoglycosides). Bisphosphonates are eliminated by the kidney and should be used with caution in patients with chronic kidney disease, but do not have substantial nephrotoxicity. (Choice B) Bisphosphonates decrease bone resorption, thus lowering serum calcium levels. They are sometimes used in the treatment of severe hypercalcemia (eg, hypercalcemia of malignancy). (Choice D) Venous thromboembolism is a recognized complication of selective estrogen receptor modulators (eg, raloxifene), but these medications do not often cause esophagitis. Bisphosphonates do not appreciably increase the risk of thromboembolism. (Choice E) Vitamin B12 deficiency can be seen in long-term proton pump inhibitor (eg, omeprazole) therapy, possibly due to decreased acid-dependent cleavage of vitamin B12 from dietary proteins. Although proton pump inhibitors may be taken by patients with bisphosphonate-induced esophagitis, bisphosphonates do not affect vitamin B12 metabolism. Educational objective: Medication-induced esophagitis is a common adverse effect of bisphosphonates. Bisphosphonates are also associated with increased risk of osteonecrosis of the jaw and atypical femoral fractures.

A 67-year-old man comes to the office with right tibial pain that started 3 months ago and has increased in intensity over time. He also has had progressive hearing impairment for the last year. Physical examination reveals local tenderness and a lumpy protuberance over the right tibia. After extensive evaluation, the patient undergoes a bone biopsy. The pathologist identifies numerous multinucleated cells, some containing over 100 nuclei. Which of the following factors is essential for the differentiation of the cells described by the pathologist? Fibroblast growth factor Insulin-like growth factors Osteoprotegerin Receptor activator of nuclear factor kappa Transforming growth factor beta

This patient has pain and deformity of the long bones with hearing loss (due to bony deformity of the skull); these are typical features of Paget's disease of bone. The initial abnormality in Paget's disease is excessive osteoclastic bone resorption followed by increased bone formation by osteoblasts; this results in high bone turnover in the affected areas. The new bone formation is disorganized, and pagetic bone lesions typically appear on imaging as thickened areas of mixed sclerosis and lucency. Osteoblasts are cells with a single nucleus that arise from mesenchymal stem cells found in the periosteum and bone marrow. In contrast, osteoclasts originate from the mononuclear phagocytic cell lineage and are ultimately formed when several precursor cells fuse to create a multinucleated mature cell. Osteoclasts in Paget's disease are typically very large and can have up to 100 nuclei (normal osteoclasts have 2-5). The 2 most important factors for osteoclastic differentiation, macrophage colony-stimulating factor (M-CSF) and receptor for activated nuclear factor kappa-B ligand (RANK-L), are produced by osteoblasts and bone marrow stromal cells. Osteoprotegerin (OPG) is a physiologic decoy receptor that decreases binding of RANK-L to RANK. Inhibition of RANK-L to RANK receptor interaction reduces the differentiation and survival of osteoclasts, resulting in decreased bone resorption and increased bone density (Choice C). OPG loss-of-function mutations cause juvenile Paget's disease. A monoclonal antibody (denosumab) that inhibits the RANK/RANK-L interaction also leads to increased bone density and is commonly used for the treatment of osteoporosis. (Choice A) Fibroblast growth factors (FGFs) regulate chondrogenesis and osteogenesis. FGFs induce proliferation of osteoblastic precursor cells and anabolic function of mature osteoblasts. Abnormalities in the FGF receptor result in the congenital short-limbed dwarfism known as achondroplasia. (Choice B) Insulin-like growth factors (IGF-I and IGF-II) are synthesized by various tissues, including the liver and bone. IGF-I increases osteoblastic replication and collagen synthesis; it also decreases collagen degradation by inhibiting the enzyme matrix metalloproteinase-13 (MMP-13). The net effect of IGF-I on the bone is anabolic. (Choice E) Transforming growth factor beta increases the replication of osteoblast precursors, leading to increased formation of mature osteoblasts. Transforming growth factor beta also increases collagen synthesis and decreases bone resorption by increasing osteoclastic apoptosis. Educational objective: Osteoclasts originate from hematopoietic progenitor cells. Macrophage colony-stimulating factor and receptor for activated nuclear factor kappa-B ligand (RANK-L) play an important role in osteoclast differentiation. Paget's disease of bone is characterized by increased numbers of abnormal osteoclasts, excessive bone turnover and disorganized bone remodeling.

A 24-year-old woman comes to the emergency department due to joint pain. A week ago, the patient had a flu-like illness with fever, malaise, and fleeting joint aches in her wrists, ankles, and knees. Over the past 2 days, the joint pain worsened, and she developed new swelling of the right knee. The patient has no chronic medical problems and has had no similar symptoms in the past. She does not use tobacco, alcohol, or injection drugs. She is sexually active, uses an oral contraceptive for birth control, and her last menstrual period was a week ago. Temperature is 38 C (100.4 F). On examination, there is no scleral icterus, facial rash, or oral ulcers but a few scattered painless vesiculopustular lesions are present on the upper extremities. The right knee is warm, swollen, and tender with decreased range of motion. Both ankles and the left wrist are mildly tender to palpation but not swollen or erythematous. Arthrocentesis of the right knee joint yields 20 mL of cloudy fluid with a leukocyte count of 50,000/mm3 (90% neutrophils). Additional evaluation of the joint fluid is most likely to show which of the following? Gram-negative intracellular diploccoci Gram-negative oxidase-negative rods Gram-positive alpha-hemolytic cocci in chains Gram-positive catalase-positive cocci in clusters Needle-shaped negative birefringent crystals Rhomboid-shaped positively birefringent crystals

This patient has purulent arthritis of the knee, oligoarticular joint pain, and a few vesiculopustular lesions on the extremities, raising suspicion for disseminated gonococcal infection (DGI). DGI is one of the most common causes of septic arthritis in young, sexually active individuals. It is due to the spread of Neisseria gonorrhoeae from a (usually asymptomatic) genitourinary infection into the systemic circulation. Patients typically present with either purulent arthritis or the triad of polyarthralgia, dermatitis, and tenosynovitis; however, some overlap in these 2 syndromes can occur (as in this patient). Microscopy of blood, urine, or joint fluid samples will often reveal gram-negative intracellular diplococci. (Choice B) Salmonella appears as a gram-negative oxidase-negative rod. Patients with sickle cell disease are at increased risk of septic arthritis due to salmonella. However, salmonella is an uncommon cause of septic arthritis in patients who do not have sickle cell disease. (Choice C) Viridans streptococci, a gram-positive alpha-hemolytic cocci in chains, is a common cause of infective endocarditis and can lead to septic arthritis (due to septic emboli). Patients with infective endocarditis can have flu-like symptoms and skin findings (eg, Osler nodes, Janeway lesions). However, vesiculopustular lesions on the arms are much more characteristic of DGI. (Choice D) Staphylococcus aureus, a catalase-positive gram-positive cocci in clusters, is the most common cause of septic arthritis. However, most cases in young adults are linked to injection drug use. In addition, the presence of vesiculopustular lesions and oligoarticular arthralgias make gonococcal arthritis more likely. (Choices E and F) Gout is caused by monosodium urate crystals that appear as needle-shaped negatively birefringent crystals. Pseudogout is caused by calcium pyrophosphate dihydrate crystals that appear as rhomboid-shaped positively birefringent crystals. Both gout and pseudogout cause inflammatory arthritis (eg, redness, warmth, pain, disability) with elevated leukocyte counts on joint fluid aspiration. However, migrating arthralgias and vesiculopustular lesions would be atypical. Educational objective: Septic arthritis in a young, sexually active adult should raise suspicion for disseminated Neisseria gonorrhoeae infection. Patients may also have the triad of polyarthritis, a vesiculopustular skin rash, and tenosynovitis. N gonorrhoeae is a gram-negative diplococci that is usually identified by microscopy, culture, or nucleic acid amplification.

A 44-year-old woman comes to the emergency department due to acute-onset, severe, right lower quadrant abdominal pain, nausea, vomiting, and hematuria over the last 4 hours. She had a similar episode of acute pain a year ago, but it resolved in a few hours and she did not seek medical intervention. The patient has no other medical conditions and takes no medication. She smokes a pack of cigarettes daily. She is sexually active and has never been pregnant. Temperature is 36.9 C (98.4 F), blood pressure is 140/90 mm Hg, and pulse is 102/min. There is mild tenderness to deep palpation in the right lower quadrant. Laboratory results are as follows: Serum chemistry Urea nitrogen 15 mg/dL Creatinine 1.0 mg/dL Glucose 90 mg/dL Calcium 11 mg/dL Phosphorus 2.5 mg/dL Which of the following is the most likely cause of this patient's current condition? Appendicitis Diverticulitis Glomerulonephritis Ovarian torsion Renal cell carcinoma Renal infarction Ureterolithiasis

This patient has recurrent abdominal pain, vomiting, and hematuria. In conjunction with hypercalcemia and hypophosphatemia, this presentation suggests ureterolithiasis due to hyperparathyroidism. Most kidney stones are made up of calcium salts and are idiopathic, but conditions that increase calcium excretion increase the risk of stone formation. Primary hyperparathyroidism leads to increased bone resorption, decreased urinary phosphate reabsorption, and increased 1,25-dihydroxyvitamin D formation, all of which result in hypercalcemia and hypophosphatemia. Despite the increased fractional reabsorption of calcium induced by PTH, net urinary calcium excretion is elevated due to the increased filtered calcium load, raising the risk for stone formation. Pain from ureterolithiasis, which occurs when the stone obstructs renal drainage, typically waxes and wanes. Obstruction at the ureteropelvic junction normally presents with flank or upper abdominal pain, whereas an obstructing stone at the ureterovesical junction usually presents with lower abdominal or groin pain. Other common symptoms include hematuria, nausea, and vomiting. (Choices A and B) Appendicitis presents with acute periumbilical or right lower quadrant pain and tenderness, but it would not usually cause recurrent symptoms. Acute diverticulitis can cause recurrent lower abdominal symptoms but is much more common on the left and typically occurs in older (age >60) patients. Neither of these conditions are associated with hematuria or hypercalcemia. (Choice C) Glomerulonephritis is a category of kidney disorders characterized by hematuria, typically with red cell casts. Common associated features include hypertension, oliguria, and acute renal failure. Patients with various nephritic syndromes may have moderate flank pain, but acute, severe lower abdominal pain is not consistent with glomerulonephritis. (Choice D) Ovarian torsion can present with acute, severe lower abdominal or pelvic pain. It can be recurrent but would not cause hematuria. (Choice E) Renal cell cancer can cause hematuria and hypercalcemia; however, pain (if present) typically presents in the flank rather than the lower abdomen. (Choice F) Renal infarction typically occurs due to acute obstruction of the renal arteries (eg, cardiac thromboembolism, aortic dissection). Patients often develop abdominal pain and hematuria but also typically have fever and marked hypertension (due to renin release). In addition, renal infarction is not associated with hypercalcemia or hypophosphatemia. Educational objective: Most kidney stones are made of calcium salts and are idiopathic, but conditions that increase renal calcium excretion can increase the risk of stones. Hyperparathyroidism is a common cause of recurrent kidney stones and is typically associated with mild hypercalcemia and hypophosphatemia.

A 45-year-old woman comes to the office due to gradually progressive pain, stiffness, and swelling of her hand joints for the past several months. The patient has had prolonged morning stiffness and significant restriction of her daily activities due to the pain, as well as generalized fatigue. She describes the symptoms over the last several weeks as "disabling," and she needs assistance with activities of daily living in the morning. The patient has tried over-the-counter pain medications, such as acetaminophen and ibuprofen, with only minimal relief of her symptoms. She has no other medical problems. Family history is significant for diabetes mellitus on her maternal side. Which of the following drugs would provide the most rapid relief of her symptoms? Colchicine Hydroxychloroquine Methotrexate Minocycline Prednisone Sulfasalazine

This patient has swelling, pain, and morning stiffness in multiple joints for >6 weeks, consistent with rheumatoid arthritis (RA). The foundation of management for RA is disease-modifying antirheumatic drugs (DMARDs), which alleviate pain and inflammation and reduce long-term joint destruction and disability. Examples of DMARDS are methotrexate (typically first-line), sulfasalazine, hydroxychloroquine, minocycline, and tumor necrosis factor-alpha inhibitors. However, the response to DMARD therapy typically takes weeks (Choices B, C, D, and F). Therefore, short-term treatment with anti-inflammatory therapies, including systemic and intraarticular glucocorticoids (eg, prednisone) or nonsteroidal anti-inflammatory drugs (NSAIDs), can provide rapid, temporary relief of symptoms in patients starting on DMARDs. However, they do not provide adequate long-term control of disease or prevention of joint deformity. Glucocorticoids exert their anti-inflammatory effects by inhibiting phospholipase A2, which decreases prostaglandin and leukotriene synthesis. Glucocorticoids also depress the immune response by inhibiting the transcription of multiple cytokines and adhesion proteins, which reduces leukocyte recruitment and activation. (Choice A) Colchicine is used in the management of both acute and chronic gout. Colchicine works by binding to tubulin and inhibiting microtubule polymerization. This results in disruption of neutrophil chemotaxis and phagocytosis, thereby reducing the inflammatory response to uric acid crystals. Colchicine is not effective in RA. Educational objective: The foundation of management for rheumatoid arthritis is disease-modifying antirheumatic drugs, which alleviate pain and inflammation and reduce long-term joint destruction. However, the response to treatment may take several weeks. Nonsteroidal anti-inflammatory drugs and glucocorticoids can provide rapid symptom relief in the interim.

A 45-year-old man comes to the emergency department due to 2 days of left knee swelling and pain. The patient has no significant past medical history apart from an episode of facial palsy 3 months ago. Six months ago, he went on a hiking trip to New Hampshire but has no other travel history. The patient is in a long-standing monogamous relationship with his wife and has had no other sexual partners. Temperature is 37.1 C (98.8 F). Examination shows left knee joint swelling with no surrounding erythema. The remainder of the musculoskeletal examination is unremarkable. There is no heart murmur. Which of the following might have prevented this patient's knee condition? Allopurinol Ceftriaxone Lamivudine Live attenuated vaccine Mefloquine Metronidazole

This patient has symptoms consistent with long-standing Lyme disease. Lyme disease is caused by the spirochete Borrelia burgdorferi, which is transmitted by the bite of an Ixodes tick. Lyme disease is endemic in the northeastern United States and northern Europe. Like syphilis, another spirochetal infection, Lyme disease progresses through a predictable series of phases: Early localized phase: Days to weeks following exposure, patients experience flu-like symptoms and the characteristic cutaneous eruption, erythema chronicum migrans (ECM). Early disseminated phase: Weeks or months later, there may be central nervous system (eg, facial palsy) and/or cardiac involvement (eg, atrioventricular nodal block). Late Lyme disease: This phase, which occurs months to years post-exposure, is rare given the frequency with which patients receive antibiotics for other indications. Patients may experience asymmetric arthritis (most often involving a single knee joint, as in this patient) and/or subacute encephalopathy with decreased memory, somnolence, and mood changes. Lyme disease is easily treated with doxycycline or penicillin-type antibiotics (eg, ceftriaxone). This patient does not appear to have noted ECM, for which he would likely have received doxycycline. However, treatment with ceftriaxone when he developed the facial palsy (a possible manifestation of early disseminated Lyme disease) would have prevented progression to late Lyme disease. (Choice A) Allopurinol competitively inhibits xanthine oxidase, suppressing uric acid synthesis. This drug is used to treat gout and prevent uric acid kidney stones and tumor lysis syndrome. (Choice C) Lamivudine is a nucleoside reverse transcriptase inhibitor used for HIV treatment. (Choice D) A Borrelia burgdorferi vaccine, now discontinued due to low sales, contains a recombinant outer surface protein from these bacteria, not live organisms. (Choice E) Mefloquine is an antimalarial drug used in prevention and treatment of Plasmodium falciparum infections, among others. (Choice F) Metronidazole is effective against anaerobic bacteria (eg, Clostridium difficile, Bacteroides) and parasites (eg, Entamoeba histolytica, Giardia lamblia, Trichomonas vaginalis). Educational objective: Early Lyme disease causes flu-like symptoms and erythema chronicum migrans. The second stage of Lyme disease may involve atrioventricular block and facial palsy. Late Lyme disease can cause chronic asymmetric large joint arthritis and encephalopathy. Lyme disease is easily treated with doxycycline or penicillin-type antibiotics (eg, ceftriaxone).

A 22-year-old man comes to the office due to 3 months of progressive back pain. He has also had intermittent subjective fever. The patient emigrated from Nepal for a master's program 6 months ago. He has no other medical conditions, takes no medication, and does not use tobacco, alcohol, or illicit drugs. Temperature is 38.3 C (100.9 F), blood pressure is 122/78 mm Hg, and pulse is 84/min. BMI is 19 kg/m2. Tenderness is noted over the lumbar spine; flexion, extension, and rotation of the spine are limited due to pain and muscle spasm. MRI of the lower spine reveals partial destruction of the anterior portion of the L1-L3 vertebral bodies and a fluid collection beneath the anterior longitudinal ligament. Which of the following is the most likely diagnosis? Hodgkin lymphoma Osteosarcoma Rhabdomyosarcoma Staphylococcus aureus infection Tuberculosis

This patient who recently emigrated from a tuberculosis-endemic region (Nepal) now has progressive back pain, intermittent fever, and vertebral bone destruction with an adjacent fluid collection (abscess), raising strong suspicion for Mycobacterium tuberculosis spondylitis (Pott disease). Primary tuberculosis infections are acquired via inhalation and cause localized pulmonary infection that is typically contained over weeks by the cell-mediated immune response. However, transient bacteremia can occur prior to containment and result in the hematogenous seeding of highly vascular organs (eg, spleen, liver, bones); the vertebrae are particularly likely to be affected due to their extensive venous plexuses. Pott disease typically occurs months to years following primary pulmonary infection. It is characterized by intermittent fever and slowly worsening pain in the lumbar or lower thoracic spine. The infection frequently spreads behind the anterior ligament to the adjacent vertebrae and intervertebral disc space, leading to contiguous bone destruction and abscess formation. (Choice A) Hodgkin lymphoma is often associated with intermittent fever but typically presents with regional adenopathy and/or a mediastinal mass. Vertebrae destruction and a fluid collection are not common features. (Choices B and C) Osteosarcoma is the most common primary bone tumor in young adults; however, the long bones are primarily affected, and imaging typically shows a lytic bone lesion. Rhabdomyosarcoma is a soft-tissue sarcoma that usually arises in the head/neck, genitourinary tract, or extremities; imaging usually reveals a soft-tissue mass. Neither is typically associated with fever and a fluid collection; these features make an infectious process far more likely. (Choice D) Staphylococcus aureus is the most common cause of spinal epidural abscess. Although the radiographic appearance can be similar to Pott disease, most cases arise in the setting of intravenous drug use, hematogenous spread of a distant infection (eg, endocarditis), or direct inoculation during a spinal procedure. This patient's recent emigration from a tuberculosis-endemic area makes Pott disease more likely. Educational objective: Mycobacterium tuberculosis spondylitis (Pott disease) is usually the result of hematogenous seeding of vertebrae from primary pulmonary infection. Manifestations typically arise months or years later (due to reactivation) and include chronic, progressive back pain, fever, and radiographic evidence of vertebral bone destruction and fluid collection.

A 56-year-old man comes to the urgent care center with severe knee pain. He was feeling well until he awoke this morning with acute pain, redness, and swelling in the knee. The patient does not smoke but did drink heavily at a wedding reception the night before. Past medical history is notable for recent peptic ulcer disease. He is treated with oral colchicine and experiences significant relief of symptoms within 12 hours. The drug used in this patient most likely affects which of the steps shown in the diagram below? ID=858 Phospholipids>AA AA> Leukotrienes AA> Prostaglandins LTC4/D4/E4 > Vascular and other effects Prostaglandins> Vascular and other effects Leukocyte Attraction> Inflammation

This patient with acute atraumatic monoarticular arthritis involving the knee has typical symptoms of gout. The usual first-line treatment of acute gouty arthritis is nonsteroidal anti-inflammatory drugs (NSAIDs) (eg, indomethacin), but these are relatively contraindicated in patients with a history of peptic ulcer. Colchicine can also be used to treat acute gouty arthritis and results in resolution of symptoms in most patients. Colchicine works primarily by inhibiting microtubular polymerization. It binds to the tubulin protein that helps form microtubules, preventing their aggregation. This in turn disrupts cytoskeletal-dependent functions such as chemotaxis, phagocytosis, and degranulation. Colchicine also reduces the formation of leukotriene B4. It does not have any effect on the metabolism or urinary excretion of uric acid. Important adverse effects of colchicine are nausea, abdominal pain, and diarrhea. Colchicine should not be used in elderly patients or those with severe renal dysfunction. (Choices A and E) Glucocorticoids inhibit the enzyme phospholipase A2, which converts membrane phospholipids to arachidonic acid. This in turn decreases the formation of inflammatory prostaglandins and leukotrienes. Glucocorticoids can be given systemically or by intra-articular injection for treatment of gout in patients who cannot take NSAIDs or colchicine. (Choices B and D) The leukotriene pathways can be blocked by inhibiting the 5-lipoxygenase pathway or by blocking the leukotriene receptors. Zileuton is a specific inhibitor of the enzyme 5-lipoxygenase and inhibits leukotriene formation (LTB4, LTC4, LTD4, and LTE4). Montelukast inhibits LTD4 receptors. Leukotriene modifiers are used in the treatment of asthma and allergic rhinitis but have no role in the treatment of gout. (Choice C) Cyclooxygenase (COX) catalyzes the conversion of arachidonic acid to prostanoids. It exists in 2 isoforms, COX-1 and COX-2. COX-2 is an inducible enzyme that is upregulated in inflammatory cells to increase synthesis of pro-inflammatory arachidonic acid metabolites. Standard NSAIDs (eg, indomethacin, ibuprofen) inhibit both COX-1 and COX-2, while selective COX-2 inhibitors (eg, celecoxib) inhibit only COX-2. Educational objective: Colchicine is used for treatment of acute gouty arthritis in patients who cannot take nonsteroidal anti-inflammatory drugs. It inhibits leukocyte migration and phagocytosis by blocking tubulin polymerization. Significant side effects of colchicine include nausea and diarrhea.

A 65-year-old man comes to the emergency department due to fevers and malaise for the last 48 hours. The patient initially experienced fevers and chills, which subsequently progressed to fatigue to the point that he had difficulty performing routine tasks this morning. He has also had a mild headache. The patient lives in eastern Massachusetts with his wife. He is retired and spends time gardening and swimming at a local public pool. The patient traveled to Thailand 2 years ago with his family and to Florida 2 months ago. Temperature is 38.9 C (102 F), blood pressure is 122/70 mm Hg, and pulse is 114/min and regular. The oropharynx is dry and has no lesions. Lung examination demonstrates coarse crackles bilaterally. The abdomen is soft and mildly tender throughout with palpable splenomegaly. He has no rash or open wounds. Peripheral blood smear is shown in the exhibit. Which of the following is the most likely source of this patient's infection? ID=11524 Aedes mosquito Brown recluse spider Dog bite Freshwater consumption Ixodes tick Scratch from a garden bush Spore inhalation

This patient with an acute febrile illness has a peripheral blood smear showing intraerythrocytic ring inclusions. Parasites that can potentially cause disease in red blood cells include Plasmodium (malaria) and Babesia (babesiosis). In this patient, the epidemiology (residence in northeastern United States, outdoor exposure in the summer) is suggestive of babesiosis. This organism is transmitted by the Ixodes scapularis tick, which is also a vector for Borrelia burgdorferi (Lyme disease) and Anaplasma (human granulocytic anaplasmosis). (Choice A) Aedes is the vector for dengue and chikungunya, which have been diagnosed in Florida but have different presentations (eg, rash, arthralgias) and a short incubation period (<14 days). Malaria is transmitted by Anopheles and is endemic in Thailand. Although it can also cause intraerythrocytic ring inclusions, it has a typical incubation period of weeks; this patient visited Thailand 2 years ago. (Choice B) Brown recluse spider bites can cause fever and malaise and, less commonly, anemia; however, they would not be associated with intraerythrocytic organisms. (Choice C) Dog bites can be associated with a variety of organisms causing soft tissue infection, including Pasteurella, streptococci, and staphylococci. (Choice D) Freshwater consumption can be associated with intraintestinal bacterial and parasitic infections such as cryptosporidiosis and giardiasis. However, these infections typically cause gastrointestinal illnesses. (Choice F) Exposure to plants and soil can be associated with bacterial infections, including Nocardia and fungal infections such as sporotrichosis. These most commonly present as soft tissue infections; systemic infections are rare in immunocompetent patients. (Choice G) Fungal spore inhalation can lead to pulmonary infections, including histoplasmosis and blastomycosis. These organisms are not endemic to the northeastern United States and are not associated with intraerythrocytic organisms. Educational objective: Babesiosis should be considered in patients with febrile illness who reside in geographic areas with Ixodes ticks. The diagnosis can be established with identification of intraerythrocytic organisms on peripheral blood smear.

A 48-year-old man comes to the hospital due to 10 days of persistent fever, chills, fatigue, and dyspnea. He had a splenectomy at age 28 after a motor vehicle accident and received the appropriate vaccines. The patient does not use tobacco, alcohol, or illicit drugs. He recently traveled from New York City to Connecticut for a family wedding. His temperature is 38.6 C (101.5 F), blood pressure is 118/70 mm Hg, and pulse is 108/min. Lung examination reveals coarse crackles. Laboratory studies are as follows: Complete blood count Hemoglobin 9.6 g/dL Reticulocytes 4% Platelets 108,000/mm3 Leukocytes 11,000/mm3 Liver function studies Total bilirubin 1.8 mg/dL Direct bilirubin 0.6 mg/dL Alkaline phosphatase 120 U/L Aspartate aminotransferase (SGOT) 87 U/L Alanine aminotransferase (SGPT) 74 U/L Lactate dehydrogenase 188 U/L (normal: 60-100 U/L) Haptoglobin 36 mg/dL (normal: 50-150 mg/dL) Chest x-ray reveals bilateral infiltrates. A peripheral blood smear shows normocytic, normochromic anemia with ring- and cross-shaped intraerythrocytic inclusions. The vector responsible for this patient's current condition can also transmit which of the following organisms? Borrelia burgdorferi Dengue virus Haemophilus influenzae Histoplasma capsulatum Neisseria meningitidis Plasmodium falciparum

This patient with an acute febrile illness, thrombocytopenia, hemolytic anemia (indirect hyperbilirubinemia, elevated lactate dehydrogenase, low haptoglobin), abnormal liver function tests, and intraerythrocytic inclusions (ring-shaped and "Maltese cross" forms) has babesiosis, a tick-borne infection. A definitive diagnosis can be made from a Giemsa-stained blood smear showing intraerythrocytic parasites. Splenectomy places patients at increased risk for severe babesiosis, which can manifest as acute respiratory distress syndrome (dyspnea, coarse crackles, bilateral infiltrates on chest x-ray). Babesiosis (caused by Babesia microti) and Lyme disease (caused by Borrelia burgdorferi) are both transmitted by the Ixodes tick and occur in similar geographic regions. Coinfection is common and could be considered in the appropriate epidemiologic setting (eg, patients who present with babesiosis in a region endemic for Lyme disease and who do not improve despite appropriate treatment). (Choices B and F) Dengue virus is transmitted by Aedes mosquitos, and Plasmodium falciparum is transmitted by Anopheles mosquitos. Dengue fever and malaria may present as acute febrile illnesses, but both infections occur mainly in tropical regions. Malaria is characterized by intravascular hemolysis, anemia, and abnormal liver function tests (similar to babesiosis), but the blood smear frequently shows ring-shaped (not cross-shaped) erythrocyte inclusions. (Choices C and E) Asplenic patients are at increased risk for life-threatening infections with encapsulated organisms (eg, Streptococcus pneumoniae, Haemophilus influenzae, Neisseria meningitidis) and therefore receive protective vaccines. Intraerythrocytic inclusions are also not seen in H influenzae and N meningitidis infections. (Choice D) Histoplasmosis is a common mycotic infection that is most commonly found in the Ohio and Mississippi river basins and is associated with activities involving exposure to bird or bat droppings (eg, caving). Educational objective: Babesiosis and Lyme disease are transmitted by the Ixodes tick and occur in similar geographic regions. Coinfection is common.

A 30-year-old woman has had diarrhea and weight loss for the past several months. She also has diffuse bone pain and generalized weakness. The patient has a history of primary hypothyroidism for which she takes levothyroxine. She is 162.5 cm (5 ft 4 in) tall and weighs 45 kg (99.2 lb). BMI is 17 kg/m2. On physical examination, the abdomen is soft and nontender. Initial laboratory evaluation reveals microcytic anemia, hypoalbuminemia, normal magnesium level, and normal serum TSH concentration. The patient is found to have positive anti-tissue transglutaminase IgA antibodies. Which of the following sets of additional laboratory findings are most likely present in this patient? Serum calcium, serum phosphorus, serum parathyroid hormone up, down, up down, down, up down, up, down down, up, up normal, up, up

This patient with diarrhea, weight loss, and a positive tissue transglutaminase antibody assay has celiac disease, an immune-mediated hypersensitivity to dietary gluten. Celiac disease is characterized by villous atrophy in the small intestine, leading to malabsorption of dietary fats and fat-soluble vitamins (ie, A, D, E, K). The resulting vitamin D deficiency can present as rickets in children and osteomalacia in adults. Vitamin D increases intestinal absorption of calcium and phosphorus; deficiency reduces calcium absorption, which in turn stimulates release of parathyroid hormone (PTH). Vitamin D also directly inhibits PTH release, and therefore vitamin D deficiency facilitates a significant rise in PTH (secondary hyperparathyroidism). PTH induces release of calcium and phosphorus from bones, leading to decreased bone mineralization. (Although most phosphorus in the body is in the form of hydroxyapatite in bone, circulating phosphorus is primarily in the form of phosphate/phosphoric acids.) Typical laboratory findings in vitamin D deficiency include: low 25-hydroxyvitamin D, which reflects total body vitamin D stores (PTH stimulates renal conversion of 25-hydroxyvitamin D to 1,25-dihydroxyvitamin D; therefore, 1,25-dihydroxyvitamin D may remain within laboratory norms). elevated PTH. increased alkaline phosphatase, reflecting increased bone turnover. low serum phosphorus, due to decreased intestinal absorption and increased PTH-mediated renal excretion. PTH may initially maintain normal serum calcium levels by reducing urinary calcium excretion. However, hypocalcemia may eventually develop as bone stores are depleted in later or more severe cases. (Choice A) Primary hyperparathyroidism presents with elevated PTH, mild hypercalcemia, and low serum phosphorus (due to increased renal excretion) but is usually due to a parathyroid adenoma and is not associated with celiac disease. (Choice C) In hypoparathyroidism, low serum PTH is accompanied by hypocalcemia and an increase in serum phosphorus. Hypoparathyroidism can be caused by severe hypomagnesemia, which can be seen with prolonged diarrhea, but this patient's magnesium level is normal. (Choices D and E) Chronic kidney disease causes hyperphosphatemia due to decreased filtration and excretion of phosphate. Concurrently, there is decreased renal production of 1,25-dihydroxyvitamin D (the activated form) due to inadequate renal metabolic activity and suppression of 1-alpha-hydroxylase by hyperphosphatemia. Hypocalcemia may be present, although increased PTH sometimes maintains calcium within laboratory norms. Pseudohypoparathyroidism is characterized by resistance to PTH and presents with similar biochemical markers. Educational objective: Malabsorption caused by celiac disease can lead to vitamin D deficiency. Patients have decreased serum phosphorus, increased serum parathyroid hormone (secondary hyperparathyroidism), and low (or normal) serum calcium.

A 56-year-old woman is brought to the emergency department due to several days of progressive dyspnea, productive cough, and fever. Today, her son found her very short of breath and obtunded. The patient has a history of hypertension, type 2 diabetes mellitus, and long-standing rheumatoid arthritis. Examination reveals bilateral pneumonia and severe respiratory distress. The patient is lethargic without focal neurologic deficits. Urgent endotracheal intubation is performed for mechanical ventilation, and the patient is started on broad-spectrum antibiotics. Repeat examination 2 hours later shows that she has developed areflexic, flaccid paralysis of all extremities. Which of the following is the most likely cause of the neurologic deficits in this patient? Cerebral septic emboli Diabetic neuropathy Gullain-Barre syndrome Malignant hyperthemia Vertebral subluxation

This patient with rheumatoid arthritis underwent urgent endotracheal intubation due to severe pneumonia and subsequently developed sudden-onset quadriparesis. Long-standing rheumatoid arthritis frequently involves the cervical spine and causes joint destruction with vertebral malalignment (subluxation). The atlantoaxial joint is most often involved as the atlas (C1) has a high degree of mobility relative to the axis (C2 odontoid and body). Chronic symptoms of cervical subluxation include neck pain, stiffness, and neurologic findings (eg, sensory loss, muscle weakness). Endotracheal intubation can acutely worsen the subluxation and cause compression of the spinal cord and/or vertebral arteries. Acute spinal cord injury results in flaccid paralysis with decreased or absent reflexes below the level of the compression due to spinal shock; the paralysis eventually becomes spastic as spinal shock resolves over the ensuing days to weeks. (Choice A) Cerebral septic emboli are usually due to infective endocarditis of the left-sided heart valves. Patients can develop acute neurologic findings including unilateral paralysis; quadriparesis would be less likely. This patient also has no cardiac murmur or other stigmata (eg, petechiae, splinter hemorrhages) to suggest endocarditis. (Choice B) Diabetic neuropathy presents with a slowly progressive decrease in vibratory sensation, proprioception, and temperature sensation. Early neuropathy can cause decreased ankle reflexes, whereas later disease can cause widespread loss of reflexes with motor weakness. (Choice C) Guillain-Barré syndrome presents with progressively worsening, symmetrical, flaccid muscle weakness with absent or decreased reflexes. The abnormalities typically start in the lower extremities and ascend upward over the course of days to weeks. (Choice D) Malignant hyperthermia is usually due to anesthesia exposure (eg, halothane, succinylcholine), causing increased and sustained muscle contraction with myocyte breakdown. Patients typically develop diffuse muscle rigidity followed by hyperthermia and possible myoglobinuria. Educational objective: Long-standing rheumatoid arthritis can affect the cervical spine, causing vertebral malalignment (subluxation) that can affect the atlantoaxial joint. Extension of the neck during endotracheal intubation can worsen the subluxation, leading to acute compression of the spinal cord and/or vertebral arteries.

A 67-year-old man comes to the office due to slowly worsening pain in both legs. He has had no trauma, fever, or chills. The pain is deep and aching, is present throughout the day, and increases with weight bearing. Vital signs are normal. Examination shows bilateral bowing of the legs with palpable warmth over the anterior shins but no redness. The remainder of the examination shows no abnormalities. Imaging studies reveal bone expansion, with cortical and trabecular thickening affecting both the tibia and fibula. Which of the following patterns of serum laboratory values is most likely in this patient? Calcium, phosphorus, alkaline phosphatase normal, normal, normal up, normal, normal up, up, normal up, up, up normal, normal, up normal, down, up down, down, up

This patient with skeletal pain, deformity (ie, bowing of the legs), and focal warmth has Paget disease of bone (osteitis deformans). Paget disease is a chronic disorder characterized by excessive and disordered bone formation. The affected bone becomes weakened, which can lead to bone pain, bowing, fracture, or arthritis of adjacent joints. Increased blood flow in pagetic lesions can be apparent as local warmth or bruits and can occasionally lead to high-output heart failure. Patients with Paget disease frequently have an elevated serum alkaline phosphatase level due to increased formation of new bone. However, calcium, phosphorus, and parathyroid hormone levels are normal, as calcium homeostasis remains intact by the disease process. X-rays are usually diagnostic, showing mixed lytic-sclerotic lesions, thickening of cortical and trabecular bone, and bony deformities. Educational objective: Paget disease of bone is characterized by disordered bone formation. Involvement of long bones can lead to bone pain, bowing, fracture, or arthritis of adjacent joints. Serum alkaline phosphatase is elevated due to increased production of new bone, but calcium and phosphorus levels remain normal.

A 3-year-old boy who recently immigrated to the United States is brought to the emergency department with fever, malaise, and a painful, swollen right knee. He is hypotensive and tachycardic. Medical history is significant for a recent episode of acute otitis media and several vaccinations that are not up to date. Arthrocentesis of the right knee shows cloudy synovial fluid. Gram stain of the aspirate reveals pleomorphic, gram-negative coccobacilli. Cultures performed on a blood agar plate supplemented with a disk containing hematin and nicotinamide adenine dinucleotide (NAD+) grow colonies only near the disk. The organism responsible for this patient's condition most likely produces which of the following virulence factors? Capsule Cytotoxic exotoxin Fimbriae Hemolysis Hyaluronidase

This patient's arthrocentesis findings are diagnostic for septic arthritis due to Haemophilus influenzae. H influenzae is a small, pleomorphic, gram-negative coccobacillus that requires both X factor (hematin) and V factor (NAD+) to grow. Because these factors are found within erythrocytes, optimal concentrations are present only in lysed blood agar (chocolate agar). Growth on regular blood agar requires exogenous supplementation of X and V factors, which are typically provided by an impregnated disk. Localized infections (eg, acute otitis media, sinusitis) are caused predominantly by unencapsulated (nontypeable) strains of H influenzae, although H influenzae type b (Hib) can also cause these infections in undervaccinated patients. Type b strains have a polyribosylribitol phosphate polysaccharide capsule that prevents phagocytosis. This allows the organism to invade the vasculature and spread hematogenously to distant sites, predisposing to invasive disease (eg, septic arthritis, meningitis, bacteremia). In vaccinated patients, antibodies against the type b capsule provide immunity by promoting complement fixation and opsonization. Therefore, invasive Hib infections have been rare since the advent of the Hib immunization. (Choice B) Cytotoxic exotoxins are secreted by certain bacteria (Clostridium botulinum, Clostridium tetani, Corynebacterium diphtheriae) to promote host cell death. No strains of H influenzae produce an exotoxin. (Choice C) Fimbriae are proteinaceous projections from bacterial cells that attach to target tissues during the initial stages of an infectious process. Fimbriae on H influenzae enable attachment to endothelial cells during colonization of the respiratory tract but are not the major virulence factor contributing to invasive disease. (Choice D) Staphylococcus aureus produces alpha-hemolysin, which causes infection by destroying the cell membrane of red and white blood cells. Hemolysins are not secreted by H influenzae. (Choice E) Hyaluronidase is an enzyme used by bacteria (eg, S aureus, Streptococcus pyogenes, Clostridium perfringens) to digest extracellular ground substance and enhance their ability to spread. Hyaluronidase is not produced by H influenzae. Educational objective: Haemophilus influenzae is a gram-negative coccobacillus that requires both X factor (hematin) and V factor (NAD+) to grow. H influenzae type b has an antiphagocytic polysaccharide capsule, which allows it to spread hematogenously and cause invasive disease such as septic arthritis and meningitis.

A 52-year-old man recovering from an acute myocardial infarction develops new-onset chest pain that is relieved by leaning forward. The pain is exacerbated with deep inspiration. The patient was admitted with acute inferior myocardial infarction 2 days ago and did not undergo revascularization due to his late presentation. Since admission, he has had no chest pain until this morning. On physical examination, a friction rub is present at the left sternal border. The patient is treated with a medication that inhibits COX-1 and COX-2 enzymes via irreversible acetylation, resulting in significant relief of his symptoms. Which of the following agents was most likely used in this patient? Acetaminophen Aspirin Budesonide Celecoxib Colchicine Diclofenac Ibuprofen Indomethacin

This patient's presentation of recent (within 1-3 days) myocardial infarction, pleuritic chest pain improved by leaning forward, and a friction rub on cardiac examination likely indicates peri-infarction pericarditis. Although most patients with peri-infarction pericarditis improve with supportive measures, those with sufficient or persistent symptoms are typically treated with aspirin in combination with colchicine. Aspirin (acetylsalicylic acid) is a nonsteroidal anti-inflammatory drug (NSAID) that irreversibly inhibits cyclooxygenase-1 and -2 (COX-1 and -2) enzymes via acetylation, preventing conversion of arachidonic acid to prostaglandins, prostacyclin, and thromboxane. COX-1 acetylation inhibits generation of thromboxane A2 in platelets (antithrombotic effect). COX-2 acetylation blocks prostaglandin production in inflammatory cells (eg, activated lymphocytes, neutrophils), resulting in anti-inflammatory, antipyretic, and analgesic effects. (Choice A) Acetaminophen is an analgesic and antipyretic agent that reversibly inhibits COX enzymes (primarily in the central nervous system). It lacks anti-inflammatory properties due to its weak inhibition of COX in peripheral tissues. (Choice C) Budesonide is a glucocorticoid agent with potent anti-inflammatory activity. Corticosteroids reduce COX-2 expression and decrease synthesis of arachidonic acid via phospholipase A2 inhibition. (Choice D) Celecoxib is a selective COX-2 inhibitor that provides anti-inflammatory benefits without interfering with the physiological functions of COX-1. In contrast to the non-selective NSAIDs, selective COX-2 inhibitors have no significant effect on platelet function and are associated with a lower incidence of gastrointestinal bleeding. However, selective COX-2 inhibition is associated with an increased incidence of thrombosis. (Choice E) Colchicine exerts its anti-inflammatory effects by inhibiting polymerization of beta-tubulin into microtubules, preventing migration and activation of neutrophils. (Choices F, G, and H) Diclofenac, ibuprofen, and indomethacin are NSAIDs that reversibly inhibit COX-1 and COX-2. Educational objective: Aspirin (acetylsalicylic acid) is a nonsteroidal anti-inflammatory drug (NSAID) that irreversibly inhibits cyclooxygenase-1 and -2 (COX-1 and -2) enzymes via acetylation. In contrast, other NSAIDs (eg, diclofenac, ibuprofen, indomethacin) reversibly inhibit COX-1 and COX-2.

A 27-year-old man comes to the emergency department with progressive right knee swelling and pain. He has no history of trauma to the area. The patient has no other medical conditions and takes no medications. X-rays reveal a large lytic lesion involving the proximal tibia with extensive soft-tissue swelling. After additional confirmatory testing, the patient undergoes a right-sided, above-knee amputation. Histologic examination of the resected mass is shown in the exhibits. Which of the following is the most likely diagnosis? ID=15636 Chondrosarcoma Ewing sarcoma Metastatic adenocarcinoma Osteoid ostemoa Osteosarcoma

This patient's spindle-shaped tumor cells admixed with bone and osteoid indicates osteosarcoma, the most common primary bone tumor in children and young adults. Most cases arise in the metaphyses of long bones (eg, proximal tibia), the location of the growth plate and site of greatest bone proliferation. Patients typically present with pain and soft-tissue swelling, and x-ray usually reveals a lytic bone lesion. Osteosarcomas arise from a malignant mesenchymal stem cell that generates cartilage, bone, or fibrous tissue. Therefore, the diagnosis is confirmed when histopathology reveals neoplastic spindle-shaped stromal cells admixed with tumor osteoid and thin trabeculae of bone. (Choice A) Unlike osteosarcomas, chondrosarcomas and fibrosarcomas do not produce osteoid or bone. Chondrosarcomas are characterized by neoplastic chondrocytes in a hyaline cartilage matrix, usually with small calcifications. (Choice B) Ewing sarcoma, the second most common primary bone malignancy in young patients, often arises in long bones and causes progressive pain, swelling, and lytic bone lesions. However, histopathology reveals sheets of small, round, cells separated by fibrous septae and patches of necrosis/hemorrhage; no osteoid or bone is produced. (Choice C) Adenocarcinoma of the lung often metastasizes to bone. However, histology would show neoplastic glands lined with mucin-producing cells. (Choice D) Osteoid osteoma is a small, benign, bone-forming tumor that typically occurs in adolescent boys. Histopathology shows irregular patterns of woven bone lined by a single layer of benign-appearing osteoblasts. The presence of a large lytic bone lesion and highly pleomorphic spindle-shaped tumor cells makes this diagnosis unlikely. Educational objective:Osteosarcoma is the most common primary bone tumor in children and young adults and typically arises near the metaphyses of long bones. Patients usually have slowly worsening pain and soft-tissue swelling. X-ray typically reveals a lytic bone lesion, and biopsy classically shows pleomorphic, spindle-shaped tumor cells that generate osteoid and thin trabeculae of neoplastic bone.

A 47-year-old woman comes to the physician with progressive joint pain and swelling in her hands for the past several months. She also has easy fatigability that has gradually worsened over the same period. Morning activities are especially difficult due to stiffness lasting 1 to 2 hours after waking. Examination shows warmth, swelling, and tenderness involving the proximal interphalangeal joints, metacarpophalangeal joints, and wrists bilaterally. A blood sample is obtained for laboratory analysis. Autoantibodies against which of the following components are most specific for this patient's condition? Centromeres Citrullinated peptides Double-stranded DNA Fc portion of human IgG Nuclear basic proteins Phospholipids

This patient's symmetric polyarthritis (involving the MCP and PIP joints) with prolonged morning stiffness and associated fatigue is highly suggestive of rheumatoid arthritis (RA). The diagnosis is made clinically, but the presence of anti-cyclic citrullinated peptide (anti-CCP) antibodies is helpful for confirmation. Tissue inflammation causes arginine residues in proteins such as vimentin to be enzymatically converted into citrulline through a process called citrullination. This alters the shape of the proteins, which can then serve as neoantigens that generate an immune response. In RA, the immune response against citrullinated proteins is exaggerated, resulting in high titers of anti-CCP antibodies that are not usually present in other inflammatory conditions. Thus, anti-CCP antibodies have a high specificity for RA. Antibodies to citrullinated peptides/proteins are usually measured by enzyme-linked immunosorbent assay (ELISA) using a mixture of CCPs as the antigen. (Choice A) Anticentromere antibodies are found in the majority of patients with CREST syndrome. (Choice C) Antibodies to double-stranded DNA (anti-dsDNA) are specific for systemic lupus erythematosus. (Choice D) Rheumatoid factors are autoantibodies targeting the Fc portion of human IgG that occur in most patients with RA. Their diagnostic utility is limited by their poor specificity as they are found in approximately 10% of healthy individuals, in 30% of patients with systemic lupus erythematosus, and in nearly all patients with mixed cryoglobulinemia. (Choice E) The presence of antinuclear antibodies (ANA) is a nonspecific finding in many connective tissue disorders. Antinuclear antibodies characteristically occur in IgM form in patients with RA, but they are found less frequently than rheumatoid factors. (Choice F) Antiphospholipid antibodies are found in patients with systemic lupus erythematosus and antiphospholipid antibody syndrome. Antiphospholipid antibody syndrome causes hypercoagulability, paradoxical partial thromboplastin time (PTT) prolongation, and recurrent miscarriages (spontaneous abortions). Educational objective: Rheumatoid arthritis is characterized by symmetric polyarthritis (involving the metacarpophalangeal and proximal interphalangeal joints) with prolonged morning stiffness and associated fatigue. Antibodies to citrullinated peptides/proteins have a high specificity for the condition.

Incidentally, one of the male patients followed in the study is hospitalized with right knee pain and swelling. A sample of his synovial fluid shows negatively birefringent crystals under polarized light microscopy. To achieve rapid improvement in this patient's symptoms, therapy should be directed toward inhibiting which of the following types of cells? Eosinophils Lymphocytes Neutrophils Synovial cells Mast cells

This patient's synovial fluid analysis shows negatively birefringent crystals (ie, monosodium urate crystals) under polarized light, which is diagnostic for gouty arthritis. Neutrophils are the primary cells responsible for the intense inflammatory response seen in patients with gout. Phagocytosis of urate crystals by neutrophils causes the release of various cytokines and inflammatory mediators that lead to further neutrophil activation and chemotaxis, resulting in a positive feedback loop that amplifies the inflammatory response. Nonsteroidal anti-inflammatory drugs (NSAIDs) are first-line therapy for treating acute gouty arthritis. They inhibit prostanoid biosynthesis (eg, prostaglandins, prostacyclin, thromboxanes), exerting a broad anti-inflammatory effect that includes inhibition of neutrophils. Patients with contraindications to NSAIDs (eg, peptic ulcer disease, renal impairment) are often treated with colchicine, which impairs neutrophil migration and phagocytosis by interfering with microtubule formation. Colchicine also decreases tyrosine phosphorylation in response to monosodium urate crystals, resulting in decreased neutrophil activation. (Choice A) Eosinophils function in defense against parasitic infections and are also pathogenic in patients with asthma, allergy, hypereosinophilic syndromes, and vasculitides such as Churg-Strauss syndrome. (Choice B) Lymphocytes produce delayed-type hypersensitivity reactions that do not play a role in gout. (Choice D) Synovial cells and macrophages play a role in initiating the inflammatory response in gouty arthritis. However, targeting these cells would not eliminate the inflammatory amplification caused by neutrophils, which is the central mechanism involved in precipitating an acute gouty attack. (Choice E) Mast cell degranulation can be inhibited by medications such as cromolyn sodium, which is used in conditions such as asthma and allergic rhinitis. Educational objective: Nonsteroidal anti-inflammatory drugs (NSAIDs) are first-line therapy for treating acute gouty arthritis. They inhibit cyclooxygenase and exert a broad anti-inflammatory effect that includes inhibition of neutrophils. When NSAIDs are contraindicated (eg, peptic ulcer disease, renal impairment), colchicine is useful in the acute management of gout as it inhibits neutrophil chemotaxis and phagocytosis by preventing microtubule formation.

A 45-year-old woman comes to the office for follow-up of arthritis. For the last year, she has had fatigue, weight loss, and progressive joint pain in the hands associated with prolonged morning stiffness. Past medical history includes hypothyroidism, for which she takes levothyroxine. The patient does not use tobacco or alcohol. Examination shows doughy swelling involving multiple metacarpophalangeal joints in both hands. Following initial diagnostic testing, a multidrug treatment regimen is started. Soon after treatment begins, she develops painful mouth ulcers and nausea. Liver function tests show new aspartate transaminase and alanine transaminase elevations. Which of the following medications is most likely responsible for the adverse effects seen in this patient? Hydroxychloroquine Methotrexate Minocycline Naproxen Prednisone

This patient, a middle-aged woman with polyarthritis, morning stiffness, and systemic symptoms, has typical features of rheumatoid arthritis (RA). Methotrexate is the preferred first-line disease-modifying treatment for most patients with moderate to severe RA. Methotrexate is a folate antimetabolite that halts purine and pyrimidine synthesis through competitive inhibition of dihydrofolate reductase. Methotrexate preferentially inhibits growth of rapidly dividing cells, such as inflammatory and neoplastic cells. However, this inhibitory effect also causes toxicity to tissues with rapid cellular turnover, such as oral and gastrointestinal mucosa (ulcerations), hair follicles (alopecia), and bone marrow (pancytopenia). Methotrexate can also cause hepatotoxicity (hepatitis, fibrosis, cirrhosis) and pulmonary fibrosis. (Choice A) Hydroxychloroquine is a well-tolerated antirheumatic drug used in mild RA and systemic lupus erythematosus. Its most significant toxicity is irreversible retinopathy, and patients should have regular ophthalmologic examinations. (Choice C) Minocycline is a tetracycline antibiotic with weak antirheumatic activity. Common side effects include photosensitivity dermatitis. (Choice D) Nonsteroidal anti-inflammatory drugs (NSAIDs [eg, naproxen]) are used for initial symptom relief in RA, although they do not prevent the long-term joint damage caused by the disease. Significant side effects include gastritis/gastric ulcers and acute kidney injury. (Choice E) Glucocorticoids are used for initial treatment and acute flares of RA. Short-term side effects include insomnia and hyperglycemia. Long-term side effects are significant and include weight gain, osteoporosis, and muscle weakness; these drugs also potentiate the risk of gastric ulcers in patients taking NSAIDs. Educational objective: Methotrexate is the preferred disease-modifying treatment for patients with moderate to severe rheumatoid arthritis. Significant adverse effects include stomatitis, bone marrow suppression, and liver function abnormalities.

A 35-year-old man comes to the clinic with acute right knee pain and swelling. His symptoms have been present for a week and have moderately worsened over this period. The patient is able to bear weight but has significant pain when climbing stairs or walking for extended distances. He attempted treatment with ibuprofen, which provided prompt but only temporary relief. Past medical history is unremarkable, though the patient was seen by his primary care provider for a diarrheal illness 2 weeks before onset of the current symptoms. Examination shows a moderate-sized effusion at the right knee. Cultures of a joint aspirate reveal no bacteria. Which of the following is most likely associated with this patient's joint symptoms? C1 inhibitor deficiency High titers of antistreptolysin O antibodies Histocompatibility antigen HLA-B27 Positive serum antinuclear antibodies Positive serum rheumatoid factor

This patient, a young man with acute lower extremity arthritis and a sterile joint effusion following a gastrointestinal infection, has typical symptoms of reactive arthritis. Reactive arthritis is a spondyloarthropathy that preferentially affects HLA-B27-positive individuals. It typically presents as an asymmetric arthritis of the large joints. Common associated symptoms include conjunctivitis, urethritis, and keratoderma blennorrhagicum. Reactive arthritis is most common following infections with Campylobacter, Shigella, Salmonella, Yersinia, Chlamydia, or Bartonella. The clinical manifestations are caused by immune complexes involving bacterial antigens. However, it does not represent disseminated infection, and joint aspirates are sterile (ie, it is a 'reactive' not infectious arthritis). (Choice A) C1 inhibitor deficiency is associated with hereditary angioedema. C1 inhibitor degrades C1 and prevents excessive complement activation and inflammation. (Choice B) High titers of antistreptolysin O antibody are seen in nonpurulent complications of group A streptococcal infections, such as rheumatic fever and glomerulonephritis. (Choices D and E) Antinuclear antibody and rheumatoid factor are nonspecific markers of autoimmune disease, primarily systemic lupus erythematosus and rheumatoid arthritis, respectively. These conditions are less common in men, do not usually present with acute monoarthritis, and are not associated with recent diarrheal illness. Educational objective: Reactive arthritis is a spondyloarthropathy associated with HLA-B27 that can occur following infection with Chlamydia, Campylobacter, Salmonella, Shigella, or Yersinia. It presents with sterile arthritis due to deposition of immune complexes.

A 62-year-old woman comes to the office due to activity-related joint pain in the hands and periodic morning stiffness that lasts 10-15 minutes. The pain is moderately severe and has begun to limit her activities. The patient has attempted to treat the pain with acetaminophen, which provided only partial relief. Past medical history is notable for hypertension and diabetes mellitus. Physical examination shows firm nodules over the distal interphalangeal joints bilaterally as shown in the image below. ID=1771 Which of the following is the most likely cause of this patient's symptoms? Fibromyalgia Gout Infectious arthritis Osteoarthritis Reactive arthritis Rheumatoid arthritis

This patient, an older woman with use-dependent joint pain, has typical features of osteoarthritis (OA). Patients with OA of the hands may also have osteophyte formation leading to bony enlargement of the distal interphalangeal (DIP) and proximal interphalangeal (PIP) joints (Heberden and Bouchard nodes, respectively). OA also commonly involves the spine and the large weight-bearing joints of the lower extremities (knees, hips). Morning stiffness is more typical of the inflammatory arthritides such as rheumatoid arthritis (RA), but may be experienced for a short duration (<30 minutes) in patients with OA as well. In contrast, morning stiffness in RA may persist for hours and is often accompanied by systemic symptoms (eg, fever, weight loss). RA typically causes symmetric arthritis of the metacarpophalangeal, PIP, and other joints, but DIP joint involvement is less common (Choice F). In contrast to the bony enlargement in OA, the involved joints in RA have active synovitis, with warmth and spongy swelling. (Choice A) Fibromyalgia is characterized by diffuse musculoskeletal pain that worsens with weather changes, psychological stress, or reduced sleep. It does not cause joint deformities. (Choice B) Gout causes acute, episodic inflammatory monoarthritis that is characterized by a red, swollen, and painful joint. It usually affects the first metatarsophalangeal joint, knees, or ankles. (Choice C) Septic arthritis causes intense pain, swelling, and redness of affected joints. The knee, hip, and shoulder are most commonly involved. The finger joints are less often affected and bony joint enlargement is not typical. Moreover, systemic signs, including fever, chills, and malaise, may be present. (Choice E) Reactive arthritis is characterized by urethritis, conjunctivitis/uveitis, and arthritis (mainly of the knees, ankles, and feet) and is most common in young men. Hand joint involvement is not typical. Educational objective: Osteoarthritis of the hands is characterized by osteophyte formation leading to hard bony enlargement of the distal interphalangeal joints (Heberden nodes) and proximal interphalangeal joints (Bouchard nodes). Brief morning stiffness may be present.

A 64-year-old man comes to the emergency department due to acute knee pain. He developed redness, swelling, and severe pain affecting the right knee the day before his arrival. The patient has had 3 similar episodes in the past year that resolved quickly with over-the-counter analgesics. He has a history of polycythemia vera but has been noncompliant with phlebotomy treatments. Vital signs and cardiopulmonary examinations are normal. The right knee is red and warm. There is severe pain with movement at the joint; the other joints are grossly normal. Needle aspiration of the joint is performed. Which of the following is the most likely finding on analysis of this patient's synovial fluid? Clear fluid with few white blood cells Gram-negative diplococci Gram-positive cocci in clusters Many leukocytes with no crystals or organisms Needle-shaped, negatively birefringent cystals Numerous red blood cells Rhomboid-shaped, positively birefringent crystals

This patient, with recurrent acute monoarthritis and history of a myeloproliferative disorder, has typical features of gout. Gout occurs when there is an increased uric acid concentration in serum and synovial fluid, leading to monosodium urate crystal formation and deposition in synovium and cartilage. Gout is most common in joints where cooler temperatures favor crystal formation, especially in the distal lower extremity (eg, first metatarsophalangeal joint, ankle, knee). Uptake of urate crystals by neutrophils leads to free radical release, cytokine production, and joint inflammation. The hyperuricemia that predisposes to gout can be idiopathic (primary gout) or due to increased urate production (eg, myeloproliferative disorders) or decreased urate clearance by the kidneys. Gouty arthritis presents with acute onset of severe joint pain, warmth, erythema, and swelling. The diagnosis is established by synovial fluid analysis, which shows increased white blood cells. Monosodium urate crystals are seen under polarized light microscopy and may be present both intra- and extracellularly. They are characteristically needle-shaped and negatively birefringent (when the crystals are aligned parallel to the slow ray of the compensator, they appear yellow; when aligned perpendicular, they appear blue). (Choice A) Osteoarthritis causes chronic, progressive arthritis in the distal hands and weight-bearing joints. Synovial fluid analysis shows normal to minimal inflammatory findings. (Choices B and C) Gonococcal arthritis presents with fever, skin lesions, and inflammatory mono- or oligoarthritis. Staphylococcal septic arthritis causes a fulminant monoarthritis, usually with fever. Recurrent episodes are more characteristic of gout. (Choice D) Rheumatoid arthritis causes a chronic polyarthritis, typically involving the metacarpophalangeal joints of the hands. Synovial fluid will show a sterile inflammatory effusion. (Choice F) Nontraumatic hemarthrosis is most commonly seen in patients taking anticoagulants (eg, warfarin) or with bleeding disorders (eg, hemophilia). (Choice G) Crystal arthritis due to calcium pyrophosphate deposition (pseudogout) most commonly affects the knee and is often similar to gout. However, it is less common and not associated with myeloproliferative disorders. Educational objective: Gout results from the deposition of monosodium urate crystals in the joints and soft tissues. Under polarized light, urate crystals appear needle-shaped and negatively birefringent. Conditions that increase uric acid production or decrease uric acid clearance can increase the risk of gout.

A 34-year-old man comes to the physician due to painful urination. Physical examination shows a watery penile discharge. Urethral swabs obtained from the patient are negative for gonococcal infection. He is treated appropriately and his symptoms resolve. Two weeks later, he develops acute conjunctivitis, right knee pain, and vesicular rash on his palms and soles. This patient's condition is most likely associated with which of the following? Esophageal dysmotility Hyperparathyroidism Polymyositis Sacroliitis Tabes dorsalis

Urethritis, conjunctivitis, and mono- or oligoarticular arthritis constitute the classic triad of reactive arthritis (ReA). This seronegative (rheumatoid factor-negative) spondyloarthropathy most commonly affects patients age 20-40 and is associated with HLA-B27. Symptoms generally manifest 1-4 weeks following a primary infection causing urethritis or enteritis and are caused by an autoimmune reaction initiated by the infecting pathogen. Skin findings include keratoderma blennorrhagicum (hyperkeratotic vesicles on the palms and soles) and circinate balanitis (serpiginous annular dermatitis of the glans penis). Axial involvement, including sacroiliitis, may occur in about 20% of cases. (Choice A) Esophageal dysmotility is a classic finding in scleroderma although it can occur in other conditions. (Choice B) Hyperparathyroidism may occur in the setting of multiple endocrine neoplasia. (Choice C) Polymyositis may occur in the setting of dermatomyositis or as a separate entity. (Choice E) Tabes dorsalis is a manifestation of tertiary syphilis. Secondary syphilis causes maculopapular or pustular (but not vesicular) rash that can involve the palms and soles; it is generally associated with polyarthritis (mono-arthritis is very rare). Educational objective: The classic triad of reactive arthritis is nongonococcal urethritis, conjunctivitis, and arthritis. It is an HLA-B27-associated arthropathy that occurs within several weeks following a genitourinary or enteric infection. It belongs to the group of seronegative spondyloarthropathies (including ankylosing spondylitis) and can cause sacroiliitis in about 20% of cases.

A 3-year-old boy is evaluated for delayed developmental milestones, including short stature. There is no family history of bone conditions. The family recently immigrated to the United States. The patient was exclusively breastfed until age 6 months and has since had a varied diet of fruits, vegetables, and meats. He takes no medications or supplements. Vital signs are normal. Physical examination reveals frontal bossing, dental enamel defects, and bilateral, symmetric, bead-like enlargement of the costochondral junctions. If a histologic examination of the bone is performed, which of the following changes is most likely to be seen in this patient? Apoptotic osteoblasts Enlarged osteoclasts Excessive fibrous tissue replacement Excessive mineralized matrix Excessive unmineralized matrix

Vitamin D is derived from food and generated in the skin due to sunlight exposure (ie, conversion of 7-dehydrocholesterol to cholecalciferol). Endogenous production from sunlight accounts for approximately half of daily requirements (~200-400 IU/day [required: 600]). The dietary share is usually obtained primarily from fortified dairy/dairy-alternative foods; oily fish (eg, salmon, herring) is also an abundant source. Other foods contain vitamin D but are often insufficient to meet requirements. Vitamin D deficiency reduces intestinal absorption of calcium. Parathyroid hormone secretion increases to mobilize calcium from bone and maintain plasma calcium levels but causes renal phosphate wasting. The resulting hypophosphatemia causes impaired mineralization of bone. In children, this manifests as rickets, characterized by excessive unmineralized osteoid matrix at the epiphyseal (growth plate) cartilage. Clinical manifestations of rickets include frontal bossing, craniotabes (softened skull bones), and costochondral widening from cartilage overgrowth (rachitic rosary). Dental enamel hypoplasia can also be seen. Weight-bearing children may show lower extremity bowing (genu varum). Radiographs of growth plates (eg, distal ulna) often reveal metaphyseal plate widening and cupping. (Choice A) Glucocorticoids (eg, prednisone) cause osteoporosis because they inhibit proliferation and differentiation of osteoblast precursors and increase apoptosis of mature osteoblasts. Fragility fractures may occur, but this patient's deformities are more consistent with rickets. (Choice B) The early osteolytic phase of Paget disease of bone is characterized by proliferation of abnormally large, hypernucleated osteoclasts. Paget disease causes bone pain and deformity (eg, frontal bossing, bowing of long bones) but findings are typically focal and asymmetric and usually occur in adults age >50. (Choice C) Replacement of bone with disorganized fibrous connective tissue (fibrous dysplasia) can occur in a single bone (eg, femur) or multiple bones (polyostotic). Polyostotic fibrous dysplasia (McCune-Albright syndrome) is typically associated with café au lait macules and pituitary endocrine disorders (eg, precocious puberty, thyrotoxicosis, acromegaly). (Choice D) Osteosarcoma, the most common malignant bone tumor in children, is characterized by excessive production of mineralized bone. The lesions are typically single and would not cause short stature, frontal bossing, or enamel defects. Educational objective: Vitamin D deficiency decreases intestinal absorption of calcium. Parathyroid hormone secretion increases to maintain plasma calcium, which causes renal phosphate wasting and impaired bone mineralization. In children, this causes rickets characterized by excessive unmineralized osteoid matrix at the epiphyseal cartilage.

A 68-year-old Caucasian woman comes to the emergency department due to several hours of severe upper back pain. The patient developed the pain after experiencing a sudden jolt while driving over a pothole. She has no previous history of back pain or major trauma and has not seen a doctor in many years. The patient is retired and lives a sedentary lifestyle. She drinks a glass of wine daily and eats a healthy, balanced diet. BMI is 18.4 kg/m2. On examination, there is point tenderness over the T10 vertebra. Neurologic examination is unremarkable. X-ray of the spine reveals a T10 compression fracture. On the graph, area C represent the normal serum concentrations of calcium and parathyroid hormone. ID=982 PTH, Serum Calcium low, high high, high medium, medium low, low low, high

his patient with a thoracic compression fracture has multiple risk factors for osteoporosis (Caucasian ethnicity, female sex, postmenopausal status). Osteoporosis is characterized by decreased bone strength resulting from low bone mass and microarchitectural deterioration of bone tissue. The presence of a fragility fracture due to minimal trauma (eg, fall from a standing height) is strongly suggestive of osteoporosis. Early postmenopausal osteoporosis primarily involves cancellous bone, which is especially important for maintaining the integrity of the vertebral column. Therefore, vertebral fractures are the most common manifestation of osteoporosis. With advancing age, the cortical bone (compact bone) that makes up the shafts of long bones and outer envelopes of all bones becomes more involved, leading to an increased risk of hip fracture. In primary osteoporosis (ie, osteoporosis that is not due to an underlying medical disorder), serum calcium, phosphorus, and parathyroid hormone (PTH) levels are typically in the normal range. (Choice A) High serum PTH with low serum calcium (area A in the diagram) is usually seen in patients with renal failure or vitamin D deficiency. (Choices B and E) High calcium, high PTH (area B) is consistent with primary hyperparathyroidism. High calcium levels with low PTH (area E) would be seen in patients with PTH-independent hypercalcemia (PTH is appropriately suppressed by high calcium levels), which can occur with humoral hypercalcemia of malignancy and vitamin D toxicity. (Choice D) In patients with hypoparathyroidism, low PTH levels cause hypocalcemia. Educational objective: Patients with osteoporosis have low bone mass, resulting in increased susceptibility to fragility fractures (ie, those occurring with minimal or no trauma). In primary osteoporosis (not caused by a medical disorder), serum calcium, phosphorus, and parathyroid hormone levels are typically normal.


संबंधित स्टडी सेट्स

Intro to Digital Forensics - Midterm Review

View Set

Elinkeinoverolaki käytännössä

View Set

Physiology- Motor Nervous System

View Set